[obm-l] RE: [obm-l] Novos avanços sobre a Hipótese do Continuum

2021-07-25 Por tôpico Paulo Santa Rita
Oi Bouskela e demais
membros desta lista... obm-l !

No mínimo interessante...  Na verdade, dizer que a cardinalidade do contínuo é 
"C" é apenas uma convenção e demonstração de ignorância, pois não sabemos 
(ainda) a que álefe da sequência do Cantor este "C" corresponde...

Se "C" for igual ao terceiro álefe, isto é, ao "álefe índice dois", é 
inevitável a pergunta sobre exemplos de conjunto com a cardinalidade do segundo 
álefe... O Erdos conseguiu um resultado interessante neste sentido, se não me 
falha a memória, sobre conjunto de funções analíticas.

Outro fato notável que ilustra a (maravilhosa) intuição do Godel é a previsão 
de que esta discussão toda sobre a hipótese do contínuo levaria a um conjunto 
de axiomas tipo ZFC onde as técnicas de forcing estariam "devidamente domadas" 
e novos e produtivos axiomas seriam descobertos.

Realmente interessante o artigo que você destacou.

Por oportuno e para não fugir ao espírito desta lista, alguém aqui conhece uma 
prova elementar (sem usar função zeta) da identidade entre a função seno e o 
produto de Euler  ?

sen(x) = x*[1  -  (x/pi)^2]*[1  -  (x/(2*pi))^2]*[1  -  (x/(3*pi))^2]*...*[1  - 
 (x/n*pi)^2]*...

Um abraço a Todos !
Paulo Santa Rita
1250721151E


Um abraço a todos !



De: owner-ob...@mat.puc-rio.br  em nome de 
bousk...@gmail.com 
Enviado: domingo, 18 de julho de 2021 17:06
Para: obm-l@mat.puc-rio.br 
Assunto: [obm-l] Novos avanços sobre a Hipótese do Continuum


Recebi da Quanta Magazine o artigo identificado a seguir:



How Many Numbers Exist? Infinity Proof Moves Math Closer to an Answer

For 50 years, mathematicians have believed that the total number of real 
numbers is unknowable. A new proof suggests otherwise.



Este artigo foi escrito por Natalie Wolchover em 15JUL2021 e, particularmente, 
o achei interessantíssimo.

--
Esta mensagem foi verificada pelo sistema de antivírus e
acredita-se estar livre de perigo.

-- 
Esta mensagem foi verificada pelo sistema de antivírus e
 acredita-se estar livre de perigo.



RE: [obm-l] {Filename?} Problema Interessante de Geometria

2015-06-09 Por tôpico Paulo Santa Rita
Ola Fabiola, Prof da Fabiola e carissimo Ralph,
Vou fazer um esboço de prova aqui. Considere os triângulos OPiPi+1 e OQiQi+1. 
Como as areas são iguais e PiPi+1 e igual a QiQi+1 e, além disso, PiPi+1 é 
paralelo a QiQi+1 então as distancias OP ( de O ate PiPi+1) e OQ ( de O até 
QiQi+1 ) são iguais, vale dizer :
1) QiQi+1 está na reta onde esta PiPi+1 ou2) QiQi+1 está na reta diametralmente 
oposta a reta que contem PiPi+1. Vou aqui esquecer o caso 2) que tera um 
raciocinio identico.Consideremos então o caso 1. Eu afirmo que QiQi+1 coincide 
com PiPi+1 ! Por que ? Porque se não coincidirem traçamos Qi+1Qi+2 paralelo a 
Pi+1Pi+2 e a distancia de O a Qi+1Qi+2 sera diferente da distancia de O a 
Pi+1Pi+2  e os triangulos necessariamente deverão ter areas diferentes ... 
absurdo ! 
Assim, QiQi+1 coincide com PiPi+1. E como ste raciocinio vale para qualquer 
i=1...nentão segue que os poligonos são congruentes.
Penso que é so aperfeiçoar esta linha de raciocinio qu o problema sai fácil
UM abração a todos !




Date: Mon, 8 Jun 2015 21:03:00 -0300
Subject: [obm-l] {Filename?} Problema Interessante de Geometria
From: ralp...@gmail.com
To: obm-l@mat.puc-rio.br

Ola a todos.
Eu e minha aluna de Mestrado Fabiola encontramos um problema bem facil de 
enunciar que esclareceria um ponto da dissertacao de mestrado dela... No 
entanto, a gente soh encontrou umas solucoes bem complicadas na literatura, e 
mesmo assim parecem ser apenas para alguns casos particulares simetricos... 
Entao coloco aqui -- quem tiver uma solucao elegante ganha um agradecimento na 
dissertacao! :) :)
(Eu pensei ateh em sugerir esse problema para alguma OBM, mas como ainda nao 
sei resolver e acabei mostrando a alguns alunos, vou soltar logo ele aqui.)
"Sao dados dois poligonos convexos P1P2...Pn e Q1Q2...Qn (onde n>4) contendo a 
origem O em seu interior. Sabe-se que:-- Eles tem lados respectivamente 
paralelos (isto eh, PiP_{i+1} // QiQ_{i+1} para i=1,2,...,n, indices modulo 
n);-- Triangulos com vertice em O e um lado do poligono tem areas 
respectivamente iguais (isto eh, Area(OPiP_{i+1}) = Area(OQiQ_{i+1}) para 
i=1,2,...n, indices modulo n).Pergunta-se: os poligonos tem que ser 
congruentes?"
Quem quiser brincar, vide o Geogebra anexo que ilustra o caso n=6 (fiz uma 
copia de Q longe da origem para facilitar a visualizacao -- a "origem" para Q 
eh O_1). Pode brincar como quiser com os Q's, e com P_1 -- os outros pontos sao 
calculados para satisfazer as condicoes acima... Mas alguem consegue fazer o 
poligono P fechar (isto eh, P1=P7) sem que ele seja congruente ao Q (mas 
mantendo ambos convexos e mantendo a origem O dentro de P?)
Nota: se n=4, dois paralelogramos distintos de mesma area centrados na origem 
sao contra-exemplo!

Abraco, Ralph.
-- 
Esta mensagem foi verificada pelo sistema de antivírus e
 acredita-se estar livre de perigo.



[obm-l] RE: [obm-l] RE: [obm-l] RE: [obm-l] Todo polígono convexo é plano?

2015-02-08 Por tôpico Paulo Santa Rita
Oi Pedro.
Foi o que eu disse : convexidade é relativo a conjunto de pontos. Assim, 
existem poliedros convexo, tais como um paralelepípedo e figuras convexas no 
R^4, R^5, ... R^n.

From: brped...@hotmail.com
To: obm-l@mat.puc-rio.br
Subject: [obm-l] RE: [obm-l] RE: [obm-l] Todo polígono convexo é plano?
Date: Sun, 8 Feb 2015 04:41:10 +0300




Caro Paulo Santa Rita e demais colegas,

Existem também figuras não planas convexas. Por exemplo: os poliedros convexos.
Parece-me contudo que os polígonos convexos são, necessariamente, planos.
Será que estou certo?
Abraços do Pedro Chaves.
_

From: paulo.santar...@live.com
To: obm-l@mat.puc-rio.br
Subject: [obm-l] RE: [obm-l] Todo polígono convexo é plano?
Date: Fri, 6 Feb 2015 09:13:36 -0200

Oi Pedro e demais membros
desta lista de discussão ... obm-l,

 Me parece que a *ideia de convexidade* é relativa a conjunto de pontos, em 
particular, a conjunto de pontos de um plano :

"Um conjunto V de pontos de um plano é convexo se qualquer segmento de reta que 
liga dois de seus pontos está inteiramente contido em V"

Se A e B são pontos de R^n e "t" é um real pertencente a [0,1], o conjunto de 
todos os pontos da forma At+(1-t)B é o segmento de reta que liga A a B. Com 
isso é possível generalizar o conceito de convexidade.

Voce não esta querendo falar de conexidade ?

Um Abraço a todos
PSR,60602140912

> From: brped...@hotmail.com
> To: obm-l@mat.puc-rio.br
> Subject: [obm-l] Todo polígono convexo é plano? 
> Date: Wed, 4 Feb 2015 20:36:47 +0300
> 
> Caros Colegas,
> 
> Existem polígonos reversos convexos? Ou qualquer polígono convexo é, 
> necessariamente, plano?
> 
> Abraços do Pedro Chaves!
> __
> 
> 
> 
> -- 
> Esta mensagem foi verificada pelo sistema de antivírus e
> acredita-se estar livre de perigo.
> 
> 
> =
> Instruções para entrar na lista, sair da lista e usar a lista em
> http://www.mat.puc-rio.br/~obmlistas/obm-l.html
> =

-- 
Esta mensagem foi verificada pelo sistema de antivírus e 
acredita-se estar livre de perigo.
--

Esta mensagem foi verificada pelo sistema de antivírus e 

 acredita-se estar livre de perigo.   
-- 
Esta mensagem foi verificada pelo sistema de antivírus e
 acredita-se estar livre de perigo.



[obm-l] RE: [obm-l] Todo polígono convexo é plano?

2015-02-06 Por tôpico Paulo Santa Rita
Oi Pedro e demais membrosdesta lista de discussão ... obm-l,
 Me parece que a *ideia de convexidade* é relativa a conjunto de pontos, em 
particular, a conjunto de pontos de um plano :
"Um conjunto V de pontos de um plano é convexo se qualquer segmento de reta que 
liga dois de seus pontos está inteiramente contido em V"
Se A e B são pontos de R^n e "t" é um real pertencente a [0,1], o conjunto de 
todos os pontos da forma At+(1-t)B é o segmento de reta que liga A a B. Com 
isso é possível generalizar o conceito de convexidade.
Voce não esta querendo falar de conexidade ?
Um Abraço a todosPSR,60602140912
> From: brped...@hotmail.com> To: obm-l@mat.puc-rio.br
> Subject: [obm-l] Todo polígono convexo é plano? 
> Date: Wed, 4 Feb 2015 20:36:47 +0300
> 
> Caros Colegas,
> 
> Existem polígonos reversos convexos? Ou qualquer polígono convexo é, 
> necessariamente, plano?
> 
> Abraços do Pedro Chaves!
> __
> 
> 
> 
> -- 
> Esta mensagem foi verificada pelo sistema de antivírus e
>  acredita-se estar livre de perigo.
> 
> 
> =
> Instruções para entrar na lista, sair da lista e usar a lista em
> http://www.mat.puc-rio.br/~obmlistas/obm-l.html
> =
  
-- 
Esta mensagem foi verificada pelo sistema de antivírus e
 acredita-se estar livre de perigo.



[obm-l] arranjos caóticos - o problema

2015-01-20 Por tôpico Paulo Santa Rita
Caro Bouskela e demais membrosde lista de discussão ... obm-l,
Se não me falha a memória, foi na transição entre os séculos XVII e XVIII que o 
*Nicolau Bernoulli  ( E onde  está o Prof Nicolau Saldanha, também sumido ? )  
propôs ao Euler o famoso problemas das "cartas embaralhadas" ( permutações 
caóticas ):
PROBLEMA JÁ RESOLVIDO  : De quantas maneiras uma pessoa pode colocar N cartas 
em N envelopes de forma que nenhuma carta chegue ao destinatário correto ?
Parece que tanto o Euler quanto o Nicolau resolveram o problema. E deste 
trabalho resultou a já bem conhecida expressão :
N! = N! (  (1/(2!)) - (1/(3!)) + ... + ( ((-1)^N)*(1/(N!)) )  )(1)
que nos dá o total de permutações caóticas de N objetos. Registro aqui que :
1) "permutações caóticas" são também conhecidas como 
"desarranjos"(derangements) 2)  A notação N! é também conhecida como  
"subfatorial de N" 
Note que o trabalho do Euler e do Nicolau neste particular caso está obviamente 
incompleto, pois poderíamos perguntar :
PROBLEMA PARA RESOLVER : De quantas maneiras uma pessoa pode colocar Ncartas em 
N envelopes de forma que exatamente P delas (0 =< P =< N )  não chegueao seu 
destinatário correto ?
Note que a expressão fechada ( fórmula ) que vamos obter, no caso em que P=N, 
deve nos fornecer a fórmula dada acima e já descoberta pelo Euler e Nicolau. 
Até onde eu sei, este problema está "em aberto". Veja abaixo que o verbete 
sobre"partial derangements"  do http://Mathworld.wolfram.come é pobre, 
incompleto e não resolve a questão. Nós estamos pensando aqui em uma "formula 
fechada".
Um abraço a todosPSR,32001140C2A








  
-- 
Esta mensagem foi verificada pelo sistema de antivírus e
 acredita-se estar livre de perigo.



RE: RES: [obm-l] Sumidos

2015-01-20 Por tôpico Paulo Santa Rita
Caríssimo Bouskela,
Olha eu aqui  ! Espera "um pouquinho" que vou voltar a participar. Aproveito a 
oportunidade para sugerir a leitura de um artigo de divulgação científica que 
aIolanda me convenceu  publicar. Prometi a ela que tentaria falar de 
MecânicaQuântica SEM USAR MATEMÁTICA.
O propósito da revista é unir o subjetivo (arte) ao objetivo (ciência)> Nada 
melhorque falar da interpretação de Copenhagem, que destroi a objetividade 
clássica tacitamente admitida e introdução o (a consciência do) observador  (no 
momentoonde há o colapso da função de onda )
Será que consegui ? Só os colegas podem dizer. O link do artigo é (pag 5 ) : 
http://issuu.com/iolandabrazao/docs/revista_encenao_-_janeiro_2015
Um abraço a todos !PSR, 32001140A39


From: bousk...@gmail.com
To: obm-l@mat.puc-rio.br
Subject: RES: [obm-l] Sumidos
Date: Thu, 18 Dec 2014 10:45:45 -0200

Olá! É bom ter todos de volta! Saudações! Mas… cadê o Nehab? Cadê o Santa Rita? 
Cadê o Rogerio Ponce? Cadê tantos outros? Será que viraram Papai Noel (não sei 
qual é o plural de “Papai Noel”)? Feliz Natal! Feliz 2015! (Peço que não 
entendam 2015! como o fatorial de 2015)Albert Bouskelábousk...@gmail.com De: 
owner-ob...@mat.puc-rio.br [mailto:owner-ob...@mat.puc-rio.br] Em nome de João 
Maldonado
Enviada em: quinta-feira, 18 de dezembro de 2014 02:19
Para: obm-l@mat.puc-rio.br
Assunto: RE: [obm-l] Sumidos Haha, tbm tava com saudade das suas questões 
marcone :)

O ano tava muito corrido, não deu pra acompanhar muito aqui...
Agora que peguei férias provavelmente vou ter mais tempo

[]'s
JoãoFrom: marconeborge...@hotmail.com
To: obm-l@mat.puc-rio.br
Subject: [obm-l] Sumidos
Date: Wed, 17 Dec 2014 23:36:32 +João Maldonado e outros sumidos fizeram 
falta, 
-- 
Esta mensagem foi verificada pelo sistema de antivírus e 
acredita-se estar livre de perigo.
-- 
Esta mensagem foi verificada pelo sistema de antivírus e 
acredita-se estar livre de perigo. 
--

Esta mensagem foi verificada pelo sistema de antivírus e 

 acredita-se estar livre de perigo.   
-- 
Esta mensagem foi verificada pelo sistema de antivírus e
 acredita-se estar livre de perigo.



[obm-l] RE: [obm-l] RE: [obm-l] Re: [obm-l] Pontuação mínima em campeonato

2011-05-26 Por tôpico Paulo Santa Rita

oi Marcone e demais colegasdesta lista ... OBM-L,
Voce deve agradecer ao Ralph, porque eu não li direito a sua questão e terminei 
por abordar o problema da faixa de promoçãoe não de rebaixamento. Enfim, tratei 
de uma outra questão.
Um AbraçãoPSR,52605110707

From: marconeborge...@hotmail.com
To: obm-l@mat.puc-rio.br
Subject: [obm-l] RE: [obm-l] Re: [obm-l] Pontuação mínima em campeonato
Date: Thu, 26 May 2011 05:17:56 +








Oi,Paulo .Oi, Ralph.Muito obrigado.
 



Date: Wed, 25 May 2011 14:59:48 -0300
Subject: [obm-l] Re: [obm-l] Pontuação mínima em campeonato
From: ralp...@gmail.com
To: obm-l@mat.puc-rio.br


Se fosse apenas um turno, era mais difícil. Com turno e returno, é mais 
simples, e é generalizável...
 
A chave é olhar para os 17 melhores times, isto é, para os 17 times que 
terminaram (terminariam, terminarão?) o campeonato com a melhor posição.
 
Quantas partidas incluem pelo menos um desses 17? São 17x16 que eles jogam 
entre si, mais 17x3x2 que eles disputaram contra os 3 "piores", num total de 
17x22 partidas. Isto dá um total de 17x22x3 pontos em disputa por estes 17 
times.
 
Então, pelo menos um desses 17 times terá 22x3=66 pontos ou menos. Portanto, 67 
pontos são com certeza suficientes para você se livrar do rebaixamento.
 
--//--
 
Agora falta ver que 66 pontos não garante nada. De fato, você pode imaginar uma 
situação em que:
i) Nas partidas em que esses 17 times jogaram entre si (turno e returno), o 
"mandante" sempre ganha.
ii) Nas partidas em que esses 17 times jogaram com os 3 piores, os 17 sempre 
ganham.
(iii) Faça o que você quiser com as partidas que os 3 piores jogaram entre si, 
não interessa.)
 
Então todos esses 17 times teriam a mesma pontuação: 16x3+3x3x2=66. Com todos 
eles empatados, alguém com 66 seria rebaixado. Assim, 66 não é garantia de 
ficar na "série A".
 
---///---
 
Então: 67 pontos (bom, antes de o campeonato começar, e independente do 
critério de desempate) é o que você precisa para garantir não-rebaixamento.
 
Abraço,
Ralph

2011/5/25 marcone augusto araújo borges 


Como calcular o mínimo de pontos para uma equipe estar livre do rebaixamento 
(independente de qualquer critério de desempate)em um campeonato de 20 times em 
que os quatro últimos colocados são rebaixados?Cada time enfrenta seus 19 
adversários,jogando 2 vezes com cada um deles e a vitória vale 3 pontos,o 
empate vale 1 ponto e a derrota,zero.
É muito complicado?   

[obm-l] RE: [obm-l] Re: [obm-l] RE: [obm-l] Re: [obm-l] RE: [obm-l] Re: [obm-l] [obm-l] RE: [obm-l] Número de partições de um conjunto

2011-05-25 Por tôpico Paulo Santa Rita

Olá Rogério e demais colegas desta lista ... OBM-L,
Esta resposta está errada, pois ela pressupõe que as soluções do problema 
anterior podem ser agrupadas em grupos de 4!=24soluções, o que só ocorre quando 
a solução e formada por conjuntos dois a dois distintos. Por exemplo,
{ {1B,1P}, {1P,1A}, {1B,1A}, {6B,8P,13A} }
é uma solução no primeiro problema e qualquer uma das suas 4!=24 também são, 
formando portanto 24 soluções distintasque podem ser agrupadas em uma única 
partição ( na qual a ordem dos conjuntos é irrelevante ), a saber :
{ {1B,1P}, {1P,1A}, {1B,1A}, {6B,8P,13A} } que será uma única solução para o 
segundo problema.
Mas agora considere a solução do primeiro problema :
{ {1B,1P},{1P,1A},{1B,1P},{6B,9P,14A} }
Devido a igualdade entre o primeiro e terceiro conjuntos, não há 4!=24 
permutações duas a duas distintas que podem seragrupadas para formar a partição 
( solução do segundo problema ) seguinte :
{ {1B,1P},{1P,1A},{1B,1P},{6B,9P,14A} } que é uma solução válida para o segundo 
problema.
Um AbraçoPSR,42505110B2A
Date: Wed, 25 May 2011 11:05:27 -0300
Subject: [obm-l] Re: [obm-l] RE: [obm-l] Re: [obm-l] RE: [obm-l] Re: [obm-l] 
[obm-l] RE: [obm-l] Número de partições de um conjunto
From: abrlw...@gmail.com
To: obm-l@mat.puc-rio.br

Ola' Paulo e colegas da lista,

para este novo problema basta dividirmos a solução do problema anterior pelo 
numero de permutacoes entre os participantes.
Ou seja, basta dividir o resultado anterior por 4! = 24. 


[]'s
Rogerio Ponce.

PS: enviei para a lista a seguinte correcao:

  As brancas podem ser divididas de binom( 11 , 3 ) = 165 formas diferentes.
  As pretas podem ser divididas de binom( 13 , 3 ) = 286 formas diferentes.

  E as azuis podem ser divididas de binom( 18 , 3 ) = 816 formas diferentes.


  Logo, ha' 165*286*816 formas diferentes de se distribuir todas bolas entre 4 
pessoas.






Em 25 de maio de 2011 08:46, Paulo Santa Rita  
escreveu:






Ola Rogerio e demaiscolegas desta lista ... OBM-L,
Agora entendi. Esta solução está correta :
Se Y é o total de bolas de uma mesma cor e P1, P2, ..., Pn são as pessoas, 
então consideramos as
soluções inteiras nao-negativas da equação linear
X1 + X2 + ... + Xn = Y
E para cada solução (x1, x2, ...,xn ) desta equação consideramos que x1 será o 
total de bolas da
cor Y que serão dadas a pessoa P1, x2 será o total de bolas da cor Y que serão 
dadas a pessoa P2e assim sucessivamente. 
O produto entre as quantidades de soluções para cada cor será a resposta.

Agora, considere que nesta solução nos sabemos a quem será dadas as bolas, ou 
seja, existem aspessoas P1, P2, ..., Pn. E se o nosso interesse fosse somente 
em montar partiçoes de um conjunto
com elementos repetidos ?
"Num conjunto A existem :
8 bolas brancas, iguais e indistinguiveis entre si10 bolas prestas, iguais e 
indistinguiveis entre si
15 bolas azuis, iguais e indistinguiveis entre si
De quantas maneiras podemos particionar o conjunto A em 4 conjuntos ?"
No problema anterior { A,{},{},{}} e {{},{},{},A} são autenticas e corretas
soluções distintas. Neste agora, não.
Um AbraçoPSR,425051108A1

Date: Wed, 25 May 2011 05:19:03 -0300
Subject: [obm-l] Re: [obm-l] RE: [obm-l] Re: [obm-l] [obm-l] RE: [obm-l] Número 
de partições de um conjunto

From: abrlw...@gmail.com
To: obm-l@mat.puc-rio.br

Ola' Paulo e colegas da lista,
o problema e' encontrar a quantidade de divisoes de 8 bolas brancas, 10 pretas 
e 15 azuis entre 4 pessoas.


Para isso, basta multiplicarmos a quantidade de formas de se dividir as bolas 
de cada cor entre as pessoas.

Para as brancas, por exemplo, equivale a encontrarmos o numero de solucoes nao 
negativas da equacao:
X1 + X2 + X3 + X4 = 8
e assim por diante.

Lembrando que o numero de solucoes nao negativas de
   X1+...+Xn =  p


   e' binom(n-1+p,n-1), obtemos o seguinte:

As brancas podem ser divididas de binom( 10 , 2 ) = 45 formas diferentes.
As pretas podem ser divididas de binom( 12 , 2 ) = 66 formas diferentes.
E as azuis podem ser divididas de binom( 17 , 2 ) = 136 formas diferentes.



Logo, ha' 45*66*136 formas diferentes de se distribuir todas bolas entre 4 
pessoas.

[]'s
Rogerio Ponce


Em 25 de maio de 2011 00:38, Paulo Santa Rita  
escreveu:







Oi Willy e Rogerio e demaiscolegas desta lista ... OBM-L,
Não consigo entender o raciocínio de vocês. Vejam se é isso que vocês estão 
falando :
Uma das duas pessoas ( digamos, o "José" ) pode receber 


1) 0,1,2, ..., 8 bolas brancas. Seja A o conjunto dessas possibilidades 
2) 0,1,2,...,10 bolas pretas. Seja B o conjunto dessas possibilidades3) 
0,1,2,...,15 bolas azuis. Seja C o conjunto dessas possibilidades


A cardinalidade do produto cartesianos AxBxC encerra o total das possíveis 
maneiras de José receber as bolas. O restante é a parte da outra pessoa, 
digamos, da "Maria". Assim, uma 3-upla (0,1,4) significa que Jose recebeu 1 
bola preta e 4 azuis, ficando a Maria, 

port

[obm-l] RE: [obm-l] Re: [obm-l] RE: [obm-l] Re: [obm-l] [obm-l] RE: [obm-l] Número de partições de um conjunto

2011-05-25 Por tôpico Paulo Santa Rita

Ola Rogerio e demaiscolegas desta lista ... OBM-L,
Agora entendi. Esta solução está correta :
Se Y é o total de bolas de uma mesma cor e P1, P2, ..., Pn são as pessoas, 
então consideramos assoluções inteiras nao-negativas da equação linear
X1 + X2 + ... + Xn = Y
E para cada solução (x1, x2, ...,xn ) desta equação consideramos que x1 será o 
total de bolas dacor Y que serão dadas a pessoa P1, x2 será o total de bolas da 
cor Y que serão dadas a pessoa P2e assim sucessivamente. 
O produto entre as quantidades de soluções para cada cor será a resposta.
Agora, considere que nesta solução nos sabemos a quem será dadas as bolas, ou 
seja, existem aspessoas P1, P2, ..., Pn. E se o nosso interesse fosse somente 
em montar partiçoes de um conjuntocom elementos repetidos ?
"Num conjunto A existem :
8 bolas brancas, iguais e indistinguiveis entre si10 bolas prestas, iguais e 
indistinguiveis entre si15 bolas azuis, iguais e indistinguiveis entre si
De quantas maneiras podemos particionar o conjunto A em 4 conjuntos ?"
No problema anterior { A,{},{},{}} e {{},{},{},A} são autenticas e 
corretassoluções distintas. Neste agora, não.
Um AbraçoPSR,425051108A1

Date: Wed, 25 May 2011 05:19:03 -0300
Subject: [obm-l] Re: [obm-l] RE: [obm-l] Re: [obm-l] [obm-l] RE: [obm-l] Número 
de partições de um conjunto
From: abrlw...@gmail.com
To: obm-l@mat.puc-rio.br

Ola' Paulo e colegas da lista,
o problema e' encontrar a quantidade de divisoes de 8 bolas brancas, 10 pretas 
e 15 azuis entre 4 pessoas.

Para isso, basta multiplicarmos a quantidade de formas de se dividir as bolas 
de cada cor entre as pessoas.

Para as brancas, por exemplo, equivale a encontrarmos o numero de solucoes nao 
negativas da equacao:
X1 + X2 + X3 + X4 = 8
e assim por diante.

Lembrando que o numero de solucoes nao negativas de
   X1+...+Xn =  p

   e' binom(n-1+p,n-1), obtemos o seguinte:

As brancas podem ser divididas de binom( 10 , 2 ) = 45 formas diferentes.
As pretas podem ser divididas de binom( 12 , 2 ) = 66 formas diferentes.
E as azuis podem ser divididas de binom( 17 , 2 ) = 136 formas diferentes.


Logo, ha' 45*66*136 formas diferentes de se distribuir todas bolas entre 4 
pessoas.

[]'s
Rogerio Ponce


Em 25 de maio de 2011 00:38, Paulo Santa Rita  
escreveu:






Oi Willy e Rogerio e demaiscolegas desta lista ... OBM-L,
Não consigo entender o raciocínio de vocês. Vejam se é isso que vocês estão 
falando :
Uma das duas pessoas ( digamos, o "José" ) pode receber 

1) 0,1,2, ..., 8 bolas brancas. Seja A o conjunto dessas possibilidades 
2) 0,1,2,...,10 bolas pretas. Seja B o conjunto dessas possibilidades3) 
0,1,2,...,15 bolas azuis. Seja C o conjunto dessas possibilidades

A cardinalidade do produto cartesianos AxBxC encerra o total das possíveis 
maneiras de José receber as bolas. O restante é a parte da outra pessoa, 
digamos, da "Maria". Assim, uma 3-upla (0,1,4) significa que Jose recebeu 1 
bola preta e 4 azuis, ficando a Maria, 
portanto, com (8-0,10-1,15-4)=(8,9,11), ou seja, com 8 brancas, 9 pretas e 11 
azuis.
Se esse é o raciocínio, então ele está certo. NESTE CASO PARTICULAR !
Com mais pessoas - se entendi o que vocês disseram - o simples uso de 
combinações não resolve. Por exemplo, ao tomar Binom(11,3), 
estaremos considerando conjuntos identicos de 3 bolas brancas como se fossem 
distintos ...
Confirmem se eu realmente entendi como vocês pensaram.  
Um abração
PSR,425051100A1




Date: Tue, 24 May 2011 18:29:40 -0300
Subject: [obm-l] Re: [obm-l] [obm-l] RE: [obm-l] Número de partições de um 
conjunto

From: wgapetre...@gmail.com
To: obm-l@mat.puc-rio.br



2011/5/23 Rogerio Ponce 


Ola' Paulo e colegas da lista,
minha sugestao e' calcular de quantas formas podemos dividir as bolas de cada 
cor ( --> #solucoes nao negativas), e multiplicar tudo no final.

[]'s
Rogerio Ponce



Isso me parece ser a maneira mais simplesExistem 9 maneiras de se dividir 8 
bolas identicas entre duas pessoas (e C(11,3) de dividir entre 4 pessoas). Faz 
a mesma coisa para as demais e depois multiplica.

Obtemos 9*11*16, para 2 pessoas e obtemos C(11,3)*C(18,3)*C(13,3) para as 4 
pessoas.  

  

[obm-l] RE: [obm-l] Combinatória

2011-05-25 Por tôpico Paulo Santa Rita

Ola Marcone e demaiscolegas desta lista ... OBM-L,
Ou eu entendi mal ou esta questão é muito simplória para figurar aqui. Deveria 
ser postada num desses Sites de Vestibular que há aosmontes ai pela internet. 
Aqui nós queremos Matemática Olímpica, aquelas questões que exigem reflexão e 
criatividade.
Um AbraçoPSR,4250511094A

From: marconeborge...@hotmail.com
To: obm-l@mat.puc-rio.br
Subject: [obm-l] Combinatória
Date: Wed, 25 May 2011 10:59:32 +








Determinar a quantidade de sequências de n termos cujos termos pertencem ao 
conjunto {0,1,2} que possuem um número ímpar de zeros.Alguem poderia ajudar?
  

[obm-l] RE: [obm-l] FW: Pontuação mínima em campeonato

2011-05-25 Por tôpico Paulo Santa Rita

Olá Marcone e demais colegas desta lista ... OBM-L,
Muito complicado porque não há resposta para o seu problema e a formulação está 
errada, pois :
1) Existir algum critério de desempate é necessário2) Esta pontuação mínima 
depende do "ponto" onde está o campeonato.
Explico.
Para facilitar a sua visualização, considere 8 clubes que disputam 2 vagas em 
um campeonato de1 turno. Pode ocorrer que no final do campeonato 3 deles tenham 
tido um único empate e vencidoos demais jogos. Sejam E1, E2 e E3 estas equipes. 
Pode ocorrer o seguinte :
E1 empatou com E2E2 empatou com E3E3 empatou com E1
Cada uma dessas equipes tem exatamente 4 pontos perdidos. Todas as demais 
equipes tem, aomenos, 9 pontos perdidos ( pois, ao menos, perderam para E1, E2 
e E3 ). Logo, E1,  E2  e  E3terminam empatados em primeiro lugar. Como não há 
critério de desempate, como escolherquais serão as duas equipes que terão 
direito as duas vagas ?
Isso mostra que um critério de desempate é necessário.
Suponha agora que os 8 clubes disputam 3 vagas e que há um critério de 
desempate. ANTESDO CAMPEONATO COMEÇAR é correto dizer que o número maximo de 
pontos que podemser perdidos para assegurar uma das vagas é 4, isto é, 5 ou 
mais pontos perdidos NÃO ASSEGURAuma das vagas ( verifique )
Entretanto, se ao final da primeira rodada todos os jogos terminarem empatados, 
os numerosacima mudam, ou seja, a quantidade de pontos que voce procura DEPENDE 
DA RODADA, eleé uma função do ponto em que está o campeonato.
PROBLEMA1) . Seja N o número de clubes e P o número de vagas, P < [N/2], em um 
campeonato de turno único. Mostre que se um clube, ao final do campeonato, 
tiver D derrotas e E empates, entãoo número máximo de outros clubes que podem 
ter uma pontuação maior ou igual a dele é 2*D + E
OBS1 : [N/2] -> função pisoOBS1 : Admita que há critérios de desempate
PROBLEMA 2) Generalize o problema acima para o caso de campeonatos com mais de 
um turno.
Um AbraçoPSR,42505110940
From: marconeborge...@hotmail.com
To: obm-l@mat.puc-rio.br
Subject: [obm-l] FW: Pontuação mínima em campeonato
Date: Wed, 25 May 2011 11:58:21 +









 



From: marconeborge...@hotmail.com
To: obm-l@mat.puc-rio.br
Subject: Pontuação mínima em campeonato 
Date: Wed, 25 May 2011 10:42:02 +




Como calcular o mínimo de pontos para uma equipe estar livre do rebaixamento 
(independente de qualquer critério de desempate)em um campeonato de 20 times em 
que os quatro últimos colocados são rebaixados?Cada time enfrenta seus 19 
adversários,jogando 2 vezes com cada um deles e a vitória vale 3 pontos,o 
empate vale 1 ponto e a derrota,zero.
É muito complicado?
 
  

[obm-l] RE: [obm-l] Re: [obm-l] [obm-l] RE: [obm-l] Número de partições de um conjunto

2011-05-24 Por tôpico Paulo Santa Rita

Oi Willy e Rogerio e demaiscolegas desta lista ... OBM-L,
Não consigo entender o raciocínio de vocês. Vejam se é isso que vocês estão 
falando :
Uma das duas pessoas ( digamos, o "José" ) pode receber 
1) 0,1,2, ..., 8 bolas brancas. Seja A o conjunto dessas possibilidades 
2) 0,1,2,...,10 bolas pretas. Seja B o conjunto dessas possibilidades3) 
0,1,2,...,15 bolas azuis. Seja C o conjunto dessas possibilidades
A cardinalidade do produto cartesianos AxBxC encerra o total das possíveis 
maneiras de José receber as bolas. O restante é a parte da outra pessoa, 
digamos, da "Maria". Assim, uma 3-upla (0,1,4) significa que Jose recebeu 1 
bola preta e 4 azuis, ficando a Maria, portanto, com (8-0,10-1,15-4)=(8,9,11), 
ou seja, com 8 brancas, 9 pretas e 11 azuis.
Se esse é o raciocínio, então ele está certo. NESTE CASO PARTICULAR !
Com mais pessoas - se entendi o que vocês disseram - o simples uso de 
combinações não resolve. Por exemplo, ao tomar Binom(11,3), estaremos 
considerando conjuntos identicos de 3 bolas brancas como se fossem distintos ...
Confirmem se eu realmente entendi como vocês pensaram.  
Um abraçãoPSR,425051100A1




Date: Tue, 24 May 2011 18:29:40 -0300
Subject: [obm-l] Re: [obm-l] [obm-l] RE: [obm-l] Número de partições de um 
conjunto
From: wgapetre...@gmail.com
To: obm-l@mat.puc-rio.br


2011/5/23 Rogerio Ponce 

Ola' Paulo e colegas da lista,
minha sugestao e' calcular de quantas formas podemos dividir as bolas de cada 
cor ( --> #solucoes nao negativas), e multiplicar tudo no final.

[]'s
Rogerio Ponce


Isso me parece ser a maneira mais simplesExistem 9 maneiras de se dividir 8 
bolas identicas entre duas pessoas (e C(11,3) de dividir entre 4 pessoas). Faz 
a mesma coisa para as demais e depois multiplica.
Obtemos 9*11*16, para 2 pessoas e obtemos C(11,3)*C(18,3)*C(13,3) para as 4 
pessoas.  

RE: [obm-l] [obm-l] RE: [obm-l] Número de partições de um conjunto

2011-05-24 Por tôpico Paulo Santa Rita

Oi Rogério e demais colegas desta lista ... OBM-L,
Não consegui entender a sua sugestão. Entretanto, dentre as diversas maneiras 
de resolver que conheço há uma cuja estruturase assemelha, a saber :
1) Como vamos dividir 33 bolas entre duas pessoas então basta determinar 
quantas vamos dar a uma particular pessoa,pois o que restar será 
necessariamente da segunda pessoa. Para uma pessoa particular podemos dar 
0,1,2,3,...,33bolas2) Fixada uma das opções de doação acima, digamos, K, 
precisamos escolher K bolas do total disponível. Seja portantoP as bolas 
pretas, B as brancas e A as azuis. Devemos ter :
A + B + P = K
3) As solucoes inteiras não negativas da equação acima são as diversas maneiras 
de dar K bolas a uma das pessoas ( eportanto, a maneira de dar 33-K bolas a 
outra )
4) Note que a equação anterior precisa ser trata com cuidado se K > 8, pois 
dispomos apenas de 8 bolas brancas eportanto não podemos considerar as soluções 
com B>8; igualmente, temos que tomar algum cuidado com assoluções em que K>15, 
pois so dispomos de 15 bolas azuis.
5) Em síntese, para cada K em {0,1,2,...,33} as soluções de A + B + P = K 
constituem as maneiras de doar Kbolas a uma particular pessoa ( e "33-K" a 
outra ). Toda a dificuldade do problema consiste em saber como cuidardos 
intervalos de K's : 0 =< K =< 8,   9=< K =< 15   e   16= #solucoes nao negativas), e multiplicar tudo no final.

[]'s
Rogerio Ponce




Em 22 de maio de 2011 19:44, Paulo Santa Rita  
escreveu:






Oi Pedro e demaiscolegas desta lista ... OBM-L,
Este problema de dividir um conjunto em grupos de 2 ou mais subconjuntos é 
relativamente bem conhecido ... um problema próximoa este e talvez mais 
desafiador consiste em determinar de quantas maneiras distintas podemos dividir 
um conjunto com elementos
repetidos entre duas ou mais pessoas. Por exemplo. Seja :
10 bolas pretas  ( iguais entre si e indistinguíveis )8 bolas brancas  ( iguais 
entre si e indistinguíveis )15 bolas azuis ( iguais entre si e 
indistinguíveis )


De quantas maneiras distintas podemos dividir as bolas acima entre 2 pessoas ? 
E entre 4 pessoas ?
Um AbraçãoPSR,1220511132D
Date: Sun, 22 May 2011 18:26:04 -0300

Subject: [obm-l] Re: [obm-l] Re: [obm-l] Re: [obm-l] RE: [obm-l] Número de 
partições de um conjunto
From: pedromatematic...@gmail.com
To: obm-l@mat.puc-rio.br


Falou cara muitíssimo obriado.
Olá Paulo Santa Rita, há quanto tempo não conversávamos não é mesmo?
Olha meu erro foi fazer o "r" variar de 1 até n-r salvo o engano, depois somei 
todos os resultados, por isso deu aquele somatório. Mas sua solução como sempre 
foi brilhante.


Abração e muito obrigado.

Em 20 de maio de 2011 12:54, Alessandro Madruga Correia 
 escreveu:


Olá,
me intrometendo...


Caro Wily como fizestes para aparecer a imagem?
Paulo volto a falar contigo!


Ele utilizou esse site, http://www.codecogs.com/latex/htmlequations.php



-- 
  ,= ,-_-. =.   [] Alessandro Madruga Correia
 ((_/)o o(\_))  Viaconnect -- Suporte Técnico +55 (54) 4009 3444
  `-'(. .)`-'   "Certamente, tenho arriscado minha saúde algumas vezes pelo


  \_/   excesso de trabalho, mas e daí? Somente os repolhos não têm
nervos, nem preocupações. E o que conseguem com seu bem-estar  
perfeito?" (Carl Gustav Jacob Jacobi)




-- 


Pedro Jerônimo S. de O.
Júnior


Professor
de Matemática


Geo João Pessoa
– PB 



  

  

[obm-l] RE: [obm-l] Re: [obm-l] RE: [obm-l] Re: [obm-l] Re: [obm-l] Re: [obm-l] RE: [obm-l] Número de partições de um conjunto

2011-05-23 Por tôpico Paulo Santa Rita

Oi Salhab e demais colegas desta lista ... OBM-L,
A solução está errada. Para ver isso claramente, considere duas bolas pretas e 
tres brancas, que representamos por PPBBB. Suponhamos que precisamos dividir 
estas bolas entre duas pessoas. Usando a sua solução, teriamos as duas divisões 
abaixo :( OBS : o simbolo "|" representa a vareta )
PB | PBBBP ! PBB
CONTADAS COMO DISTINTAS, quando, em verdade, elas representam A MESMA DIVISÃO : 
{B,P} e {P,B,B}. Perceba que aordem com que uma pessoa recebe as bolas é 
indiferente ...
Um abraçãoPSR,2230511DC

Date: Mon, 23 May 2011 10:30:03 -0300
Subject: [obm-l] Re: [obm-l] RE: [obm-l] Re: [obm-l] Re: [obm-l] Re: [obm-l] 
RE: [obm-l] Número de partições de um conjunto
From: msbro...@gmail.com
To: obm-l@mat.puc-rio.br

Olá Paulo,uma solução é colocar todas as bolas em uma linha e adicionar K 
"varetas", onde K=número de pessoas - 1.Então, contar o número de permutações.
No seu caso, teríamos 10 bolas pretas, 8 bolas brancas, 15 bolas azuis e 1 
vareta (2 pessoas).
Assim, o número de permutações é:(10+8+15+1)! / (10! 8! 15! 1!) = 34! / (10! 8! 
15!)
Para 4 pessoas, vamos utilizar 3 varetas. E ficamos com:(10+8+15+3)! / (10! 8! 
15! 3!) = 36! / (10! 8! 15! 3!)

Abraços,Salhab

2011/5/22 Paulo Santa Rita 






Oi Pedro e demaiscolegas desta lista ... OBM-L,
Este problema de dividir um conjunto em grupos de 2 ou mais subconjuntos é 
relativamente bem conhecido ... um problema próximo
a este e talvez mais desafiador consiste em determinar de quantas maneiras 
distintas podemos dividir um conjunto com elementosrepetidos entre duas ou mais 
pessoas. Por exemplo. Seja :

10 bolas pretas  ( iguais entre si e indistinguíveis )8 bolas brancas  ( iguais 
entre si e indistinguíveis )15 bolas azuis ( iguais entre si e 
indistinguíveis )

De quantas maneiras distintas podemos dividir as bolas acima entre 2 pessoas ? 
E entre 4 pessoas ?

Um AbraçãoPSR,1220511132D
Date: Sun, 22 May 2011 18:26:04 -0300
Subject: [obm-l] Re: [obm-l] Re: [obm-l] Re: [obm-l] RE: [obm-l] Número de 
partições de um conjunto

From: pedromatematic...@gmail.com
To: obm-l@mat.puc-rio.br


Falou cara muitíssimo obriado.
Olá Paulo Santa Rita, há quanto tempo não conversávamos não é mesmo?
Olha meu erro foi fazer o "r" variar de 1 até n-r salvo o engano, depois somei 
todos os resultados, por isso deu aquele somatório. Mas sua solução como sempre 
foi brilhante.


Abração e muito obrigado.

Em 20 de maio de 2011 12:54, Alessandro Madruga Correia 
 escreveu:


Olá,
me intrometendo...


Caro Wily como fizestes para aparecer a imagem?
Paulo volto a falar contigo!


Ele utilizou esse site, http://www.codecogs.com/latex/htmlequations.php



-- 
  ,= ,-_-. =.   [] Alessandro Madruga Correia
 ((_/)o o(\_))  Viaconnect -- Suporte Técnico +55 (54) 4009 3444
  `-'(. .)`-'   "Certamente, tenho arriscado minha saúde algumas vezes pelo


  \_/   excesso de trabalho, mas e daí? Somente os repolhos não têm
nervos, nem preocupações. E o que conseguem com seu bem-estar  
perfeito?" (Carl Gustav Jacob Jacobi)




-- 


Pedro Jerônimo S. de O.
Júnior


Professor
de Matemática


Geo João Pessoa
– PB 



  

  

[obm-l] RE: [obm-l] Re: [obm-l] Re: [obm-l] Re: [obm-l] RE: [obm-l] Número de partições de um conjunto

2011-05-22 Por tôpico Paulo Santa Rita

Oi Pedro e demaiscolegas desta lista ... OBM-L,
Este problema de dividir um conjunto em grupos de 2 ou mais subconjuntos é 
relativamente bem conhecido ... um problema próximoa este e talvez mais 
desafiador consiste em determinar de quantas maneiras distintas podemos dividir 
um conjunto com elementosrepetidos entre duas ou mais pessoas. Por exemplo. 
Seja :
10 bolas pretas  ( iguais entre si e indistinguíveis )8 bolas brancas  ( iguais 
entre si e indistinguíveis )15 bolas azuis ( iguais entre si e 
indistinguíveis )

De quantas maneiras distintas podemos dividir as bolas acima entre 2 pessoas ? 
E entre 4 pessoas ?
Um AbraçãoPSR,1220511132D
Date: Sun, 22 May 2011 18:26:04 -0300
Subject: [obm-l] Re: [obm-l] Re: [obm-l] Re: [obm-l] RE: [obm-l] Número de 
partições de um conjunto
From: pedromatematic...@gmail.com
To: obm-l@mat.puc-rio.br

Falou cara muitíssimo obriado.
Olá Paulo Santa Rita, há quanto tempo não conversávamos não é mesmo?
Olha meu erro foi fazer o "r" variar de 1 até n-r salvo o engano, depois somei 
todos os resultados, por isso deu aquele somatório. Mas sua solução como sempre 
foi brilhante.

Abração e muito obrigado.

Em 20 de maio de 2011 12:54, Alessandro Madruga Correia 
 escreveu:

Olá,
me intrometendo...


Caro Wily como fizestes para aparecer a imagem?
Paulo volto a falar contigo!


Ele utilizou esse site, http://www.codecogs.com/latex/htmlequations.php


-- 
  ,= ,-_-. =.   [] Alessandro Madruga Correia
 ((_/)o o(\_))  Viaconnect -- Suporte Técnico +55 (54) 4009 3444
  `-'(. .)`-'   "Certamente, tenho arriscado minha saúde algumas vezes pelo

  \_/   excesso de trabalho, mas e daí? Somente os repolhos não têm
nervos, nem preocupações. E o que conseguem com seu bem-estar  
perfeito?" (Carl Gustav Jacob Jacobi)



-- 


Pedro Jerônimo S. de O.
Júnior

Professor
de Matemática

Geo João Pessoa
– PB 


  

[obm-l] RE: [obm-l] Um problema curioso e... insolúvel

2011-05-20 Por tôpico Paulo Santa Rita

Olá Bouskela e demaiscolegas desta lista ... OBM-L,
Tão simples ! Tão simples quanto afirmar que para todo natural N > 2 não existe 
inteiros A, B e C tais que A^N  +  B^N  =  C^N  ? A beleza e a profundida estão 
justamente na simplicidade ... O enunciado é pitoresco e talvez por isso mesmo 
dá origem a diversas interpretações. Vou enunciar aqui os pressupostos que 
entendo estarem contidos no enunciado da questão :
1) a expressão "correnteza desprezível" significa que "se o nadador resolver 
boiar, ele ficará parado." Sem essa hipótese o problema torna-se trivial, pois 
caso a correnteza fosse significativa, bastaria ele boiar e observar a direção 
em que corre o rio. A seguir, tomando qualquer outra direção não paralela, 
bastaria nadar sempre nesta direção que necessariamente chegaria à margem.
2) com "escuridão" ou "peixes vorazes devoram os olhos" o criador da questão 
quer dizer que o nadador pode saber que fez uma curva para a direita ou para a 
esquerda EM RELAÇÃO A DIREÇÃO ANTERIOR, mas não é capaz de calcular o valor 
exato do ãngulo que esta nova direção tem com a anterior. Esta hipótese também 
é necessária, sob pena, mais uma vez, do problema tornar-se trivial ...
Acho razoável supor o seguinte :
3) O nadador nada a velocidade constante em cada trecho retilineo, digamos, 1 
braçada/segundo. 4) O nadador, até por ser nadador, sabe o quanto avança em 
cada braçada, digamos, 1 braçada avança 50 cm
Agora considere a seguinte situação :
Ao ficar cego, ele dá um numero arbitrário de braçadas, digamos, K braçadas. 
Toma então uma nova direção. Nesta nova direção ele dá novamente K braçadas e 
para ( bóia ). Temos aqui um triangulo isósceles ( lados : K braçadas, k 
braçadas, distancia ao ponto original fornecido pelo sistema de navegação ).
Se podemos supor que o nadador é também um Matemático não-medíocre, ele esta 
neste momento boiando e analisando o triãngulo acima - do qual conhece as 
medidas dos três lados - e projetando o que deverá fazer para atingir uma 
margem ...

É preciso parar aqui e esperar a próxima manifestação do Bouskela neste 
sentido, pois introduzi muitas hipóteses e bem pode suceder que algumas delas 
não estejam no rol daquelas admitidas pelo criador da questão. De qualquer 
forma, afirmo que gostei da questão e gostaria de pensar um pouco mais sobre 
ela.
Um Abraço a todosPSR, 620051109AA From: bousk...@msn.com
To: obm-l@mat.puc-rio.br
Subject: [obm-l] Um problema curioso e... insolúvel
Date: Thu, 19 May 2011 15:54:18 -0300



Olá a todos, Uma curiosidade: – Parece-me que o problema abaixo (tão simples!) 
permanece em aberto.  Um nadador está nadando (o que mais pode fazer um 
nadador?) em um ponto qualquer de um rio horizontal, retilíneo, com correnteza 
desprezível, comprimento infinito e largura finita. Subitamente, peixes 
extremamente vorazes devoram os olhos do malfadado nadador, ou, com menos 
drama, cai a noite absolutamente escura. Qual é a trajetória que o nadador deve 
trilhar, i.e., nadar, para atingir – seguramente – uma das margens, nadando a 
menor distância possível? Obs.: – O malfadado nadador tem, implantado em sua 
cabeça, um sistema de navegação que lhe informa, continuamente, a sua posição 
em relação ao ponto inicial (o ponto no qual os peixes devoraram os seus 
olhos).  Saudações,Albert bouskelabousk...@msn.com  


[obm-l] RE: [obm-l] Re: [obm-l] RE: [obm-l] Re: [obm-l] Vale a "demonstração"?

2011-05-19 Por tôpico Paulo Santa Rita

Oi Ralph e demais colegasdesta lista ... OBM-L,
Complicado ! Se eu estivesse ensinando indução matemática, não aceitaria como 
demonstração poisneste ponto é natural requerer o reconhecimento explícito dos 
passos e elementos da demonstração. Por outro lado, se fosse uma questão de 
outro assunto, no qual tal formula fosse necessária, eu NEM EXIGIRIA uma 
demonstração : os passos abaixo são mais que suficientes para sugerir que a 
pessoa conhece a técnica.
Penso também que não só o nível em que tal fórmula é exigida é necessário 
considerar para respondera questão, mas também a própria mentalidade do 
professor que corrige, se é que podemos chamar assim.Existe o "Prof Picuinha", 
aquele que cumprimenta a recem eleita miss Universo reparando que o dedão dopé 
da miss não é bonito. Este cara essencialisa o trivial e trivializa o 
essencial. È o dracula dacriatividade. Esse cara vai exigir a demonstração 
"detalhe" por "detalhe", esquecendo que nenhummatematico do mundo em todos os 
tempos fez qualquer demonstração real de um resultado novoseguindo tal mediocre 
rigor.
Mas devemos convir que tambem existe o "Prof Globo Ciencia", que só se preocupa 
com o ibope juntoa galera, esquecendo totalmente do conjunto de saudaveis 
valores que necessariamenteseguem junto a qualquer educação séria. Acredito que 
a sabedoria, como diria aristoteles, estano "caminho do meio", uma "media 
aritmetica" entre os dois estilos acima: o Prof Socratico !






> Date: Thu, 19 May 2011 11:44:25 -0300
> Subject: [obm-l] Re: [obm-l] RE: [obm-l] Re: [obm-l] Vale a "demonstração"?
> From: ralp...@gmail.com
> To: obm-l@mat.puc-rio.br
> 
> Olha, se alguem escrevesse este argumento numa prova, eu o aceitaria
> uma demonstracao.
> 
> Ou seja, concordo com o Paulo -- a inducao formal seria tao imediata,
> que para mim nao vale a pena escreve-la explicitamente. Ela nao
> acrescentaria nada NESTE CASO.
> 
> "Pebolim."
> 
> Abraco,
>   Ralph
> 
> 2011/5/19 Paulo Argolo :
> >
> > Colegas,
> >
> > Minha preocupação aqui não é obter uma demonstração, mas somente indagar da 
> > validade do procedimento apresentado. Parece-me que tal procedimento é uma 
> > demonstração por indução, que abre mão da habitual formalidade, isto é, não 
> > explicita a base de indução e o passo indutivo.
> > Os Colegas concordam?
> >
> > Abraços do Paulo!
> >
> > 
> >> Date: Wed, 18 May 2011 20:59:57 -0300
> >> Subject: [obm-l] Re: [obm-l] Vale a "demonstração"?
> >> From: hit0...@gmail.com
> >> To: obm-l@mat.puc-rio.br
> >>
> >> Sim. O "e assim sucessivamente" se chama princípio de indução.
> >> Formalmente falando, você deve mostrar que sua afirmação vale para n=1
> >> (este caso é chamado de base de indução), ou seja,
> >> a_1=q^(1-1)a_1=a^0a_1. E depois deve supor que a afirmação vale para um
> >> certo natural n e mostrar que vale para n+1 (este passo é chamado de
> >> passo indutivo).
> >>
> >> No nosso caso, se supormos que a_n=a_1.[q^(n-1)], então a_n+1 =
> >> q.a_n=q.a_1.[q^(n-1)]=a_1 q^n.
> >>
> >> 2011/5/18 Paulo Argolo
> >> >
> >>
> >>
> >> Caros Colegas,
> >>
> >> Pode-se dizer que o procedimento empregado abaixo para determinar o
> >> termo geral de uma progressão geométrica de razão q é uma real
> >> demonstração?
> >>
> >> DEMONSTRAÇÃO:
> >>
> >> Obs.: a_k , sendo k um número natural diferente de zero, indica o
> >> k-ésimo termo da progressão.
> >>
> >> Portanto, por definição de progressão geométrica:
> >>
> >> a_2 = (a_1).q
> >>
> >> a_3 = (a_2).q = (a_1).(q^2)
> >>
> >> E assim sucessivamente. Então:
> >>
> >> a_n = (a_n-1). q = (a_1).[q^(n-1)]
> >>
> >> Abraços do Paulo!
> >> =
> >> Instruções para entrar na lista, sair da lista e usar a lista em
> >> http://www.mat.puc-rio.br/~obmlistas/obm-l.html
> >> =
> >>
> >>
> >>
> >> --
> >> Tiago J. Fonseca
> >> http://legauss.blogspot.com
> > =
> > Instruções para entrar na lista, sair da lista e usar a lista em
> > http://www.mat.puc-rio.br/~obmlistas/obm-l.html
> > =
> >
> 
> =
> Instruções para entrar na lista, sair da lista e usar a lista em
> http://www.mat.puc-rio.br/~obmlistas/obm-l.html
> =
  

[obm-l] RE: [obm-l] Número de partições de um conjunto

2011-05-19 Por tôpico Paulo Santa Rita

Olá Pedro e demaiscolegas desta lista ... OBM-L,
Se eu entendesse a sua notação, opinaria. Acredito que seja Latex, mas eu não 
tenho aqui o plugin que permite a visualização. 
PROBLEMA 1
Vou supor que "r" e "s" são inteiros não-negativos e que  r + s  =<  n. Seja A 
o conjunto original com n elementos  
1) r+s = n  e  r # s
Neste caso é óbvio que a resposta será  Binom(n,r) = Binom(n,n-r) = Binom(n,s) 
pois ao escolher um conjunto, digamos, de r elementos, o que "sobra" no 
conjunto A terá n-r=s elementos e será o outro conjunto da partição.
Se r = s, divida o resultado anterior por 2
2) r+s < n  e  r # s
Neste caso, seja t = n - (r+s). Podemos formar um conjunto de t  elementos de 
Binom(n,t) maneiras. Fixada uma destas maneira, recaímos no caso anterior : 
poderemosformar Binom(n-(r+s),r) partições. Pelo principio multiplicativo segue 
que a resposta sera : Binom(n,t)*Binom(n-t ,r).
Se r=s, divida o resultado anterior por 2
Note que a resposta 2) engloba a 1), pois se r+s=n então t=n-(r+s)=0 e 
Binom(n,t)=Binom(n,0)=1 



PROBLEMA 2
Vamos escolher um dentre os n elementos e chama-lo de INTERSECÇÃO. Retirando a 
INTERSECÇÃO, sobram n-1  elementos. Sejam r' = r-1  e  s' = s-1. É facil ver 
que n-1, r' e s' recai no problema anterior, ja resolvido. Então a resposta 
aqui é : N*( resposta anterior com a devida adaptação )

Um problema de combinatória que eu acho interessante pode ser enunciado assim :
Seja A um matriz quadrada de ordem N tal que A(i,j) = j + (i -1)*N, onde j e i 
variam em K={1,2,...,N } e j representa a coluna e i representa a linha.Quantas 
matrizes quadradas B de ordem N podem ser formadas tais que :
1) B não tem elementos repetidos2) Todo elemento de B pertence a {1,2,3,..., 
N^2}3) B(i,j) é diferente de A(i,j) para todo par (i,j) pertencente a K^2   ( 
K^2 é o produto cartesiano de K por si mesmo ) 
Um abraço a todosPSR,51005111338
Date: Thu, 19 May 2011 18:29:53 +0430
Subject: [obm-l] Número de partições de um conjunto
From: pedromatematic...@gmail.com
To: obm-l@mat.puc-rio.br

No primeiro problema cheguei a algo do tipo 1/2\cdot [ C_{n}^{1} \cdot 
(2^{n-1}-1) + C_{n}^{2} \cdot (2^{n-2}-1) + C_{n}^{3} \cdot (2^{n-3}-1) 
+...+C_{n}^{n-1} ]
queria saber se alguém sabe opinaar se estou no caminho correto.

Abraços.

1. Seja X um conjunto com "n" elementos. Calcule o número de escolhas possíveis 
de dois subconjuntos disjuntos de "r" e "s" elementos, respectivamente. [E se r 
= s?]



2. O mesmo exercício anterior mas em que os dois subconjuntos possam 
intersectar-se
num único elemento.
-- 


Pedro Jerônimo S. de O.
Júnior

Professor
de Matemática

Geo João Pessoa
– PB 


  

[obm-l] RE: [obm-l] Vale a "demonstração"?

2011-05-19 Por tôpico Paulo Santa Rita

Oi Paulo e demais colegasdesta lista ... OBM-L,
Da forma como você apresentou, não, pois a a passagem de "a_n=(a_n-1).q" para 
"(a_1).[q^(n-1)]" não está suficientemente clara ... em verdade, nestapassagem 
você "já esta utilizando "justamente "aquilo que voce dever provar". Em casos 
simples tal como o que você apresenta, quando previamente já seconhece a 
fórmula final, é bastante comum o uso da indução matemática. Assim :
a_2=a_1.q  ( por definição )a_3=a_2.q ( por definição ) => a_3 =a_1.(q^2) ( 
usando o resultado da linha anterior )
Olhando estes dois casos e vendo a "ainda aparente" relação entre índice e o 
expoente, conjecturamos que :
a_n=a_1.( q^(n-1) )
Supondo isso, teremos que : a_(n+1)=a_n.q  por definição. Substituindo a 
hipótese de indução, chegamos a a_(n+1)=a_1.(a^n). Pelo principio da 
induçãofinita fica estabelecido que a expressão vale para todo n natural.
Mas as provas por indução padecem de um mal fundamental, a saber, pressupõe o 
conhecimento prévio da expressão que devemos provar. Em geral, no cursode uma 
investigação, você não conhece previamente o que deverá demonstrar. Por 
exemplo, olhe este link aqui :
http://math.stackexchange.com/questions/17320/derivation-of-the-partial-derangement-rencontres-numbers-formula
lendo, verifica-se que alguns estudantes e pesquisadores estão procurando uma 
"fórmula" para o total de "arranjos caóticos" de comprimento P que podemosfazer 
de um total de N elementos. Como este problema está  ainda hoje ( até agora, 
pois vou mostrar a solução abaixo ) em aberto e a solução pode ser vista como 
uma generalização do trabalho do Nicolau Bernoulli e Euler, eu achei que valia 
a pena pensar nele e deduzi que :
Dn,k = Binom(N,K)*{ somatorio[ i  variando de 1 até M , binom(N-i , 
N-M)*binom(N-M , i)* (!(K-i))] }
onde, nesta formula :
1) M=min{K,N-K}   2) !N = N!*( (1/2!)-(1/3!) + ... + ( (-1)^N )*(1/N!) ) se N 
>= 23) !1=0  e !0=1
A demonstração não é trivial. Não apresento aqui porque isso é apenas o 
resultado inicial de uma pesquisa mais ampla que ainda não conclui.  Mas o que 
quero ressaltar é que EU NÃO SABIA em qual expressão chegaria. Sabia apenas que 
chegaria em algum lugar. Neste sentido, o principio da indução tem pouca 
utilidade.
Mas voce pode demonstrar o seu resultado assim :
a_2 = a_1.qa_3 = a_2.q...a_n=a_(n-1).q
multiplicando membro a membro as N-1 igualdades e eliminado os fatores comuns 
que aparecem nos dois membros, chegamos a :
a_n=a_1.(q^(n-1) )
E agora não foi preciso usar indução. 
Em síntese, não há um roteiro padronizado para demonstrações. O que há são 
certos principios que devemos respeitar ( por exemplo, você não pode usar como 
certo algo que ainda não foi provado ). De resto, o que é importante é a sua 
sensibilidade e intuição, é ela que nos conduz a coisas significativas e que 
nos mostra como provar de forma irretorquivel aquilo que apenas vemos do outro 
lado, no mundo próprio da Matemática
Um abração a TodosPSR, 5190511102A

> From: argolopa...@hotmail.com
> To: obm-l@mat.puc-rio.br
> Subject: [obm-l] Vale a "demonstração"?
> Date: Wed, 18 May 2011 22:51:28 +
> 
> 
> 
> Caros Colegas,
> 
> Pode-se dizer que o procedimento empregado abaixo para determinar o termo 
> geral de uma progressão geométrica de razão q é uma real demonstração?
> 
> DEMONSTRAÇÃO:
> 
> Obs.: a_k , sendo k um número natural diferente de zero, indica o k-ésimo 
> termo da progressão.
> 
> Portanto, por definição de progressão geométrica:
> 
> a_2 = (a_1).q
> 
> a_3 = (a_2).q = (a_1).(q^2)
> 
> E assim sucessivamente. Então:
> 
> a_n = (a_n-1). q = (a_1).[q^(n-1)]
> 
> Abraços do Paulo!   
> =
> Instruções para entrar na lista, sair da lista e usar a lista em
> http://www.mat.puc-rio.br/~obmlistas/obm-l.html
> =
  

[obm-l] RE: [obm-l] Re: [obm -l] QUESTÃO AMT FINA NCEIRA

2010-12-14 Por tôpico Paulo Santa Rita

Olá a todos !
Total apoio ao que o Palmerim disse.
Um abraço a todos PSR,31412100A35

Date: Mon, 13 Dec 2010 19:23:02 -0200
Subject: [obm-l] Re: [obm-l] QUESTÃO AMT FINANCEIRA
From: palmerimsoa...@gmail.com
To: obm-l@mat.puc-rio.br


Oi Rogério,
 
está parecendo que você tem uma lista de exercícios e que os está postando para 
irmos resolvendo para você. Provavelmente você está se preparando para um 
concurso público, talvez esteja fazendo um cursinho e o professor deu uma lista 
ou  talvez você tenha adquirido um livro ou apostila de banca de jornal, mas,  
seja qual for o caso, considere o seguinte:

1º) Este grupo não se destina a resolver listas de exercícios para as pessoas;
2º) os exercícios são muito fáceis (triviais demais) e, com um pouco de esforço 
e estudo, você deveria saber resolvê-los; tudo bem, você pode até ter dúvida em 
um ponto ou outro, mas aconselho que procure estudar, aprender e tentar 
resolvê-los primeiro, e, caso não consiga, peça ajuda, mas indique o que fez e 
onde está a dúvida. Não ponha uma lista de exercícios triviais para que a gente 
resolva; vá com calma! Se você postar um exercício de cada vez, mostrando qual 
é a sua dúvida, tenho certeza que receberá ajuda aqui.

 
um abraço e boa sorte!
 
Palmerim
 
Em 13 de dezembro de 2010 18:46, Robério Alves  
escreveu:






1.  Um empréstimo de R$ 120.000,00 é feito pelo sistema de amortização 
constante, à taxa de 2% ao mês, devendo ser devolvido em 8 prestações mensais. 
Sabendo que houve um prazo de carência de 3 meses, elabore o plano de pagamento:

a. Com pagamento dos juros;
b. Com capitalização dos juros.

 

-- 
Palmerim
  

RE: [obm-l] Dificuldade numa integral

2010-12-13 Por tôpico Paulo Santa Rita





Olá Luiz Antonio e demaiscolegas desta lista ... OBM-L,
No *Máxima" isso é imediato : integrate( ( x^2 - 3*x + 7 ) / (( x^2 - 4*x + 6 
)^2 ) , x );O resultado foi (  7*arctg(  (x - 2) / ( rq(2) ) ) / 4*rq(2)   )   
+   ((3*x - 8 ) / ( 4*(x^2) - 16*x + 24 )   )
onde rq(n) é a RAIZ QUADRADA de n e arctg(y) é o arco cuja tangente é y.
Um abraço a todos !PSR,21312100E03

Date: Sun, 12 Dec 2010 10:51:09 -0200
Subject: [obm-l] Dificuldade numa integral
From: rodrigue...@gmail.com
To: obm-l@mat.puc-rio.br

Olá, pessoal!!!Tudo bem???Estou com dificuldade para resolver esta integral:
[(x^2 - 3x + 7)/((x^2 - 4x + 6)^2)]dx
Será que alguém pode me ajudar?
Tentei resolver pelas frações parciais mas empaquei.Um abraço para todos e 
muito obrigado.Luiz Antonio
  

[obm-l] RE: [obm-l] Números Transcendentes + Com binatória

2010-10-22 Por tôpico Paulo Santa Rita

Olá Luiz e demais colegasdesta lista ... OBM-L,
1) Quem provou que os números transcendentes são infinitos ?
Cantor demonstrou diretamente que os *NÚMEROS ALGÉBRICOS* são enumeraveis. Como 
ele também havia demonstrado que os números reais não são enumeráveis, os reais 
não-algébricos, vale dizer, os NÚMEROS TRANSCENDENTES, não podem ser 
enumeráveis ( se fossem enumeráveis, os números reais, sendo a união disjunta 
de algébricos e transcendentes, seria enumerável ... ). Portanto, pode-se dizer 
que Cantor *DEMONSTROU INDIRETAMENTE* que existem infinitos números 
transcendentes.
Note que o conceito de número transcendente é caracterizado indiretamente, pois 
dizemos que um número é transcendente quandoele não é algébrico, isto é, nos 
tomamos o conceito bem estabelecido ( um número é algébrico quando ele é 
solução de umaequação algébrica com coeficientes inteiros ) de número algébrico 
para falar sôbre os transcendentes. Este procedimento, em Matemática, é 
tipicamente uma suave confissão de ignorância e desconhecimento ... Em verdade, 
criamos uma *sacola* e passamosa proceder assim : o que não é algébrico nós 
jogamos aqui. A verdade é que sabemos muito pouco sôbre estes números. Essa 
ignorância,inclusive, pode estar ligada a hipótese do contínuo, pois, quem sabe 
se neste ninho de gatos que são os numeros transcendentes não se escondeaquele 
famoso e tão procurado conjunto não-enumerável com cardinalidade inferior a dos 
reais ?
Os números transcendentes é uma terra de ninguém.
2) Como descobrir se um número real r é transcendente ? Demosntrando que r não 
é algébrico. Existem uns pouquíssimos e pobríssimos resultadosque servem para 
caracterizar algumas familias de transcendentes. Por exemplo :
TEOREMA DE GELFOND : Se A é um número algébrico não-nulo e diferente de 1 e B é 
um irracional, então A^B é transcendente.Do teorema acima concluimos, por 
exemplo, que "raiz_2(2)^raiz_2(2)" é transcendente ( raiz_2(2) = raiz quadrada 
de dois ). São também transcendentes:N^raiz2(2), onde N é um natural maior que 
1.
OBS : O resultado acima responde a uma das famosas perguntas elaboradas pelo 
Hilbert
TEOREMA DE LINDEMAN : e^A é transcendente para todo A algébrico não nulo  ( e= 
2,7 ... = número de Euler = base dos logaritmos naturias )
NUMEROS DE LIOUVILLE : Todo número A tal que para todo natural N existem p e q 
inteiros tais que modulo(A - (p/q) ) <  1/(q^N)  Um exemplo classico de numero 
de Liouville e :
A= (1/10) + (1/(10^2)) + (1/(10^6)) + ... + (1/(10^(N!))) + ...
Deve existir mais resultados parciais que não me ocorrem agora.
Nem todo todo número transcendente é número de Liouville, ou , melhor ainda, 
nenhuma das familias de numeros caracterizáveis pelos resultados acimaexaure 
todos os numeros trancendentes. É também importante destacar que o conceito de 
NUMERO TRANSCENDENTE esta atrelado ao conceito de númeroalgébrico, que, por sua 
vez, esta associado ao conceito de polinomio com coeficientes inteiros. Ora, 
existem diversos outros exemplos de corpos alem dosracionais e reais( e entre 
eles, por exemplo, A+B*raiz2(2), onde A e B são racionais, formam um corpo 
"entre" Q e R ). Portanto, é possivel extender o conceito de número 
transcendente para outros corpos, podendo-se falar em NUMERO TRANSCENDENTE 
SOBRE O CORPO TAL.
Em minha opinião, este imbricamento entre os conceitos de transcendente e 
algébrico, em que pese nos ter permitido ver pela primeira vez os 
transcendentes,é um obstaculo a ser vencido para uma melhor compreensão da 
eventual *estrutura* e *beleza* que há neste universo ( dos transcendentes ) 
dominio ... Talvezo estudo do que há nos transcendentes relativos a outros 
corpos ( incluindo uma olhada especial nos finitos )  poderia lançar alguma luz 
aqui. O que é certoé que a conceituação atual é pobre para abordar tais números 
e há muito o que descobrir aqui.
Note que ha muito outros conceitos ( por exemplo, número computável , conjunto 
magro, medida de um conjunto ) que podem ser aplicados a estas classesde 
números ( as classes caracterizadas pelos resultados acima ). Eu me lembro, por 
exemplo, que alguem ja associou a ideia de conjunto magroao conjunto dos 
números de Liouville ( acho que é que numeros de Liouville é complementar de um 
conjunto magro ou algo proximo disso ) 
Um Abraço a TodosPSR,62210100A15

 


> Date: Thu, 21 Oct 2010 10:16:53 -0200
> Subject: [obm-l] Números Transcendentes + Combinatória
> From: rodrigue...@gmail.com
> To: obm-l@mat.puc-rio.br
> 
> Olá, pessoal!!!
> Tudo bem???
> Estou querendo saber quem provou que os números transcendentes são
> infinitos. Além disso, como descobrir, dentro dos reais, um número
> transcendente? É possível gerá-los?
> Outra coisa, estou com dificuldades num problema muito simples de
> combinatória: "Quantos anagramas da palavra ESCOLA apresentam as
> vogais ou as consoantes juntas?" Fiz pelo complementar mas acho que
> está errado...
> Alguém pode me ajudar???
> Um abração para todos.
> Luiz
> 
> ===

[obm-l] RE: [obm-l] Sequênci as Binárias e Concat enação

2010-10-10 Por tôpico Paulo Santa Rita

Olá Bruno e demais colegasdesta lista ... OBM-L,
Não entendi o enunciado da questão que você postou. Mas, por coincidência, eu 
estava justamente tratando de um problema no N-cubo unitário  -  N-cubo 
unitário e o conjunto de todas as N-uplas (x1, ..., xn) nas quais xi=0 ou xi=1 
para todo i=1, ..., N  -  cujas operações podem ser parafraseadas em termos de 
operações com números binários da seguinte forma : 
Seja B um número natural ímpar maior que 1 tal que sua representação na base 2 
contem N dígitos. Tomarei por B* o número natural tal que B+B*=(2^N) - 1 . Note 
que B* é necessariamente um natural par. Se IMAGINARMOS que cada dígito de B 
está grafado num pequeno quadrado e que a direita do seu dígito menos 
significativo (dígito mais a direita ) e a esquerda do seu dígito mais 
significativo ( dígito mais a esquerda ) existem infinitos quadradinhos onde 
grafamos zero, o binário B passa a pertencer a uma "fita de leitura de 
comprimento infinito".
Representarei o dígito menos significativo de B por X1, o mais significativo 
por Xn.  Se p > n então Xp = 0

Considere o seguinte algoritmo :
1) i=02) i =i+13) Se Xi   XOR   Xi+1 = 1 VOLTE ao passo 2 4) J = 05) j = j + 
16) Yj = 1 SE j =1 SENAO Yj = (Xi  XOR  Xi+1) AND Yj-1  +   ( NOT (Xi XOR Xi+1) 
) AND Xi7)  Zj = (Xi   XOR   Xi+1)  XOR  Yj8 ) Faca i=i+19 ) Se i < N+2   VOLTE 
  ao passo 5
Neste algoritmo, XOR é a clássica e bem conhecida porta lógica, assim definida 
: 0 XOR 0 = 1 XOR 1 = 0   e  1 XOR 0=0 XOR 1=1. Esta operação é associativa e 
representa o "OU EXCLUSIVO". Partindo de um dos vertices do N-cubo cuja 
coordenada mais a direita é um, vale dizer, em nossa notação, de um número 
ímpar em binário, os Z's obtidos com o algoritmo acima constituem o NOVO 
VÉRTICE  que me interessa analisar. O novo vértice pertence a um M-cubo 
unitário e eu consigo demonstrar que reiterando o algoritmo nós teremos que 
necessariamente M < N  em algum momento.
A questão não é trivial e o que eu preciso é obter o NOVO VÉRTICE em função de 
B*
Um Abraço a todosPSR,11010100A35
From: collares.br...@hotmail.com
To: obm-l@mat.puc-rio.br
Subject: [obm-l] Sequências Binárias e Concatenação
Date: Tue, 5 Oct 2010 20:19:32 +








Seja A={01,100,101}, e B={0,1,11}. Decida se as sequências binárias abaixo são 
geradas univocamente:

a) A*
b) B*
c) {00}*A*

Obs: A*=EUAUA²UA³U...

Grato


BRUNO MARQUES COLLARS
  

RE: [obm-l] Axioma ou teorema?

2010-09-30 Por tôpico Paulo Santa Rita

Olá Guilherme e demaiscolegas desta lista ... OBM-L,
Em tese, qualquer afirmação que seja um axioma em sistema formal pode vir a a 
ser um teorema em outro e vice-versa. Portanto, não tem sentido perguntarse uma 
afirmação qualquer, em si e desvinculada de um contexto, é um axioma ou teorema 
... A aifrmação a que você se refere é um axioma ou postulado nocontexto da 
Geometria Euclidiana. Alias, este axioma foi formulado pela primeira vez pelo 
Arquimedes e foi o primeiro exemplo de axioma métrico que se tem notícia. Com 
ele, entre outras aplicações, Arquimedes prova que uma poligonal envolvente é 
"maior" ( mede mais ) que qualquer poligonal envolvida. Ele tambemusa isso na 
quadratura da parabola e na aplicação do método da exaustão, do Eudoxo.
É claro que em outro contexto este "axioma" pode virar um "teorema". Por 
exemplo, em Analise Funcional.
Seria interessante esclarecer se para um dado conjunto de objetos existem 
afirmações que são irredutiveis, nos sentido de que seriam indemonstraveis 
emqualquer formalização factivel com tais objetos. Seriam como "atomos 
logicos". Mas eu sou mais de acreditar de que um tal possivel absoluto e 
incompativelcom o nosso tempo e a nossa epoca... Alguem saberia dizer algo 
inteligente neste sentido ?
Um AbraçãoPSR,22709100907

From: rjguilhermevie...@hotmail.com
To: obm-l@mat.puc-rio.br
Subject: [obm-l] Axioma ou teorema?
Date: Sat, 25 Sep 2010 23:09:12 +0300








Caros colegas,

A afirmação "O menor caminho entre dois pontos A e B é o segmento de reta AB" é 
um axioma? Ou é um teorema?
Bem... creio que seja um axioma, pois me parece que não há como demonstrar o 
teorema, sem incorrer em petição de princípio.


Abraços!
Guilherme
  

[obm-l] RE: [obm-l] combinat ória

2010-08-26 Por tôpico Paulo Santa Rita

Ola a todos !
Eis aqui uma sugestão :
1) IMAGINE oito posições vazias, digamos : - - - - - - - - . Nestas posições 
serão colocadas as letras. É fácil ver que se escolhermos as 4 posições que 
serãoocupadas pelas vogais, as posições restantes só poderão ser preenchidas de 
uma única maneira pelas consoantes, pois estas deverão estar em 
ordemalfabética. Assim, podemos ignorar as consoantes e nos preocuparmos 
exclusivamente com a quantidade de maneiras distintas de dispor 4 letras em 
oito posições, 3 das quais iguais entre si.
2) Podemos escolher 4 posições dentre 8 possíveis de Binom(8,4). Fixada uma 
destas escolhas, qualquer maneira de dispor as 4 vogais pode ser IMAGINADAcomo 
uma permutação de 4 elementos, dos quais 3 sao iguais entre si. Segue daqui que 
dado as posições escolhidas, existem 4!/(1!*3!)=4 maneiras distintasde dispor 
as vogais. Pelo princípio multiplicativo da análise combinatória concluímos que 
existem  4*Binom(8,4)=4*70=280 anagramas que atendem as condições de simetria 
do seu problema.
3) O raciocínio acima podes ser generalizado. Por exemplo, considere a palavra 
"MICROSOFT". Quantos anagramas podem ser formados com esta palavra demaneira 
que as consoantes estejam em ordem alfabética ? Pelo que já vimos acima, 
esquecemos esta consoantes e buscamos as maneiras de escolher 3posições nas 9 
possíveis. Isto dá : Binom(9,3). Fixada uma destas escolhas, achamos o total de 
permutações de 3 elementos com 2 repetidos. Isto fornece:3!/2!=3. Logo. o  
total de anagramas será 3*binom(9,3)=3*84=252
4) Agora, podemos aplicar o mesmo raciocínio a palavra "PROCURE" ? e NO caso da 
palavra "GOOGLE" ? Sim. Na primeira, existem dois R's e na segunda dois G's mas 
amaneira de dispor as consoantes, tendo sido fixado previamente as posições nas 
quais colocaremos as vogais, continua sendo única. Logo, o raciocínio continua 
válido neste caso.
5) Considere agora as palavra composta "NEELIE KROES", onde estamos 
considerando que o "espaço em branco" entre o E e o K é um caracter. É valido 
aplicaro raciocínio acima se considerarmos que o caracter "espaço em branco" 
uma vogal ? E se o considerarmos uma consoante ?
Isso e apenas uma sugestão, mas, esteja atento e não se esqueça de pesquisar.
Um Abraço a Todos !PSR,52608100B26
  
Date: Wed, 25 Aug 2010 13:36:22 -0700
From: cacar...@yahoo.com
Subject: [obm-l] combinatória
To: obm-l@mat.puc-rio.br

"Quantos anagramas da palavra BATALHÃO (desconsidere o til como diferença) tem 
as consoantes em ordem alfabética?"

Como pensar?

  

[obm-l] Bons Livros de Matem ática LEGALMENTE grá tis

2010-06-17 Por tôpico Paulo Santa Rita

Olá a todos, membrosdesta lista ... OBM-L,
Essa mensagem não trata de uma particular questão matematica, mas eunão ficaria 
bem comigo mesmo se não divulgasse aqui que no endereçoa seguir :
http://rinconmatematico.com/libros.htm
Existem muitos bons livros, a maioria em formato PDF, que podem 
legalmentesofrer download. Muitos são de famosas universidades ( Ex Princeton 
), outrosde Sites particulares, etc.
Ressalto que não verifiquei todos os link's. Se houver algum com 
irregularidadese/ou desrespeitando direito alheio, por favor, façam a denuncia 
competente.
Um abraço a todos !PSR,51706100B31






  
_
NINGUÉM PRECISA SABER O QUE VOCÊ ESTÁ COMPRANDO. LEIA MAIS SOBRE ESSE ASSUNTO 
AQUI.
http://www.microsoft.com/brasil/windows/internet-explorer/features/browse-privately.aspx?tabid=1&catid=1&WT.mc_id=1590

[obm-l] Re: [obm-l] Re: [obm-l] RES: [obm-l] Números Reais - MetaMAt

2010-02-19 Por tôpico Paulo Santa Rita
Ola Lucas e demais colegas
desta lista ... OBM-L,

Numeros nao sao criacoes humanas ...

A Fisica atual - em particular a Mecanica Quantica - nao admite uma
interpretacao pacifica, aceita por todos tal como ocorria com a Mecanica
Classica, havendo mesmo o bem colocado problema da interpretacao da
realidade. Assim, qualquer pessoa que se referir a realidade deveria, a
rigor, especificar  a qual interpretacao esta implicitamente se referindo
...

Portanto, hoje, NINGUEM esta autorizado a falar da realidade simplesmente
porque ainda nao sabemos qual a imagem que dela podemos fazer.

Por outro lado, existe uma bem conhecida tradicao oriental que diz que a
realidade e apenas uma ilusao ( Maia ) e que precisamos nos depreender das
ilusoes dos sentidos e com a mente contemplar a "verdadeira" realidade. A
Fisica atual nao corrobora este ponto de vista, mas claramente ja pode
admitir um estreito paralelo com esta percepcao, salvo as devidas
adaptacoes. Alguns caras bons, tais como O kapra, cultivaram e cultivam este
paralelo com significativa verossimilhanca ...

A Matematica vem seguindo este processo de abstracao e conceituação ha muito
mais tempo e parece-me que foi sobretudo com Gauss que nos percebemos com
grande vivacidade que nao deveriamos buscar nas coisas existentes, fisicas,
uma justificacao para as realidades conceituais que percebemos e que se nos
impoe. Ele mesmo viveu este dilema quando introduziu o plano de Argand e
passou a considerar os "numeros  complexos" sem considerar, onde, na
realidade, estes numeros se encontravam.

Parafraseando o Hegel, eu diria que nos nao devemos buscar na natureza uma
justificativa para as nossas percepcoes ou contrucoes, pois toda obra humana
e em-si maior e mais importante que toda a natureza, mas ter a certeza que
as percepcoes internas que temos do mundo proprio da Matematica e uma
condicao "sine qua non" parar entendermos o Mundo. Os elementos para a
compreensao do Universo esta dentro de nos e nao fora, a maneira de entender
melhor o mundo e precisamente a mais alta das abstraçoes Matematicas. Tem
sido assim nos ultimos seculos e acredito mesmo que esta tendencia vai se
radicalizar . E o que alguns caras bons chamam o principio antropico.

Sim, os numeros reais existem e constituem uma parcela da realidade que
podemos acessar e compreender. Eles nao precisam se adaptar a realidade,
simplesmente porque esta realidade nao existe como um dado; ao contrario, a
realidade deve se conformar a eles.

Nunca um Matematico qualquer criou alguma coisa. Nos simplesmente
descobrimos o que ja existe porque esse algo que existe e ´precisamente uma
parte de nos mesmos ... é uma pedacinho da resposta a pergunta : o que somos
? Talvez a resposta - corroborada por inumeras culturas misticas e
iniciaticas orientais - seja algo como : nos fazemos parte do universo e ele
e incompreensivel em sua totalidade se nao aprendermos como nos incluir
nesta propria compreensao ...


Em 19 de fevereiro de 2010 12:09, Lucas Reis escreveu:

> Bom, nenhum número é de verdade, ou existe na natureza. Eu pelo menos nunca
> vi um "2" por aí, um "1/3", nem um "pi". Números são criações abstratas
> (inclusive os naturais!), e matemáticos estudam implicações lógicas deles,
> sem precisar se preocupar se existem mesmo ou não.
>
> Só como curiosidade: algumas tribos amazônicas (se não me engano) não
> trabalham com números naturais. Pra eles o "dois" de "dois animais" é
> diferente do "duas" de "duas pessoas". São conceitos separados. Talvez isso
> já servisse de contraprova para quem diz que os números naturais não foram
> criados pela nossa cultura...
>
> Abraços a todos,
> Lucas
>
> Em 19 de fevereiro de 2010 11:49, Artur Costa Steiner <
> steinerar...@gmail.com> escreveu:
>
>   Qual é a crítica que ele faz aos números reais?
>>
>> Artur
>>
>>
>>
>> *De:* owner-ob...@mat.puc-rio.br [mailto:owner-ob...@mat.puc-rio.br] *Em
>> nome de *luiz silva
>> *Enviada em:* sexta-feira, 19 de fevereiro de 2010 08:36
>> *Para:* Matematica Lista
>> *Assunto:* [obm-l] Números Reais - MetaMAt
>>
>>
>>
>> Pessoal,
>>
>> Estou lendo o livro MetaMat de um matematico e cientista da computação.
>> Nele, existe uma crítica muito forte aos números reais, com utilização,
>> inclusive, da teoria da informa.
>>
>> Eu, particularmente, sempre tive dificuldades em "aceitar" os números
>> reais
>> , quanto a sua "realidade" e este livro, que encontrei por acaso na
>> Saraiva, veio como uma "luva" para minhas dúvidas.
>>
>> Alguém aqui já leu este livro ? Concorda com o ponto de vista do autor?
>> Tem argumentos contrários aos que ele expõe no seu livro ?
>>
>>
>> Abs
>> Felipe__=
>>
>>
>>  --
>>
>> Veja quais são os assuntos do momento no Yahoo! + Buscados: Top 
>> 10-
>> Celebridades-
>> 

[obm-l] Re: [obm-l] ENIGMAS CAÓTICOS!

2009-12-29 Por tôpico Paulo Santa Rita
Olá Jorge Luis e demais
colegas desta lista ... OBM-L,

Eu sei fazer os dois primeiros, mas a resposta nao e simples e nao e
curta. Fico devendo a publicacao no inicio do proximo ano. O terceiro
e trivial :

A deducao analitica e igualmente facil :

Como C(N)=N!( 1/(2!) - 1/(3!) + ... + ((-1)^N)*(1/N!)  ) entao
C(N) = N!((1/2!) - (1/3!) + ... + ((-1)^(N-1))*(1/(N-1)!) + (-1)^N
C(N) = N*(N-1)!*((1/2!) - (1/3!) + ... + ((-1)^(N-1))*(1/(N-1)!)) + (-1)^N
C(N) = N*C(N-1) + (-1)^N

O Fator N!/P! pode ser visto como uma permutacao linear entre N
objetos nos quais P sao iguais entre si : nisto esta a interpretacao
combinatoria. Para ver isso claramente considere P objetos dispostos
caoticamente em N posicoes. Podemos variar P de N-1 a 1 e em cada cada
caso teremos o total de "arranjos caoticos". Depois e so somar tudo.

A proposito, a expressao geral ( apresentada pela primeira vez aqui e
agora ) para N-1 objetos caoticamente distribuidos em N posicoes e :

AC(N,N-1) = N*( !N  + !(N-1) )

Para N-2 objetos e:

AC(N,N-2) = Bi(N,2)*(!N  + 2*!(N-1) + !N )

 a expressao geral prova-se usando o principio da inclusao-exclusao.
Mais pra frente, com mais calma, eu vou colocar aqui a demonstracao e
falar com mais detalhes.

Um Abraco a Todos !
PSR,32912090B1A




2009/12/28 Jorge Luis Rodrigues e Silva Luis :
> Olá, Pessoal!
>
> Sejam dadas duas permutações caóticas, digamos ABCDEFG e BCDEFGA. Quantas
> permutações simultaneamente caóticas em relação as duas podemos construir?
> (Engenhoso problema proposto pelo Paulo Santa Rita)
>
> Quantos são os anagramas da palavra MATEMATICA em que nenhuma das letras
> ocupa a posição ocupada na palavra MATEMATICA? Ou melhor, onde não há
> nenhuma coincidência de letra em nenhuma das posições? (Taí, um problema mal
> entendido e forte candidato ao desafio do ano 2010)
>
> Dar uma demonstração combinatória de que C(n)=n*C(n-1)+(-1)^n, onde
> C(n)=número de permutações caóticas de n objetos. (Proposto por Cláudio
> Buffara há décadas, mas permanecerá em aberto juntamente com o famoso
> problema das "Caixas de fósforos de Banach" por talvez mais uns cem anos
> adiante...)
>
> Um indivíduo fará uma reunião com 12 pessoas. Para enfatizar o caráter
> democrático da reunião todos sentarão em uma mesa redonda, cada qual em um
> lugar pré-estabelecido. O anfitrião, porém, é bisonho. E pode suceder que
> ele conduza cada participante a uma posição errada na mesa. Qual a
> probabilidade de isso ocorrer? (Esse é bacana! Parece que foi também de
> autoria do Paulo Santa Rita. Mas, afinal! Existe alguma relação entre um
> agrupamento linear e o seu correlato circular?)
>
>
> Abraços!
>
> 
> Agora a pressa é amiga da perfeição. Chegou Windows 7. Conheça.

=
Instru��es para entrar na lista, sair da lista e usar a lista em
http://www.mat.puc-rio.br/~obmlistas/obm-l.html
=


Re: [obm-l] Um criador de ovelhas

2009-12-11 Por tôpico Paulo Santa Rita
Olá Bruno e demaiscolegas desta lista  ... OBM-L,
E verdade. Este e o caso (0,1,0), equivalente a uma ovelha de 30 comuma ovelha 
de 18, que da uma media de 48/2 = 24 > 22.5
O linha de raciocinio esta correta, as (in)equacoes sao corretas.Apenas esqueci 
de considerar esta solucao.
Obrigado
PSR,61112090D2A
2009/12/11 Bruno França dos Reis :> Paulo, será que vc não 
esqueceu de contar a solução (0, 1, 0)?>> Isso corresponde à escolha da ovelha 
de 30kg e de uma ovelha de 18kg, o que> dá uma média de (30+18)/2 = 24kg > 
22.5kg.>> Dessa forma, teremos encontrado o mesmo resultado.>> Abraço,> 
Bruno>>> --> Bruno FRANÇA DOS REIS>> msn: brunoreis...@hotmail.com> skype: 
brunoreis666> tel: +33 (0)6 28 43 42 16>> http://brunoreis.com>> GPG Key: 
http://brunoreis.com/bruno-public.key>> e^(pi*i)+1=0>>> 2009/12/11 Paulo Santa 
Rita >>>> Ola Carlos e demaiscolegas desta lista ... 
OBM-L,( escreverei sem acentos)>> Vou supor que ovelhas de mesmo peso sao 
indistinguiveis.>> Nos temos 15 ovelhas. A soma dos pesos e  70 + 72 + 32 + 66 
+ 30 =270.>> Segue que o valor medio  sera 270/15 = 18. Somando a este valor25% 
dele>> mesmo, teremos  22,5. Esse é o nosso valor de referencia.>> E facil ver 
que se a ovelha de peso 30 nao entrar na escolha dos!
animais,>> entao o valor de referencia nao sera ultrapassado ( pois,neste caso, 
 o>> maior valor sera 22, inferior a 22.5 ). Portanto, umaescolha so vai 
atender>> o criterio de abate se, NECESSARIAMENTE,  aovelha de 30 kg estiver 
incluida.>>  Assim, na inequacao abaixo, INCLUIuma ovelha de 30 Kg. A inequacao 
que vai>> nos interessar sera :>> (30 + 22A + 18B + 16C + 14D) / ( 1 + A + B + 
C + D )   >   22.5>> Onde A, 0 =< A =< 3 e o total de ovelhas escolhidas das 
que pesam 22Kg; B,>> 0 =< B =< 4 e o total de ovelhas escolhidas das que pesam 
18 Kg; C, 0 =< C>> =< 2 e o total de ovelhas escolhidas das que pesam 16 Kge D, 
0 =< D =< 5 e o>> total de ovelhas escolhidas das que pesam 14 Kg.>> 
Simplificando a inequacao, temos :>> A + 9B + 13C + 17D < 15>> D #  0 nao nos 
interessa ( nao podemos escolher ovelha pesando 14 ).Logo,>> a inequacao se 
restringe a : A + 9B + 13C < 15. Olhando-a, efacil ver que o>> maior valor 
possivel para as variaveis B ou C e 1 eque somente uma das>>!
 variaveis B ou C podera ser 1, nao ambassimultaneamente. Fic!
a portanto facil>> encontrar as solucoes ( nassolucoes abaixo ja esta incluida  
uma ovelha de>> 30 Kg )  no formado(A,B,C) :>> SOLUCOES COM C=0 :>> (0,0,0)  -> 
apenas a ovelha de 30 Kg(1,0,0), (1,1,0)(2,0,0),>> (2,1,0)(3,0,0), (3,1,0)>> 
SOLUCOES COM C=1:>> (0,0,1)(1,0,1)>> Portanto, admitindo-se que animais de 
mesma massa sejamindistinguíveis,>> temos 9  possibilidades de escolha para que 
ocriterio de abate seja>> satisfeito.>> Um abraco a Todos !PSR,61112090C19>>>> 
2009/12/11 Carlos Gomes :> Olá amigos...será que>> algun 
de vcs já resolveu esta questão ou pode> resolverfoi da UFCG 2005>> 2a 
fase...>>> Um criador de ovelhas costuma adotar o seguinte critério para>> 
selecionar> animais para abater: do total de n animais escolhe-se p animais.>> 
Se a média> dos pesos  os p animais for maior que a média dos pesos dos n>> 
animais mais> 25% desta média, então os p animais  estão prontos para o>> 
abate. No curral> há 15 ovelhas  cujos pe!
sos em média são dados pela tabela>> abaixo.>>>> N° de animais            
Peso/animal        Peso(total - kg)>>>>>>  
5    14 70>>>>  
4    18 72>>>>  
2    16!>>  32>>  
3>> 22 66>>  1>> 
30 30>>>> As possibilidades existentes para que o>> 
critério de abate seja satisfeito> são>>>> a) 24  c) 18>>   
  e) 21>>>> b) 20  d) 17>>>>>>>> O>> gabarito é 
21 (alternativ!
a E)...não tô achando...>>>> valew, cgomes>> ==!
===>> Instru�ões para 
entrar na lista, sair da lista e usar a lista em>> 
http://www.mat.puc-rio.br/~obmlistas/obm-l.html>> 
=>>
=
Instru��es para entrar na lista, sair da lista e usar a lista em
http://www.mat.puc-rio.br/~obmlistas/obm-l.html
=


Re: [obm-l] Um criador de ovelhas

2009-12-11 Por tôpico Paulo Santa Rita
Ola Carlos e demaiscolegas desta lista ... OBM-L,( escreverei sem acentos)
Vou supor que ovelhas de mesmo peso sao indistinguiveis.
Nos temos 15 ovelhas. A soma dos pesos e  70 + 72 + 32 + 66 + 30 =270. Segue 
que o valor medio  sera 270/15 = 18. Somando a este valor25% dele mesmo, 
teremos  22,5. Esse é o nosso valor de referencia.
E facil ver que se a ovelha de peso 30 nao entrar na escolha dosanimais, entao 
o valor de referencia nao sera ultrapassado ( pois,neste caso,  o maior valor 
sera 22, inferior a 22.5 ). Portanto, umaescolha so vai atender o criterio de 
abate se, NECESSARIAMENTE,  aovelha de 30 kg estiver incluida.  Assim, na 
inequacao abaixo, INCLUIuma ovelha de 30 Kg. A inequacao que vai nos interessar 
sera :
(30 + 22A + 18B + 16C + 14D) / ( 1 + A + B + C + D )   >   22.5
Onde A, 0 =< A =< 3 e o total de ovelhas escolhidas das que pesam 22Kg; B, 0 =< 
B =< 4 e o total de ovelhas escolhidas das que pesam 18 Kg; C, 0 =< C =< 2 e o 
total de ovelhas escolhidas das que pesam 16 Kge D, 0 =< D =< 5 e o total de 
ovelhas escolhidas das que pesam 14 Kg.
Simplificando a inequacao, temos :
A + 9B + 13C + 17D < 15
D #  0 nao nos interessa ( nao podemos escolher ovelha pesando 14 ).Logo, a 
inequacao se restringe a : A + 9B + 13C < 15. Olhando-a, efacil ver que o maior 
valor possivel para as variaveis B ou C e 1 eque somente uma das variaveis B ou 
C podera ser 1, nao ambassimultaneamente. Fica portanto facil encontrar as 
solucoes ( nassolucoes abaixo ja esta incluida  uma ovelha de 30 Kg )  no 
formado(A,B,C) :
SOLUCOES COM C=0 :
(0,0,0)  -> apenas a ovelha de 30 Kg(1,0,0), (1,1,0)(2,0,0), (2,1,0)(3,0,0), 
(3,1,0)
SOLUCOES COM C=1:
(0,0,1)(1,0,1)
Portanto, admitindo-se que animais de mesma massa sejamindistinguíveis, temos 9 
 possibilidades de escolha para que ocriterio de abate seja satisfeito.
Um abraco a Todos !PSR,61112090C19

2009/12/11 Carlos Gomes :> Olá amigos...será que algun de 
vcs já resolveu esta questão ou pode> resolverfoi da UFCG 2005 2a 
fase...>>> Um criador de ovelhas costuma adotar o seguinte critério para 
selecionar> animais para abater: do total de n animais escolhe-se p animais. Se 
a média> dos pesos  os p animais for maior que a média dos pesos dos n animais 
mais> 25% desta média, então os p animais  estão prontos para o abate. No 
curral> há 15 ovelhas  cujos pesos em média são dados pela tabela abaixo. 
N° de animais            Peso/animal        Peso(total - kg)  
5    14 70>>  
4    18 72>>  
2    16!
 32>>  3    
22 66>>  1    
30 30 As possibilidades existentes para que o 
critério de abate seja satisfeito> são a) 24  c) 18 
e) 21 b) 20  d) 17 O gabarito é 21 
(alternativa E)...não tô achando... valew, cgomes
=
Instru��es para entrar na lista, sair da lista e usar a lista em
http://www.mat.puc-rio.br/~obmlistas/obm-l.html
=


Re: [obm-l] Teste de Primalidade AKS

2009-12-09 Por tôpico Paulo Santa Rita
Ola Joao de demais
colegas desta lista ... OBM-L,

Aqui ha uma boa descricao :
http://pt.wikipedia.org/wiki/Teste_de_primalidade_AKS

Aqui ha uma implementacao em C++ :
http://gpoulose.home.att.net/gc/src/AKS_cpp.txt

Uma abraco a todos !
PSR,4091209082A


2009/12/8 João Paulo V. Bonifácio :
> Pessoal,
>
> Estou tentando implementar o teste de primalidade AKS, mas estou tendo
> muitas dificuldades em fazê-lo.
> Alguém sabe como fazer ou pode disponibilizar o código de algum programa que
> faça este teste?
>
> Abraços!
>
> --
> João Paulo Vieira Bonifácio
>
> Universidade Federal de Uberlândia
> Faculdade de Engenharia Elétrica
> Programa de Educação Tutorial - PET/Eng. Elétrica
> Fone: (34) 9942 - 7427 / (34) 3239 - 4754

=
Instru��es para entrar na lista, sair da lista e usar a lista em
http://www.mat.puc-rio.br/~obmlistas/obm-l.html
=


Re: [obm-l] Problema

2009-11-16 Por tôpico Paulo Santa Rita
Ola benedito e demais
colegas desta lista ... OBM-L,
(escreverei sem acentos)


Seja An o conjunto de todos os triangulos cujos lados são numeros
inteiros menores ou iguais a N.  Entao, claramente,  An-1 esta contido
em An ...  Significa isso que - representando por (A) o numero de
elementos do conjunto A - podemos por  :

(An) = (An-1) + ( Bn)

onde Bn e o conjunto dos elementos de An que não estao em An-1. E
facil ver que os elementos de Bn são todos os triangulos de An nos
quais ao menos um lado vale N.

Quantos elementos tem Bn ?

Bom, a principio, e facil ver que em Bn esta o triangulo equilatero de
lado N. E igualmente facil perceber que Bn congrega tambem todos os
triangulos isosceles e nao-equilateros nos quais dois de seus lados
valem N, a saber, os triangulos {N,N,1}, {N,N, 2}, ... {N,N,N-1}.
Computando tudo isso temos N triangulos. Portanto :

(Bn)= N + (Cn)

onde Cn e o conjunto de todos os triangulos de An nos quais um, e
somente um, dos lados vale N, a saber, os escalenos cujo maior lado
vale N e os isosceles não equilateros cujos lados iguais são menores
que N. Os triangulos de lados {N,N-1,N-2} e {N,N-1,N-1} são exemplos
validos para esta duas classes.

Quantos são os elementos de Cn ?

Todos os elementos de Cn tem um único lado medindo N e, alem disso,
este lado e o maior lado. Isto implica que se representarmos
genericamente um destes triangulos por {N, A, B}, devera ocorrer :

1)A+B >=  N+1,  pois devemos ter N < A+B
2)A+B =< 2N-2,  pois A < N e B < N

significa isso que os elementos de Cn estao agrupados em classes
disjuntas, nas quais todos os elementos de uma mesma classe tem o
mesmo perimetro. Enumerando os elementos da classe {N,A,B} na qual
A+B=N+1 ate a enumeracao da classe {N,A,B} na qual A+B=2N-2 teremos
totalizado todos os elementos de Cn.

Seja portanto D2p ( indice “2p” ) a classe de triangulos {N, A, B} de
Cn na qual A+B=2p. Temos que 2p=N+1, N+2,..., 2N-2. Fixando uma D2p
qualquer, podemos IMAGINAR que cada elemento desta D2p e uma sequencia
de tres numeros, ordenados da esquerda para a direita, do maior lado
para o menor lado.

Agora, IMAGINE que as sequencias ordenadas descritas acima estao elas
mesmas ordenadas de forma decrescente pelo elemento  central ( o
segundo termo de cada 3-sequencia ). O que vemos ?

(N,  (N-1)-0,  (2p-N+1)+0)
(N,  (N-1)-1,  (2p-N+2)+1)
(N,  (N-3)-2,  (2p-N+3)+2)
...

E ate onde podemos descer ? Ate X tal que (N-1)-X  >= 2p-(N-1)+X pois
se (N-1)-X < p-(N-1)+X claramente que o triangulo {N,(N-1-X,p-(N-1)+X}
sera igual a algum dos anteriores, já computado. Assim :

X =< (N-1) – p.

Para considerar o valor X=0, fazemos: 1 + X  =<  N-p. E como 1+X deve
ser inteiro, para não dependermos da paridade de N, colocamos :

(D2p) = 1+X = piso(N – p)

De tudo que vimos chegamos a :
(An) = (An-1)  +  N  +  (Dn+1)  +  (Dn+2)  +  ...  +  (D2n-3)  +  (D2n-2)
o que resolve o problema original formulado pelo Benedito.

Agora, facamos alguns calculos praticos.

N=1  => A1= 1
Obvio, pois apenas o triangulo {1,1,1} atende as condicoes de simetria
do problema.

N=2  => A2= 3
Obvio, pois alem do triangulo {1,1,1} somente os triangulos {2,2,1} e
{2,2,2} interessam.

N=3 => A3 = A2 + 3 + D4 = 3 + 3 + piso(3-2) = 3 + 3 + 1 = 7
Os 4 novos triangulos são {3,3,3}, {3,3,2},  {3,3,1} e {3,2,2}

N=4 => A4=A3 +4+D5+D6 = 7 + 4 + piso(4 - 2,5) + piso(4 - 3)=13
Os 6 novos triangulos são {4,4,4},{4,4,3},{4,4,2},{4,4,1},{4,3,2} e {4,3,3}

Agora, vamos considerar com mais atencao a expressao que fornece o
numero de elementos de D2p:

(D2p)=piso(N - p)

Esta expressao e bonita ? Não sei … O que voces acham ? Eu tenho
minhas duvidas … A funcao “piso” da uma certa assimetria a formula,
tornando-a carrancuda. Ela e decididamente uma mulher com veu, mas eu
vou apostar e continuar esta viagem com ela para ver aonde ela me
conduz … Se ela for bela, ela sera fertil !

Esta formula nos diz quantos triangulos de lados inteiros positivos
tem perimetro p, com as limitacoes :

1)Um unico lado deve valer N
2)N+1 =<2p   =< 2N-2

E se quisessemos encontrar “todos os triangulos de lados inteiros que
tenham perimetro 2p”, independente das limitacoes acima ?  Nos temos
elementos suficientes para resolver esta questao diretamente ?

Temos. Eis como :

Se um triangulo de lados inteiros tem perimetro 2p, o maior lado
possivel deve ser L= p -1 se 2p e par; deve ser L=p – 0,5 se 2p e
impar, pois em qualquer triangulo, o maior lado deve ser menor que a
soma dos outros dois. Sintetizamos tudo isso pondo L = piso(p – 0,5).

E o menor maior lado possivel ? E claro que se M e o menor maior lado
possivel deve ocorrer que 3M  >= 2p. Assim, o menor maior lado
possivel e o menor M tal que 3M >= 2p  =>  M=teto(2p/3).

Usando a notacao  Si{ A,B : f(i) }=f(A) + f(A+1) + … + f(B-1) + f(B)
para representar o somatorio e aplicando a expressao D2p=piso(N-p) aos
resultados acima, chegamos a :

T(2p) = ( L – M + 1) + Si{ M , L : piso( (3i/2) – p ) }
onde M=teto(2p/3), L=piso(p-0,5) e T(2p) e o numero de triangulos com
lad

Re: [obm-l] 4444^4444

2009-11-11 Por tôpico Paulo Santa Rita
Olá Eric e demais colegas
desta lista ... OBM-L,

Sobre o ^ vou reproduzir aqui a belissima solucao do
Alessandro Madruga Correia, onde se sugere indiretamente a imensa
superioridade do Linux sobre o Windows

Alessandro Madruga Correia escreveu :

>Boa tarde,
>
>Já que é para usar recursos computacionais
>
>amcorr...@chronos:~$ resultado=$(calc --  ^ )
>amcorr...@chronos:~$ echo -n $resultado | tr -d 0-6 | tr -d 8-9 |  wc -c
>1605

para quem usa o GNU/Linux, o comando "bc"  faz ^ brincando e
imediatamente - NA LINHA DE COMANDO ! - sem ser necessario aprender
MAPLE  ou fazer qualquer programa ou usar qualquer outro utilitário do
genero. Se voce estudar o MAXIMA, entao nunca mais vai querer falar do
(r)Windows.

A proposito, a minha distribuicao é o Debian/GNU Linux, "o sistema
operacional universal"
http://www.debian.org/index.pt.html

Um Abraco a todos
PSR, 40B0B250B0B

2009/11/11 Eric Campos Bastos Guedes :
>
>  <40231.10.1.2.2.1251131505.squir...@webmail.viaconnect.com.br>
> Content-Type: text/plain; charset="iso-8859-1"
> Content-Transfer-Encoding: quoted-printable
> MIME-Version: 1.0
>
>
> Confirmado. O numero ^ tem 1605 algarismos '7'.
>
> Basta calcular pelo programinha Maple:
>
> contar_setes :=3D proc(n):
> =A0=A0 f :=3D x -> x - 10*floor(x/10):
> =A0=A0 g :=3D x -> piecewise(f(x)=3D7=2C1):
> =A0=A0 m :=3D n=3B
> =A0=A0 quantos_setes :=3D 0:
> =A0=A0 algarismos :=3D floor(log[10](n))+1:
> =A0=A0 for i from 1 to algarismos do
> =A0=A0=A0=A0 quantos_setes :=3D quantos_setes+g(m):
> =A0=A0=A0=A0 m :=3D floor(m/10):
> =A0=A0 od:
> =A0=A0 print('o_n=FAmero_dado_tem'=2Cquantos_setes=2C'algarismos_sete')=3B
> =A0end=3B
>
> A saida eh:
>
> contar_setes(^)=3B
> =A0=A0=A0=A0=A0=A0=A0=A0=A0=A0=A0=A0=A0=A0 o_n=FAmero_dado_tem=2C 1605=2C a=
> lgarismos_sete
>
>
> [ eric campos bastos guedes -- ]
> [ matem=E1tico=2C escritor e pesquisador - ]
> [ "A verdade tem v=E1rias faces e v=E1rias fontes" ]
> [ twitter: mathfighter --- ]
> [ Orkut: Eric Campos Bastos Guedes --- ]
> [ e-mail/MSN: fato...@hotmail.com  ]
> [ cel. (0xx 21) 8721-5420  ]
>
>
>
>
>
>
>
> 
>> Date: Mon=2C 24 Aug 2009 13:31:45 -0300
>> Subject: Re: [obm-l] ^
>> From: amcorr...@viaconnect.com.br
>> To: obm-l@mat.puc-rio.br
>>
>> Boa tarde=2C
>>
>> J=E1 que =E9 para usar recursos computacionais
>>
>> amcorr...@chronos:~$ resultado=3D$(calc --  ^ )
>> amcorr...@chronos:~$ echo -n $resultado | tr -d 0-6 | tr -d 8-9 | wc -c
>> 1605
>>
>> N=E3o entendi esse teu processo de 'pegar as orelhas dos d=EDgitos'...?
>>
>>
>>
>>> Bom=2C fiz a conta aqui de cabe=E7a=2C deu um numerozinho:
>>>
>>> 51 036325037 255080482 040250195 524395924 782475482
>>> 284514747 828975986 739479076 862531615 833001864 027197875 434172496
>>> 782212810 360936998 814871028 023280409 517604289 704232997 250059817
>>> 651452033 902564981 053474342 143982434
>>> ... [muitas MUITAS linhas deletadas]...
>>> 954187355 586887636 003568084 241221478 621695307 152384094 531375297
>>> 756356083 583426234 545493926 295612017 618527949 708463952 945505173
>>> 247787325 422994467 678743279 370416826 869347245 921635509 244741066
>>> 157981696
>>>
>>> Deixa eu ver=2C 16211 d=EDgitos. Ah=2C pera a=ED=2C errei a conta l=E1 n=
> o meio=2C
>>> ali =E9 8 e n=E3o 9 Mas tudo bem=2C agora =E9 s=F3 contar os 7 (cont=
> ei as
>>> orelhas dos d=EDgitos e dividi por 2) s=E3o 1605 d=EDgitos de n=FAme=
> ro 7.
>>> =3B) =3B) =3B)
>>>
>>> Abra=E7o=2C Ralph.
>>>
>>> P.S.: Bom=2C o que eu quero dizer =E9 que n=E3o me parece haver um m=E9t=
> odo
>>> "ol=EDmpico" para resolver este tipo de problema (se algu=E9m descobrir=
> =2C
>>> me ensine!)... Mas pior que o que eu fiz acima deve estar certo.
>>>
>>> 2009/8/24 douglas paula :
 ol=E1 amigos da lista=2C
 trago um problema que vem me enrolando h=E1 alguns dias e=2C embora j=
> =E1 tenha
 pedido ajuda em algumas comunidades sobre matem=E1tica no orkut=2C aind=
> a n=E3o
 tenho uma solu=E7=E3o:

 Quantos algarismos "7" existem no resultado de ^ ? (considere a
 nota=E7=E3o decimal)

>>
>> =3D=3D=3D=3D=3D=3D=3D=3D=3D=3D=3D=3D=3D=3D=3D=3D=3D=3D=3D=3D=3D=3D=3D=3D=
> =3D=3D=3D=3D=3D=3D=3D=3D=3D=3D=3D=3D=3D=3D=3D=3D=3D=3D=3D=3D=3D=3D=3D=3D=3D=
> =3D=3D=3D=3D=3D=3D=3D=3D=3D=3D=3D=3D=3D=3D=3D=3D=3D=3D=3D=3D=3D=3D=3D=3D
>> Instru=E7=F5es para entrar na lista=2C sair da lista e usar a lista em
>> http://www.mat.puc-rio.br/~obmlistas/obm-l.html
>> =3D=3D=3D=3D=3D=3D=3D=3D=3D=3D=3D=3D=3D=3D=3D=3D=3D=3D=3D=3D=3D=3D=3D=3D=
> =3D=3D=3D=3D=3D=3D=3D=3D=3D=3D=3D=3D=3D=3D=3D=3D=3D=3D=3D=3D=3D=3D=3D=3D=3D=
> =3D=3D=3D=3D=3D=3D=3D=3D=3D=3D=3D=3D=3D=3D=3D=3D=3D=3D=3D=3D=3D=3D=3D=3D
>                                         =20
> _
> Voc=EA j=E1 ama o Messenger? Conhe=E7a ainda mais sobre ele n

[obm-l] Re: [obm-l] Lucro Máximo

2009-11-10 Por tôpico Paulo Santa Rita
Ola Gustavo e demais
colegas desta lista ... OBM,

Eu vejo as coisas assim :

Se cada exemplar for vendido ao preco unitario de 100 - 5X as vendas
subirao para 180+30X o que implicara numa receita bruta de
(180+30X)*(100-5X). Como cada exemplar custa 40, o lucro obtido sera
L(X) = (180+30X)(100-5X) - 40(180+30X)= (180+30X)*(60-5X). O objetivo
e maximizar L(X) com a condicao de que 100-5x > 40  => X < 12.

Se eu nao errei nenhum calculo acima, a parabola L(X) tem maximo em
X=3, o que atende a condicao X < 12. Logo, cada exemplar deve ser
vendido a 100-3*5= 85

Esse e um tipico problema elementar de pesquisa operacional. Para mais
variaveis, existe um metodo charmoso chamado de Metodo Vogel. Eis aqui
uma questao que pode ser equacionada
com os metodos da pesquisa operacional:

PROBLEMA : Vai comecar um campeonato de futebol com um unico turno (
cada clube joga com todos os outros uma unica vez ) no qual participam
2N clubes. Uma vitoria vale 3 pontos, o empate vale um ponto e derrota
nao confere pontos. Ao final, os P melhores classificados ( P < N )
passarao para a proxima fase. ANTES DE COMECAR O CAMPEONATO, qual sera
a quantidade minima de pontos que um clube devera fazer para ter
certeza de estara entre os P primeiros classificados.

OBS1 : note que conforme o campeonato vai avancando o minimo de pontos
para estar entre os P primeiro muda, ou seja, ele e funcao da
"rodada".

OBS2 : Pense numa classificacao em termos de pontos perdidos que tudo
fica mais facil

um abraco a todos
PSR,3101109083A


09/11/9 Gustavo Duarte :
> É uma questão básica de ponto máximo/ mínimo,  mas nem tanto 
>
>
>  "Ao preço de $ 100,00 cada exemplar, um vendedor ambulante vende 180
> exemplares de um uma mercadoria com um custo unitário de $ 40,00 o exemplar.
> Este vendedor estima que, para cada $ 5,00 de desconto no preço unitário,
> fará aumentar 30 exemplares nas vendas. O preço da mercadotia para maximizar
> O LUCRO desse vendeor é :  "
>
> Dúvida : na função da venda :V (X) = (100 - 5x).( 180 + 30x) , Xv = 7  logo
> preço = $ 65
>
>    na função do lucro : L (X)  = ( 60 - 5x) . ( 180 + 30x) , Xv = 3
> , logo lucro = 45, assim prço ( venda) = 45 + 40 = $85.
>
> Em qual preço (de venda) terei um lucro máximo ???  Quem puder ajudar ,desde
> já agredeço .

=
Instru��es para entrar na lista, sair da lista e usar a lista em
http://www.mat.puc-rio.br/~obmlistas/obm-l.html
=


[obm-l] Re: [obm-l] Polinômios (2)

2009-11-03 Por tôpico Paulo Santa Rita
Ola Bluesman e demais
colegas desta lista ... OBM-L,
(escreverei sem acentos)

Considerando que voce esta se referindo a uma prova que esta testando
conhecimentos de nivel medio, a sua resposta esta correta.

Alias. essa prova esta muito mais para "pegadinha" do que para
"afericao de conhecimento" ... pois dizer que um polinomio tem 16
raizes complexas e falar muito pouco ( isso apenas implica que no
CONTEXTO HABITUAL ONDE TAIS QUESTOES SAO PROPOSTAS, o grau do
polinomio nao e menor que 16 ) e, alem disso, o que foi dito nao
contribui em nada para a solucao da questao : e muito mais uma forma
de desviar a atencao do estudante do que fornecer um dado importante
para a solucao. Deploravel, portanto !

Note que existem contextos em que um polinomio de grau N tem mais que
N raizes, sem que isso signifique uma derrogacao do Teorema
Fundamental da Algebra. ( veja isso aqui :
http://www.mat.puc-rio.br/~nicolau/publ/publ.html ) . Aqui esta um
exemplo do que eu falei de ser algo pouco falado ( que nao faz parte
da Matematica da Moda ) mas que, em minha opiniao, vai se tornar muito
importante num futuro proximo.

Alias, foi por isso que eu disse que no CONTEXTO HABITUAL ONDE TAIS
QUESTOES SAO PROPOSTAS, vale dizer, onde impera o teorema fundamental
da algebra, o algoritmo de divisao e o euclidiano, estamos num corpo
ordenado completo etc etc etc

um aoutra forma de verificar que a sua resposta esta correta e
eliminando as demais opcoes, pois absurdas.

Um abraco a todos !
PSR, 20311091338






2009/11/3 Bluesman :
> Olá a todos,
>
>
>
> Enviei a questão abaixo para a lista há mais ou menos dez dias.
>
> Como até agora não houve qualquer comentário, segue o meu raciocínio:
>
> Independentemente do grau de R(x), temos que o grau de B(x) é maior do que o
> grau de R(x). Portanto, ao dividirmos R(x) por B(x), temos como quociente o
> polinômio nulo. E como o polinômio nulo admite infinitas raízes, concluímos
> que a alternativa correta é a (A).
>
>
>
> Certo ou errado?
>
>
>
> Trata-se de uma questão cobrada no concurso para Professores organizado pelo
> DEPENS (Departamento de Ensino da Aeronáutica).
>
> A prova apresenta algumas questões interessantes que podem ser úteis aos
> colegas professores (para fazer o download basta acessar o site da EPCAR).
>
> Espero não ter sido precipitado reenviando o problema e aproveito para
> recomendar um livro muito bom sobre polinômios: Polynomials, de E.J.
> Barbeau. Springer.
>
>
>
> [  ]'s.
>
>
>
> Numa divisão de polinômios, dividindo-se o polinômio A(x) , que tem
>
> exatamente 16 raízes complexas, por B(x) , encontra-se o quociente
>
> C(x) e o resto R(x).
>
> Sabe-se que
>
> . B(x) <> 0  (B(x) diferente de zero)
>
> . C(x) e B(x) possuem o mesmo número de raízes complexas;
>
> . R(x) tem o maior grau possível nesta divisão.
>
> É correto afirmar que, na divisão dos polinômios R(x) por B(x) ,
>
> encontra-se um polinômio
>
> a) quociente que possui infinitas raízes.
>
> b) resto de grau zero.
>
> c) quociente que é um polinômio unitário.
>
> d) resto que possui 8 raízes complexas.
>

=
Instru��es para entrar na lista, sair da lista e usar a lista em
http://www.mat.puc-rio.br/~obmlistas/obm-l.html
=


[obm-l] Re: [obm-l] RES: [obm-l] Boa prova de Matemática

2009-11-02 Por tôpico Paulo Santa Rita
Ola Nehab e demais
colega desta lista ... OBM-L,

Eu tenho com o IME uma divida de gratidao impagavel ...

Eu ainda nao tive tempo para olhar a prova, mas, baseando-me nas declaracoes
( abaixo ) do carissimo Nehab, fico feliz ... parece que a mediocridade de
anos passados acabou.

Eu ja defendi aqui que o IME deveria (deve) colocar questoes de carater
olimpico em todos os seus vestibulares. Isto, ao meu ver, nao só prestigia o
movimento olimpico como contribui para que aqueles estudantes que decoram
"metodos de solucao" nao sejam bem sucedidos.

Matematica e criatividade, e reflexao, nao e adestramento. Além disso, o
futuro engenheiro que ja vem com esta visao e bagagem olimpicas, muito
provavelmente, sera um projetista melhor que aqueles que so aprendem
"receitas prontas". Na vida atual e sobretudo no futuro proximo, de todo
profissional decente e e sera exigido habilidade no USO CRIATIVO do
conhecimento. A erudição, quando nao acompanhada de criatividade nao passa
de um e aspecto da mediocridade.

Se o IME enveredou por esta vertente e nela pretende se manter, viva o IME !




Esta prova NÃO teve este perfil. Pelo menos este ano a prova tá mais para os
alunos mais brilhantes (talvez com perfil olímpico) e não vejo problema com
isto (no mínimo as questões 6 a 10 - 50% da prova sugerem isto...).  Que bom
se os melhores em "capacidade de interpretação do que se lê" sejam os alunos
do IME no ano que vem...

>
> Abraços
> Nehab, o eterno apaixonado por tudo que diga respeito ao IME... :-)
>
>
>
>


[obm-l] Re: [obm-l] O que ocorre ao mover o Foco da Parábol a ?

2009-08-04 Por tôpico Paulo Santa Rita
Ola Marcelo e demais
colegas desta lista ... OBM-L,
(escreverei sem acentos)

1) Tradicionalmente, as letras "a" e "b" sao usadas para representar
os semi-eixos da elipse e da hiperbole. Ocorre que no software, o
programador tem a liberdade de representar estes objetos ao modo dele
... pode ocorrer, por exemplo, que aquilo que nos matematicos
entendemos por semi-eixo maior e comumente representamos por "a" seja
presentado por "a/2". Assim, ao alterar o "a" do programa voce pode
estar alterando o "eixo-maior".

Em que pese estas liberdades, e certo que a forma da elipse e da
hiperbole dependem dos valores dos semi-eixos principais e, portanto,
alterar estes valores deve alterar a forma das conicas.

2) A parabola, por definicao, e o conjunto dos pontos de um plano
equidistantes de uma reta fixa ( DIRETRIZ) e de um ponto fixo (FOCO).
Assim, se voce alterar a reta diretirz ou o foco (ou ambos ) e normal
que a forma da parabola se altere.

Voce deu "uma pista" sobre o programa que desenha a parabola.

Se voce tracar uma reta que contem o foco e e perpendicular a reta
diretiriz, o ponto medio do segmento que liga o foco ao pe desta
perpendicular pertencera a parabola ( pois e, claramente, equidistante
do foco e da reta diretriz). Se alterando o foco ( ou a reta diretriz)
a reta diretriz se altera e porque o programador, muito provavelmente,
pensou assim :

"O usuario fornecendo a reta diretriz e o foco basta eu partir do
ponto medio ( construir o vertice da parabola ) e, a seguir, ir
construindo outros pontos equidistantes. A seguir, posso usar curvas
de Berzier para tracar as demais partes da parabola."

Note que esta maneira de ver as coisas e uma decisao do programador,
mas eu afirmo que este sofware poderia ter mais qualidade, pois nada
nos impede de TORNAR INDEPENDENTE a posicao do foco do movimento da
diretriz, vale dizer, e possivel construir um programa no qual a
alteracao da reta diretirz ( ou do foco ) NAO AFETA a posicao do foco
( reta diretirz). Esse engenheiro de sistemas estudou pouca matematica
...

Um abracao
PSR, 30408090A24


2009/8/4 Marcelo Gomes :
> Olá pessoal da lista muito bom dia.
>
> Estou montando as parametrizações das cônicas e estou seguindo pelo livro do
> Lehmann. Já montei as da Elipse e da Hipérbole. Utilizei-me de variáveis a e
> b para ambas. Em meu entendimento, por favor me corrijam se estiver errado,
> a e b funcionaram como se fossem os valores dos eixos maior e menor
> respectivamente, tanto da elipse quanto da hipérbole. Apliquei esta situação
> ao Software Régua e Compasso de Geometria Dinâmica e na medida em que
> alterava os valores de a e/ou de b as cônicas também se alteravam.
>
> Mas estou tendo uma grande dúvida em relação á Parábola. Pelo livro do
> Lehmann, págna 241 ele dá como parametrização para a Parábola as seguintes
> expressões:
>
> x = p * cotg^2 (t)   -> (p vezes cotangente ao quadrado de t) e
>
> y = 2*p cotg (t) -> (duas vezes p vezes cotangente de t) , onde p é a
> distância do Foco à Reta Diretriz.
>
> Minha Dúvida:
>
> O que ocorre se movimentarmos o Foco da Parábola ? Ou mesmo a reta Diretriz
> ? Quando apliquei no programa estas condições e movimentei a reta Diretriz
> automaticamente o Foco variou sua posição e a curva da parábola também. Está
> correta a alteração da curva da parábola pela alteração do Foco ou mesmo da
> Reta Diretriz ?
>
> Bem pessoal, desculpe se a dúvida é muito básica, talvez não tenha entendido
> alguma coisa, mas para mim estou acahndo coerente que a curva se altere já
> que a escrevi em função de p.
>
> Abração, Marcelo.
>
>
>
>
>

=
Instru��es para entrar na lista, sair da lista e usar a lista em
http://www.mat.puc-rio.br/~obmlistas/obm-l.html
=


[obm-l] Re: [obm-l] REFORÇO COMBINATÓRIO!

2009-07-15 Por tôpico Paulo Santa Rita
Ola Jorge e demais colegas
desta lista ... OBM-L,

Supondo que num mesmo hotel voce diferencia os quartos disponiveis (
exemplo : (joao,hotel A,quarto 1) # (joao, hotel A, quarto 2) ), eu
pensaria assim :

O que caracteriza univocamente uma alocacao e um trio da forma
(hotel,rapaz, quarto). Consideremos, a principio, o caso em cada rapaz
fica em um hotel ( nenhum hotel com 2 rapazes )

1) Escolhemos 3 hoteis. Isso pode ser feito de 4 formas. Fixada uma
escolha, podemos permutar os rapazes pelos hoteis escolhidos de 3!=6
modos. Teremos portanto 4*6=24 formas de colocar 3 rapazes em tres
hoteis. Fixados uma dessas escolhas, podemos varia cada rapaz em um
dos dois quartos de cada hotel, dando 24*2*2*2=24*8=192 maneiras.

Existe tambem a possibilidade de alocar dois rapazes em um hotel,
ficando o terceiro rapaz em um dos tres hoteis restantes. Para ver
como e possivel fazer isso, considere o seguinte :

2)Podemos escolher dois rapazer de 3 maneiras. Como ha 4 hoteis, isso
da 3*4 = 12 maneiras de alocar dois rapazes em um hotel. Em cada uma
destas alocacoes podemos permutar os quartos onde os rapazes ficarao,
dando portanto 12*2 = 24 maneiras distintas de alocar 2 rapazes em um
dos hoteis. Fixado qualquer uma destas maneiras, há 6 possibilidades
de alocar o terceiro rapaz ( pois restam 3 hoteis, cada um com 2
quartos ). Logo, o total de possibilidades e 24*6= 144

1) + 2) = 192+144=   336 maneiras.

Um abraco a todos
PSR,40E07090F2A


2009/7/15 Jorge Luis Rodrigues e Silva Luis :
> Turma! Tenho a ligeira impressão que estou mesmo precisando de "aulas de
> reforço", pois em um único problema cheguei a incrível marca de 4 respostas
> diferentes...e o pior pelo menos três dessas respostas estão erradas, se não
> todas...
>
> Três estudantes chegaram juntos a uma cidade para participar de um concurso
> e, não tendo feito reservas com antecedência, constataram que, em cada um
> dos quatro hotéis da cidade, existam apenas duas vagas disponíveis.
> Sabendo-se que os três não podem ficar juntos num mesmo hotel, pode-se
> afirmar que o número máximo de pessoas de hospedagem de que dispõem é igual
> a:
>
> 1) Você pode formar 3 duplas diferentes C3,2=3 e como são 4 hotéis elas
> podem ser acomodadas de 3*4=12 maneiras diferentes. Em cada caso restam, nos
> outros 3 hotéis 6 quartos diferentes para serem ocupados pelo aluno
> restante: 12*6=72
>
> 2) Dois no mesmo hotel (e outro em um dos 3 restantes): C3,2=3 maneiras de
> formar pares (C3,2)*4=12 maneiras de acomodar uma dupla em 4 hotéis. 3
> maneiras de acomodar o terceiro estudante. parcial: 12*3=36 maneiras. Um em
> cada hotel: 3*4=12. Total = 48.
>
> 3) Cada um dos 3 rapazes devem ficar sòzinhos em um dos 4 hotéis: Fixando
> qualquer um dos rapazes no 1º hotel os demais ficam automaticamente
> definidos. Essa situação gera 6 combinações possíveis, então: 6*4=24
> maneiras de dispormos os 3 rapazes: sendo 1 em cada 1 dos 4 hotéis
> existentes. Colocando 2 rapazes num mesmo hotel sobram 3 hotéis para o
> terceiro rapaz se alojar. Para 2 rapazes juntos e 1 terceiro sòzinho,
> existem: 4*3=12 maneiras de dispô-los nos 4 hotéis. Como os três rapazes
> combinados 2 a 2 geram mais 3 situações, então: 3*12=36 maneiras de
> dispormos os 3 rapazes: sendo 2 em cada 1 dos 4 hotéis e 1 em cada um dos 3
> hotéis restantes. Portanto: 24+36=60.
>
> 4) 8 * 6 * 4 = 192.
>
>
> A propósito, para chegar à estação final de uma estrada de ferro passo por 8
> estações. De quantos tipos de passagens disponho? (Essa é boa!!!)
>
>
> Abraços!
>
> 
> Novo Internet Explorer 8: mais rápido e muito mais seguro. Baixe agora, é
> grátis!

=
Instru��es para entrar na lista, sair da lista e usar a lista em
http://www.mat.puc-rio.br/~obmlistas/obm-l.html
=


Re: [obm-l] off topic off topic

2009-07-10 Por tôpico Paulo Santa Rita
Olá Hermann,

Eu recebi esta mensagem.

um abraco
PSR,61007090922

2009/7/10 Hermann :
> Não estou recebendo mais os emails da lista?
> Estou enviando esta mensagem para ver se a lista recebe meus emails.
>

=
Instru��es para entrar na lista, sair da lista e usar a lista em
http://www.mat.puc-rio.br/~obmlistas/obm-l.html
=


[obm-l] Re: [obm-l] Sequências de números reais

2009-07-06 Por tôpico Paulo Santa Rita
Ola Aline,

A demonstracao direta costuma esconder a "essencia da coisa". E
necessário voce visualiza-la antes de monta-la. No caso particular sob
consideracao, IMAGINE o ponto medio entre "a" e "b", isto e, imagine
c=(a+b) / 2. Vai chegar um momento que os Yn's  ESTARAO e PERMANECERAO
a direita de "c" e os X's ESTARAO e PERMANECERAO a esquerda de "c".
Quando isso ocorre teremos que Xn < Yn ...

Rigorosamente falando, podemos escrever assim :

Seja E = (b - a) / 2. Entao E > 0, pois b > a. Logo, por definicao de
LIMITE, temos que :

1) Existe um natural N1 tal que n > N1 implica Xn pertence a (a - E, a
+ E). Como E=(b-a)/2 segue que existe N1 tal que n > N1 implica Xn <
a+E = (a+b) /2, isto e, n > N1 => Xn < (a+b) / 2

2) Existe um natural N2 tal que n > N2 implica Yn pertence a
(b-E,b+E). Como E=(b-a)/2 segue que
existe N2 talo que n > N2 implica Yn > b-E = (a+b) /2

Tomando N3 = max{N1,N2} vemos que para n > N3 implica que Xn < (a+b)/2
< Yn, ou seja , para todo natural n > N3 teremos que Xn < Yn, que é o
que queriamos demonstrar.

Um abraco a todos !
PSR,21807091207









Como  a < b,  seja  E = (b - a) / 2. Entao E > 0. Por definicao existe
um natural No tal que N > No implica que Yn pertence a (b-E,b+E), vale
dizer,

2009/7/6 Aline Correa :
> Estou tentando resolver os exercícios do capítulo 3 do livro de Análise Real
> I do Elon e não estou conseguindo fazer algumas questões. Alguém poderia me
> ajudar?
> Segue abaixo as questões:
>
> Sejam lim xn = a e lim yn = b. Se a < b, prove que existe n0 pertence N tal
> que n > n0 => xn < yn.
>
> Diz-se que (xn) é uma sequência de Cauch quando, para todo E > 0 dado,
> existe n0 pertence N tal que m, n > n0 => |xm - xn| < E.
>
> Desde já grata.

=
Instru��es para entrar na lista, sair da lista e usar a lista em
http://www.mat.puc-rio.br/~obmlistas/obm-l.html
=


[obm-l] Re: [obm-l] GEOMETRIA COMBINATÓRIA!

2009-06-03 Por tôpico Paulo Santa Rita
Ola Jorge e demais colegas
desta lista ... OBM-L,

>
> Dados n pontos no plano (n>=3), o número de distâncias distintas entre eles
> é, pelo menos (n-3/4)^1/2-1/2 . (Problema Difícil!)
>

1) No excerto de mensagem acima voce deve estar se referindo a { [ N -
(3/4) ]^(1/2) } - (1/2) e nao a { [ (N - 3 ) /4) ]^(1/2) } - (1/2),
pois esta ultima expressao fornece ZERO distancias distintas quando
N=4, o que esta obviamente errado, visto que e facil ver que qualquer
configuracao de 4 pontos distintos num plano darao origem ao menos a
duas distancias distintas.

2) Mesmo se a expressao correta for { [ N - (3/4) ]^(1/2) } - (1/2),
tambem aqui ocorrem problemas, pois, sendo obvio ululante que tal
expressao fornece numeros irracionais para diversos N's, como
entender, por exemplo, que 5 pontos ( N=5 ) distintos no plano darao
origem ao menos a ( sqrt(17) - 1) / 2 distancias distintas ?

Voce esta se referindo ao piso de { [ N - (3/4) ]^(1/2) } - (1/2),
vale dizer, ao maior inteiro que ultrapassa { [ N - (3/4) ]^(1/2) } -
(1/2) ?

3) Mesmo se a expressao correta for PISO(  { [ N - (3/4) ]^(1/2) } -
(1/2)  ) ha problemas, pois para N=5 teremos  PISO(  { [ 5 - (3/4)
]^(1/2) } - (1/2)  ) = 1. Ora, 5 pontos distintos no plano darao
origem ao menos a 2 distancias distintas ( um pentagono regular
convexo, por exemplo).

Sera TETO(  { [ N - (3/4) ]^(1/2) } - (1/2)  ), onde TETO(X) e o menor
inteiro maior ou igual a X ?

Enfim, qual a expressao correta ? O problema pode ser formulado como segue :

PROBLEMA : Determine o numero minimo de distancias distintas exibidas
por N pontos distintos de um plano.

?

Um Abracao
PSR, 40306090D02

=
Instru��es para entrar na lista, sair da lista e usar a lista em
http://www.mat.puc-rio.br/~obmlistas/obm-l.html
=


Re: [obm-l] Triangulos e inteiros

2009-05-25 Por tôpico Paulo Santa Rita
Ola Eduardo e demais
colegas desta lista ... OBM-L

Eu disse que a solucao era "truculenta" porque nao parei para rever a
solucao, procurando melhora-la de alguma forma. Publiquei o que fui
escrevendo conforme vinha na minha cabeca. Mas o que eu queria mesmo e
ver voce novamente aqui, aumentando o nivel das discussoes da lista.

Quando eu ainda era crianca, eu li a demonstracao do Arquimedes
segundo a qual a area de um segmento da parabola e 4/3 do triangulo de
mesma base e igual altura. Ele usava o famoso "metodo da exaustao", um
dos precurssores do nosso atual Calculo Integral.  Resolvi entao fase
algo parecido com a hiperbole. Eu desejava calcular a area de um
segmento hiperbolico ( sem usar Calculo Dif ou/e Calculo Int, mesmo
porque na epoca eu nao conhecia estas ferramentas ) em funcao da area
do triangulo otimo, vale dizer, do traingulo com mesma  base e igual
altura.

Naquela epoca, o pomposo e orgulhoso titulo que imaginei foi : "Area
de um segmento hiperbolico pelo metodo da exaustao de Arquimedes"

E olha que nao so e possivel  calcular essa area como tambem se
descobre outras coisas realmente notaveis ... E entao, como fazer
isso, isto e, SEM USAR CALCULO DIF E INT,  como calcular a area de um
segmento hiperbolico em funcao do triangulo que tem a mesma base do
segmento hiperbolico ( uma corda da hiperbole ) e  altura ?

Fica o problema .

Um abraco a todos !
PSR,2250509082F



2009/5/25 Eduardo Wilner :
> Viva Paulo, Carlos e colegas da lista.
>
> Desculpem  meu atraso mas não recebí sua resposta no meu e-mail e agora, por
> acaso encontrei-a (ou as) diretamente na Lista. Estranho. Mas navegando na
> Internet muitas vezes sinto-me como na Intergaláctica. Pudera. Fui criado
> tendo tambores e sinais de fumaça como importantes meios de comunicação (
> pelo menos nos westerns das matinês de domingo).
>
> Mas vamos aos triângulos ou aos inteiros.
>
> Como sempre, seu trabalho é primoroso Paulo, entretanto não é difíci
> deixá-lo um pouco menos "truculento", como você diz, i.e, diminuir um pouco
> a mão de obra.
>
> Mantenho sua simbologia para os lados (A,B e C) , mas, como vc. mesmo
> observou no item 1), sendo X,Y e Z inteiros acho que fica mais comodo
> trabalhar com x=X/2, y=Y/2 e z=Z/2, inteiros positivos ( pares ou impares).
>
> Assim sua expressão (1) fica xyz = 4(x+y+z) e XY =< 48 fica como xy =< 12.
>
> Agora, considerando x =< y =< z ( equivale a C =< B =< A) , temos z.x^2 =<
> 12.z  ou
>
>                           x =<3 (*).
>
> Também obtemos z = 4.(x+y)/(xy-4) (**), que só admite uma solução com z
> inteiro positivo e x e y ambos impares, dentro do intervalo
>
>      4 < xy =< 12   que é  (x,yz) = (1,5,24) correspondendo ao triângulo
> (A,B,C) = (29,25,6).
>
> Soluções com x e y de paridades diferentes exigem  o denominador de (**) xy
> - 4
>
> a) igual a 2 que leva a xy = 6 com soluções (x,y,z) = (1,6,14)
> correspondente a
>
>    (A,B,C) = (20,15,7)    e   (x,y,z) = (2,3,10) => (A,B,C) = (13,12,5).
>
> b) igual a 4 que leva a xy = 8 com a unica possibilidade (x,y,z) = (1,8,9)
> correspondendo
>
>    a (A,B,C) = (17,10,9).
>
> A unica solução com x e y ambos pares é x = 2 devido a condição (*)  e y = 4
> com
> z = ¨6, já que para x=2 e y = 6 teriamos z = 4 o que contraria a escolha y
> = dois lados).
> Corresponde a (A,B,C) = (10,8,6).
>
> Portanto sua solução está correta.
>
> Um abraço.
>
> Eduardo Wilner
>
>
>
> 
> Veja quais são os assuntos do momento no Yahoo! + Buscados: Top 10 -
> Celebridades - Música - Esportes

=
Instru��es para entrar na lista, sair da lista e usar a lista em
http://www.mat.puc-rio.br/~obmlistas/obm-l.html
=


[obm-l] Re: [obm-l] FW: TERRA DOS MATEMÁTICOS!

2009-05-20 Por tôpico Paulo Santa Rita
Ola Jorge e demais colegas
desta lista ... OBM-L,

Voce gostou das "Investigacoes Aritmeticas" ? Fico feliz e obrigado
pelo elogio. Em verdade esta mensagem e a exposicao de estudos que eu
fiz quando ainda era muito jovem, crianca ainda. E apenas uma parte de
um estudo mais amplo. Na epoca o meu objetivo era encontrar as colunas
ocultas ( ou faces ocultas ) do traingulo de Pascal. Hoje eu sei com
fazer isso. Inclusive ja publiquei aqui algumas investigacoes neste
sentido.

Se voce verificar minhas primeiras mensagens para esta lista vai notar
que la eu digo que "havia descoberto coisas que nao estao nos livros".
Na verdade foram muitas coisas, pois sempre e naturalmente gostei de
pensar. Acho que e natural que todo jovem disciplinado e dedicado, que
realmente gosta de Matematica faca (re)descobertas de fatos que os
matematicos do passado ja fizeram. Por exemplo, o Gugu redescobriu um
tipo de solucao para equacoes do 3 grau ja descoberto pelo Euler e o
Nicolau ja disse aqui que redescobriu o algoritmo do calculo de raizes
quadradas. Acho que isso e natural e esperavel, nao signifcando nada
alem disso !

Seria possivel dizer o menor N tal que 1 + 1/2 + 1/3 + ... + 1/N > P,
para um P dado, sem usar
aproximacoes com a constante de Euler Macheroni ? Era isso que eu queria saber.

Nao entendi a passagem abaixo :

> Agora, quanto à série dos inversos dos primos...A Série Harmônica é um caso
> patológico de divergência. Se você somar os inversos dos naturais elevados a
> qualquer potência maior do que 1, a soma será convergente. Se for 1 ou menor
> do que 1 será divergente. Então, não existe um "menor" expoente r para o
> qual a soma dos inversos dos naturais elevados a r seja convergente. Como os
> primos são um subconjunto dos naturais, também não existe um "menor"
> expoente para o qual a soma dos inversos dos primos elevados a r seja
> convergente. Qualquer r maior do que 1 basta. O mesmo Euler provou, em 1736,
> que a soma dos inversos dos primos é divergente.

Eu me referi a soma das r-esimas potencias dos inversos dos primos.
Como a soma dos inversos dos primos e divergente entao, com certeza,
existe um r >1 tal que
a serie :
1 + (1/2) ^r + (1/3)^r + (1/5)^r + (1/7)^r + ... + (1/P) ^r +  ...
converge. Qual o menor "r" que atende esta condicao ? Euler mostrou
que r=1 nao serve, pois ele provou que a soma dos inversos dos primos
e divergente. Assim, r > 1. Qual o menor r ? Sera alguma das
constantes que conhecemos ? Sera um novo numero irracional importante
?

Bem falastes ! A serie harmonica !

Eu nao me canso de admira-la ! Ela e "altamente" sensivel. Voce
colocou um expoente "um pouquinho" maior que 1 em seus termos, ela
converge. Se mudar o sinal de + para - dos termos cujos denominadores
formam uma PA, ela converge. De alguma forma ele deve servir como uma
especie de "medida" ou termometro de convergencia, mas eu nao atinei
como fazer isso. Eu apreciaria muito se alguem pudesse falar algo a
respeito.

Um Abraco a Todos !
PSR,4200509090B





2009/5/19 Jorge Luis Rodrigues e Silva Luis :
>
>
> 
> From: jorgelrs1...@hotmail.com
> To: obm-l@mat.puc-rio.br
> Subject: TERRA DOS MATEMÁTICOS!
> Date: Tue, 19 May 2009 15:36:35 +
>
> Ok! Nehab, bom progresso para quem já foi denominada de "Terra dos
> Humoristas". Não é à toa que o autor da mais engenhosa distribuição das 3
> barras de chocolate entre quatro crianças é um Cearense, aluno do curso de
> licenciatura em matemática-UECE. Foi também o pioneiro a discordar da
> afirmação do colega Takiyama "1/x*x#x*1/x" na calculadora do
> feirante...Experimentem com seus pupilos a pueril situação: Entre as frações
> 1/5 e 1/3 temos 16 divisões iguais. Em qual das divisões se encontra a
> fração 1/4?
>
> Grande Paulo! Parabéns pela enquete "Investigações Aritméticas", pois me
> passou despercebida, na época. Uma verdadeira pérola.Campeão!
>
> Quanto à questão do menor N tal que 1+(1/2)+...+(1/N)>P, Euler demonstrou
> que a soma dos termos da Série Harmônica, para N tendendo ao infinito, é
> lnN+0,5772..., ou seja para atingir um inteiro P razoavelmente grande basta
> fazer lnN=P-0,5772... onde N é (2,718281828...) elevado a P-0,5772... Esse
> caminho permite obter uma ordem de grandeza bastante boa, mas para saber
> exatamente o menor N, teremos que trabalhar com muitas, mas muitas casas
> decimais.
>
> Agora, quanto à série dos inversos dos primos...A Série Harmônica é um caso
> patológico de divergência. Se você somar os inversos dos naturais elevados a
> qualquer potência maior do que 1, a soma será convergente. Se for 1 ou menor
> do que 1 será divergente. Então, não existe um "menor" expoente r para o
> qual a soma dos inversos dos naturais elevados a r seja convergente. Como os
> primos são um subconjunto dos naturais, também não existe um "menor"
> expoente para o qual a soma dos inversos dos primos elevados a r seja
> convergente. Qualquer r maior do que 1 basta. O mesmo Euler provou, em 1736,
> que a soma dos inversos do

Re: [obm-l] Triangulos e inteiros

2009-05-19 Por tôpico Paulo Santa Rita
Ola Nehab e demais
colegas desta lista ... OBM-L,

Poxa, eu nao sabia que a solucao de um problema tao simples poderia
servir de suporte a publicacao de um artigo em revista especializada
... Nao sei se felizmente ou infelizmente, mas, para mim, "artigo" e
aquilo que traz uma novidade ou contribuicao para a ciencia, o resto e
material de divulgacao ou/e pedagogia, coisas que eu nao conheco bem.
.

Bom, quanto ao seu desafio, eis aqui a explicacao :

Se o inraio de um triangulo e 2, sua area pode ser expressa por 2P,
onde P e o semiperimetro. Ora, isso e precisamente o perimetro do
triangulo. Logo, em tais triangulos, a area e igual ao perimetro.

Agora, amenidades a parte, aqui vai um primeiro problema relativo a
uma pesquisa com a qual me envolvi alguns anos atras. O objetio e
mostrar que toda sequencia da reta definida por mais de uma sentenca (
Ex : Xn=N/2 se N e par; Xn=2N+1 se N e impar ) tem um "caminho"
equivalente no plano. Muitas vezes fica mais facil estudar a sequencia
equivalente "do plano"

Vamos ao problema :

Acompanhe o seguinte passeio no plano : (0,0) -> (1,0) -> (1,1) ->
(0,1) ->(-1,-1) -> ((-1,0) -> (-1,-1) ->(0,-1) -> (1,-1) -> (2,-1) ->
(2,0) ->(2,1)->(2,2)->(1,2)->(0,2)->(-1,2)->(-2,-2)-> ...

Verifique que o caminho acima pode ser descrito assim : partindo de
(0,0) e caminhando sempre em sentido anti-horario de forma que jamais
passe por uma posicao ja ocupada anteriormente e mantendo-se, em cada
passo, o mais proximo possivel de (0,0).
.
Descubrar uma relacao de recorrencia (X_n,Y_n) que fornece as
coordenadas do "proximo passo" do caminho "em funcao do(s) passo(s)
anterior(es).

Um Abracao a Todos !
PSR,31905090E05









2009/5/19 Carlos Nehab :
> Oi, Paulo, Eduardo e colegas,
>
> Uma curiosidade:  o problema de determinar os triângulos de lados inteiros e
> cuja área e perímetro são representados pelo mesmo número, fixada a unidade,
> também possui 5 soluções, exatamente as soluções do problema proposto pelo
> Eduardo cuja solução você postou.
>
> Fica como desafio perceber porque os problemas são equivalentes...
>
> Este problema (o que eu mencionei) foi publicado na Revista do Professor de
> Matemática, da Sociedade Brasileira de Matemática e alguns de seus textos
> (da Revista) são utilizados pelo MEC em uma série de publicações para apoio
> aos professores de Matemática e disponível em seu portal.
>
> O Índice da Revista do Professor de Matemática você pode ver em
> http://www.rpm.org.br/cms/indice.pdf
>
> e a aconselho fortemente para os professores que atuam até o nível médio,
> pois possui centenas de idéias criativas.
>
> O texto (do MEC) a que me refiro você pode ver em
> http://portal.mec.gov.br/seb/arquivos/pdf/EnsMed/expensmat_icap3.pdf
>
> É uma iniciativa louvável do MEC, cujo site, inclusive, passou por uma
> reforma recentemente e está 1000 vezes melhor.
>
> Dê também uma olhada em (Olimpíada Brasileira de Matemática das Escolas
> Públicas)
> http://www.obmep.org.br/
>
> e veja se a gente não tem motivo para ficar orgulhoso de nosso país.
>
> Grande abraço,
> Nehab
>
>
> Paulo Santa Rita escreveu:
>
> Ola Wilner e demais colegas
> desta lista ... OBM-L,
>
> Toc, toc .. toc, toc ... Acorda Eduardo ! Sai dessa cripta, homem !
> Vem ajudar a levantar o nivel
> de discussao da nossa lista !
>
> Achei legal o problema que voce apresentou. Como ninguem quis fazer,
> eu bolei essa solucao ai embaixo, um tanto truculenta. Se nao errei
> nenhum calculo, sao apenas 5 os triangulos.
>
> Sejam  A, B e C os lados do triangulo, P o seu semiperimetro e R o
> raio do circulo inscrito. Sabemos que a area A deste triangulo pode
> ser expressa nos seguintes termos :
>
> A = RP = ( P(P-A)(P-B)(P-C) )^0.5
>
> Neste particular problema, R=2. Usando isto e trabalhando um pouco na
> expressao acima, chegaremos a :
>
> (A+B-C)(A+C-B)(B+C-A) = 16(A+B+C)(1)
>
> Logo, o produto da esquerda e par. Usando isso, por uma mera inspecao
> direta concluimos que  os lados A,B e C do nosso interesse se
> enquadram em duas categorias possiveis, vale dizer, ou são todos pares
> ou apenas dois deles são impares, não havendo uma terceira
> possibilidade.   Facamos  entao :
>
> B+C-A = X,   A+C-B=Y  e   A+B-C = Z
>
> Considerando que num triangulo qualquer lado e menor que a soma dos
> outros dois, fica facil ver o seguinte :
>
> 1) X, Y e Z são inteiros pares
> 2) X+Y+Z = A+B+C
> 3) 2A=Y+Z,  2B=X+Z e 2C=X+Y
>
> E agora a expressao (1) pode ser colocada assim :
>
> XYZ / (X+Y+Z) = 16.   E daqui, sai :
> (1/XY) + (1/XZ) + (1/YZ) = (1/16) (2)
>
> Seja S = (1/XY) + (1/XZ) + (1/YZ). Entao S = 1/16. E facil ver que as
> tres fracoes que constit

[obm-l] Inteiros e triangulos

2009-05-19 Por tôpico Paulo Santa Rita
Ola a Todos !

Inspirado na questao do Eduardo Wilner, surgiu-me o seguinte problema :

Determine todos os triangulos nao retangulos de lados inteiros cujos raios
dos circulos inscrito e circunscrito sao ambos inteiros.

Um Abraco a Todos !
PSR,31905090B09
=
Instru��es para entrar na lista, sair da lista e usar a lista em
http://www.mat.puc-rio.br/~obmlistas/obm-l.html
=


Re: [obm-l] Triangulos e inteiros

2009-05-19 Por tôpico Paulo Santa Rita
Ola Wilner e demais colegas
desta lista ... OBM-L,

Toc, toc .. toc, toc ... Acorda Eduardo ! Sai dessa cripta, homem !
Vem ajudar a levantar o nivel
de discussao da nossa lista !

Achei legal o problema que voce apresentou. Como ninguem quis fazer,
eu bolei essa solucao ai embaixo, um tanto truculenta. Se nao errei
nenhum calculo, sao apenas 5 os triangulos.

Sejam  A, B e C os lados do triangulo, P o seu semiperimetro e R o
raio do circulo inscrito. Sabemos que a area A deste triangulo pode
ser expressa nos seguintes termos :

A = RP = ( P(P-A)(P-B)(P-C) )^0.5

Neste particular problema, R=2. Usando isto e trabalhando um pouco na
expressao acima, chegaremos a :

(A+B-C)(A+C-B)(B+C-A) = 16(A+B+C)(1)

Logo, o produto da esquerda e par. Usando isso, por uma mera inspecao
direta concluimos que  os lados A,B e C do nosso interesse se
enquadram em duas categorias possiveis, vale dizer, ou são todos pares
ou apenas dois deles são impares, não havendo uma terceira
possibilidade.   Facamos  entao :

B+C-A = X,   A+C-B=Y  e   A+B-C = Z

Considerando que num triangulo qualquer lado e menor que a soma dos
outros dois, fica facil ver o seguinte :

1) X, Y e Z são inteiros pares
2) X+Y+Z = A+B+C
3) 2A=Y+Z,  2B=X+Z e 2C=X+Y

E agora a expressao (1) pode ser colocada assim :

XYZ / (X+Y+Z) = 16.   E daqui, sai :
(1/XY) + (1/XZ) + (1/YZ) = (1/16) (2)

Seja S = (1/XY) + (1/XZ) + (1/YZ). Entao S = 1/16. E facil ver que as
tres fracoes que constituem S nao podem ser concomitantemente menores
que 1/48, sob pena de S ser menor que 1/16; igualmente, nao podem ser
simultaneamente maiores que 1/48, sob pena de S ser maior que 1/16.
Logo :

3) Ao menos uma das fracoes que constituem S deve ser maior ou igual a 1/48.
4) Ao menos uma das fracoes que constituem S deve ser menor ou
igual a 1/48.

Seja portanto : XY =< 48 e  XZ  >= 48.

Com as restricoes acima ja e possivel identificar os triangulos que
estamos procurando. Para ver como fazer isso, note que :

(1/XY) + (1/XZ) + (1/YZ) = (1/16)   =>  16Z+16Y+16X=XYZ  =>
16Y+16X = (XY - 16)Z   =>   Z = (16Y + 16X) / (XY - 16)

Mas Z >= 48/X. Logo :
(16Y + 16X) / (XY - 16) >= 48/X=>  16/X <  Y  =< (24/X)+(X/2)
CASO  X=2  ( Y =< 24   e   Z >= 24 )

16/2 < Y =< (24/2)+(2/2)  =>  8 < Y =< 13 => Y=10 ou Y=12

Y = 10 :
Z=16(  (X+Y) / (XY-16) ) = 16(12/4)  => Z = 48
A=(10+48)/2=29,  B=(2+48)/2=25  e  C=(2+10)/2=6
Triangulo1 : (A,B,C)=(29,25,6)
Valido

Y=12:
Z=16(  (X+Y) / (XY-16) ) = 16(14/8)  => Z = 28
A=(12+28)/2=20,  B=(2+28)/2=15  e  C=(2+12)/2=7
Triangulo2 : (A,B,C)=(20,15,7)
Valido

***

CASO  X=4 ( Y =< 12  e  Z >= 12 )

16/4 < Y =< (24/4)+(4/2)  =>  4 < Y =< 8 => Y= 6 ou Y=8

Y = 6 :
Z=16(  (X+Y) / (XY-16) ) = 16(10/8)  => Z = 20
A=(6+20)/2=13,  B=(4+20)/2=12  e  C=(4+6)/2=5
Triangulo3 : (A,B,C)=(13,12,5)
Valido

Y=8:
Z=16(  (X+Y) / (XY-16) ) = 16(12/16)  => Z = 12
A=(8+12)/2=10,  B=(4+12)/2=8  e  C=(4+8)/2=6
Triangulo : (A,B,C)=(10,8,6)
Valido

***

CASO X=6 ( Y =< 8  e  Z >= 8  )

16/6 < Y =< (24/6)+(6/2)  =>  8/3 < Y =< 7 => Y= 4 ou Y=6

Y = 4 :
Z=16(  (X+Y) / (XY-16) ) = 16(10/8)  => Z = 20
A=(4+20)/2=12,  B=(6+20)/2=13  e  C=(4+6)/2=5
Triangulo : (A,B,C)=(12,13,5)
Invalido : ja descoberto

Y=6:
Z=16(  (X+Y) / (XY-16) ) = 16(12/20)  => Z = 9.6
Invalido : Z nao e inteiro par

***

CASO X=8 ( Y =< 6  e  Z >= 6  )

16/8 < Y =< (24/8)+(8/2)  =>  2 < Y =< 7 => Y= 4 ou Y=6

Y = 4 :
Z=16(  (X+Y) / (XY-16) ) = 16(12/16)  => Z = 12
A=(4+12)/2=8,  B=(8+12)/2=10  e  C=(8+4)/2=6
Triangulo4 : (A,B,C)=(8,10,6)
Invalido : ja descoberto

Y=6:
Z=16(  (X+Y) / (XY-16) ) = 16(14/32)  => Z = 7
Invalido : Z nao e inteiro par

***
CASO X=10 ( Y =< 4.8  e  Z >= 4.8  )

16/10 < Y =< (24/10)+(10/2)  =>  1.6 < Y =< 7.4 => Y= 2 ou Y=4

Y = 2
Z=16(  (X+Y) / (XY-16) ) = 16(12/4)  => Z = 48
A=(2+48)/2=25,  B=(10+48)/2=29  e  C=(10+2)/2=6
Triangulo : (A,B,C)=(25,29,6)
Invalido : ja descoberto

Y=4:
Z=16(  (X+Y) / (XY-16) ) = 16(14/24)  => Z = (28/3)
Invalido : Z nao e inteiro par

***

CASO X=12 ( Y =< 4  e  Z >= 4  )

16/12 < Y =< (24/12)+(12/2)  =>  (4/3) < Y =< 8 => Y= 2 ou Y=4

Y = 2
Z=16(  (X+Y) / (XY-16) ) = 16(14/8)  => Z = 28
A=(2+28)/2=15,  B=(12+28)/2=20  e  C=(12+2)/2=7
Triangulo : (A,B,C)=(15,20,7)
Invalido : ja descoberto

Y=4:
Z=16(  (X+Y) / (XY-16) ) = 16(14/24)  => Z = 8
A=(4+8)/2=6,  B=(12+8)/2=10  e  C=(12+4)/2=8
Triangulo : (A,B,C)=(6,10,8)
Invalido : ja descoberto

***
CASO X=14 ( Y =< 3.4...  e  Z >= 3.4...  )

A partir daqui, devido a restricao acima,  basta analisarmos os casos em que Y=2

Y = 2
Z=16(  (X+Y) / (XY-16) ) = 16(16/12)  => Z  nao e inteiro  => o
triangulo e invalido

***
CASO X=16, Y=2

Z=16(  (X+Y) / (XY-16) ) = 16(18/16)  => Z = 18
A=(2+18)/2=10,  B=(16+18)/2=17  e  C=(16+2)/2=9
Triangulo5 : (A,B,C)=(10,9,17)
Valido

***
CASO X=18, Y=2

Z=16(  (X+Y) / (XY-16) ) = 16(20/20)  => Z = 16
A=(2+16)/2=9,  B=(16+18)/2=17  e  C=(2+18)/2=10
Triangulo : (A,B,C)=(9,17,10)
Invalido : ja descoberto

***
CASO X=20, Y=2

Z=16(  (X+Y) / (XY-16) ) = 16(22/24)  => Z = 44

Re: [obm-l] A/C Paulo Santa Rita

2009-05-18 Por tôpico Paulo Santa Rita
Ola Joao e demais colegas
desta lista ... OBM-L,

Agora lembrei. Infelizmente nao tenho mais as figuras ali citadas.
Talvez lhe seja util falar um pouco da motivacao para este
desenvolvimento.

Quando eu estudei este assunto ( PA de ordem superior ) pela primeira
vez me deparei com o seguinte teorema :

"A soma dos termos de uma progressao aritmetica de ordem P e um
polinomio de ordem P+1"

Este teorema e criticavel de diversas maneiras. Vou citar aqui as duas
principais :

1) Pressupoe um conceito de ordem de uma progressao aritmetica que nao
esta explicitamente enunciado e explicado
2) Voce precisa resolver um sistema de equacoes lineares para
encontrar o polinomio que representa a soma dos termos da PA de ordem
mais alta.

Exemplo :

Suponha que voce deseje encontrar o polinomio que representa a soma da
progressao aritmetica de 2 ordem : 2^2, 5^2, 8^2, 11^2, ... ,
(3N-1)^2, ...

Usando esse teorema voce precisa fazer assim :

1) Supor um polinomio soma S(N) = AN^3 + BN^2 + CN + D. fazer :

2) Fazer :

S(1) = A + B + C + D = 4 = 2^2
S(2) = 8A + 4B + 2C + D = 29 = 2^2 + 5^2
S(3) = 27A + 9B + 3C + D = 110 = 2^2 + 5^2 + 8^2
S(4) = 64A + 16B + 4C + D = 131 = 2^2 + 5^2 + 8^2 + 11^2

3) Resolver o enorme sistema acima.
4) Montar o polinomio soma com coeficientes A, B, C e D


Imagina para progressoes de ordem mais alta, digamos, de 7 ordem, etc.
Um absurdo !

Portanto, era natural que eu procurasse uma forma de botar ordem nesta
bagunca e desenvolver uma forma inteligente de fazer as coisas. Esta
foi a minha principal motivacao.

Com as tecnicas que eu desenvolvi e que voce pode ver na mensagem cujo
link voce postou, voce pode encontrar o polinomio soma de uma PA de
ordem qualquer em menos de 1 minuto. No caso particular que eu citei,
temos :

S(N) = A*Binom(N,1) + B*Binom(N,2) + C*Binom(N,3)
onde A= A1=2^2 =4 , B=A2- A1= 5^2-2^2=21 e C=A3-2*A2 + A1 = 8^2 - 2*5^2 + 2^2=18
ou seja :

S(N) = 4*Binom(N,1) + 21*Binom(N,2) + 18*Binom(N,3)

As demonstracoes estao lá.

Note que isto e um pequeno aspecto de algo mais amplo. Por exemplo,
voce pode estender o conceito de progressao arimetica para incorporar
progressoes de ordem negativa e fracionaria. A sequencia 1, (1/2)^3,
(1/3)^3 , (1/4)^3 e um exemplo de uma PA de ordem -3. Neste caso, nao
nos interessa a soma de uma quantidade finita de termos, mas o valor
para onde a serie converge.

A importancia de se estudar PA's de ordem superior e poder tratar de
TRIANGULOS ARITMETICOS, que sao na verdade familias de PA's. Por
exemplo, considere a sequencia :

O triangulo de Pascal e apenas um caso particular dentro do universos
dos trainguloas aritmeticos. Enfim, este estudo tem aplicacoes simles,
como esta que voce esta abordando, mas tem tambem implicacoes nao tao
simples, que nao e cabivel espor aqui.

Um Abracao
PSR, 21805090C01



2009/5/18 João Luís :
> O link é
>
> http://www.mat.puc-rio.br/~obmlistas/obm-l.1999a/msg00191.html
>
> Na verdade, você não anexou as imagens, e sim colou na mensagem... mas elas
> não aparecem aqui!
>
> ----- Original Message - From: "Paulo Santa Rita"
> 
> To: 
> Sent: Monday, May 18, 2009 10:22 AM
> Subject: Re: [obm-l] A/C Paulo Santa Rita
>
>
> Ola joao e demais colegas
> desta lista ... OBM-L,
>
> OK ! Fico aguardando voce publicar o link.
>
> Um Abraco
> PSR,21005090A16
>
> 2009/5/18 João Luís :
>>
>> Você faz uma exposição bastante interessante sobre PAs de ordem 1, 2,
>> 3,...
>> introduzindo números binomiais (n,p), que vc denotou, na época, [N/P]
>>
>> Se ainda não deu pra lembrar, procuro a mensagem novamente e a envio
>> completa pra você.
>>
>> Um abraço e muito obrigadoe pela sua atenção.
>>
>> João Luís.
>>
>> - Original Message - From: "Paulo Santa Rita"
>> 
>> To: 
>> Sent: Monday, May 18, 2009 8:43 AM
>> Subject: Re: [obm-l] A/C Paulo Santa Rita
>>
>>
>> Ola Joao e demais colegas
>> desta lista ... OBM-L,
>>
>> Caro Joao, nao me lembro do que trata esta mensagem. Portanto, nao sei
>> dizer se ainda tenho a suposta figura que porventura tenha anexado ...
>> De forma mais especifica, qual o tema da mensagem ? E um problema
>> particular de PA ou trata-se de uma possivel ampliacao deste conceito
>> ?
>>
>> Fico feliz em saber que alguma coisa que publiquei aqui pode vir a ser
>> util para enriquecer a sua prelecao.
>>
>> Para que esta mensagem mantenha a tradicao da nossa lista, vou postar
>> aqui um problema - descobri isso, agora - antigo, postado pelo Eduardo
>> Wilner, que, pelo que vi, nao despertou o interesse de qualquer dos
>> membros desta nossa lista. E bonitinho :
>>
>> PROBLEMA : Determine todos os triangulos de lados inteiros nos quais o
>> raio do circulo inscrito vale

Re: [obm-l] A/C Paulo Santa Rita

2009-05-18 Por tôpico Paulo Santa Rita
Ola joao e demais colegas
desta lista ... OBM-L,

OK ! Fico aguardando voce publicar o link.

Um Abraco
PSR,21005090A16

2009/5/18 João Luís :
>
> Você faz uma exposição bastante interessante sobre PAs de ordem 1, 2, 3,...
> introduzindo números binomiais (n,p), que vc denotou, na época, [N/P]
>
> Se ainda não deu pra lembrar, procuro a mensagem novamente e a envio
> completa pra você.
>
> Um abraço e muito obrigadoe pela sua atenção.
>
> João Luís.
>
> - Original Message - From: "Paulo Santa Rita"
> 
> To: 
> Sent: Monday, May 18, 2009 8:43 AM
> Subject: Re: [obm-l] A/C Paulo Santa Rita
>
>
> Ola Joao e demais colegas
> desta lista ... OBM-L,
>
> Caro Joao, nao me lembro do que trata esta mensagem. Portanto, nao sei
> dizer se ainda tenho a suposta figura que porventura tenha anexado ...
> De forma mais especifica, qual o tema da mensagem ? E um problema
> particular de PA ou trata-se de uma possivel ampliacao deste conceito
> ?
>
> Fico feliz em saber que alguma coisa que publiquei aqui pode vir a ser
> util para enriquecer a sua prelecao.
>
> Para que esta mensagem mantenha a tradicao da nossa lista, vou postar
> aqui um problema - descobri isso, agora - antigo, postado pelo Eduardo
> Wilner, que, pelo que vi, nao despertou o interesse de qualquer dos
> membros desta nossa lista. E bonitinho :
>
> PROBLEMA : Determine todos os triangulos de lados inteiros nos quais o
> raio do circulo inscrito vale 2.
>
> Um Abraco a todos !
> PSR,2180509082A
>
>> 2009/5/17 João Luís :
>> Esta mensagem é sobre outra mensagem, que o PSR enviou pra esta lista, há
>> muito tempo atrás...
>>
>> Paulo,
>>
>> Preparando uma aula sobre progressões aritméticas, recorri aos arquivos da
>> lista para pesquisar o material que já foi enviado sobre esse assunto, com
>> o
>> intuito de pesquisar novos fatos, novas abordagens, e enriquecer minha
>> aula.
>>
>> Cheguei a uma mensagem sua de 15 de outubro de 1999, com o assunto
>> "Investigações Aritméticas", do qual gostei muito. Em certa altura do seu
>> texto, você cola duas figuras no corpo da mensagem. Você ainda possui
>> essas
>> figuras? Poderia enviá-las para mim?
>>
>> Se puder me ajudar, agradeço muito.
>>
>> Um abraço,
>>
>> João Luís.
>
> =
> Instrues para entrar na lista, sair da lista e usar a lista em
> http://www.mat.puc-rio.br/~obmlistas/obm-l.html
> =
>
> =
> Instru�ões para entrar na lista, sair da lista e usar a lista em
> http://www.mat.puc-rio.br/~obmlistas/obm-l.html
> =
>

=
Instru��es para entrar na lista, sair da lista e usar a lista em
http://www.mat.puc-rio.br/~obmlistas/obm-l.html
=


Re: [obm-l] A/C Paulo Santa Rita

2009-05-18 Por tôpico Paulo Santa Rita
Ola Joao e demais colegas
desta lista ... OBM-L,

Caro Joao, nao me lembro do que trata esta mensagem. Portanto, nao sei
dizer se ainda tenho a suposta figura que porventura tenha anexado ...
De forma mais especifica, qual o tema da mensagem ? E um problema
particular de PA ou trata-se de uma possivel ampliacao deste conceito
?

Fico feliz em saber que alguma coisa que publiquei aqui pode vir a ser
util para enriquecer a sua prelecao.

Para que esta mensagem mantenha a tradicao da nossa lista, vou postar
aqui um problema - descobri isso, agora - antigo, postado pelo Eduardo
Wilner, que, pelo que vi, nao despertou o interesse de qualquer dos
membros desta nossa lista. E bonitinho :

PROBLEMA : Determine todos os triangulos de lados inteiros nos quais o
raio do circulo inscrito vale 2.

Um Abraco a todos !
PSR,2180509082A

>2009/5/17 João Luís :
> Esta mensagem é sobre outra mensagem, que o PSR enviou pra esta lista, há
> muito tempo atrás...
>
> Paulo,
>
> Preparando uma aula sobre progressões aritméticas, recorri aos arquivos da
> lista para pesquisar o material que já foi enviado sobre esse assunto, com o
> intuito de pesquisar novos fatos, novas abordagens, e enriquecer minha aula.
>
> Cheguei a uma mensagem sua de 15 de outubro de 1999, com o assunto
> "Investigações Aritméticas", do qual gostei muito. Em certa altura do seu
> texto, você cola duas figuras no corpo da mensagem. Você ainda possui essas
> figuras? Poderia enviá-las para mim?
>
> Se puder me ajudar, agradeço muito.
>
> Um abraço,
>
> João Luís.

=
Instru��es para entrar na lista, sair da lista e usar a lista em
http://www.mat.puc-rio.br/~obmlistas/obm-l.html
=


Re: [obm-l] Problemas de Sangaku

2009-05-15 Por tôpico Paulo Santa Rita
Ola Denisson, Nehab e demais
colegas desta lista ... OBM-L,

Pessoal, penso que e natural que esta convivencia informal que
cultivamos aqui inevitavelmente nos leva a desenvolver certa simpatia
por algumas pessoas... Estou seriamente preocupado com o nosso amigo
Nehab, pois, pelo que estou sabendo das ultimas mensagens, ele esta em
seu escritorio, atracado com um violento Sangaku. Alguem tem noticias
dele ?

Bom, amenidades a parte, vamos tentar domar este violento sangaku. Vou
usar a figura sugerida pelo Denisson, na mensagem abaixo.

Seja AP=X. Sejam tambem :

1)  T1, T2 e T3 os pontos de tangencia do circulo ( raio R2 ) inscrito
no traingulo ABP respectivamente nos lados AB, AP e PB
2)  Q1, Q2 e Q3 os pontos de tangencia do circulo ( raio R1) inscrito
no triangulo PCD respectivamente nos lados CD, DP e PC.

R3 e o raio do circulo inscrito no triangulo BPC

E facil ver que a area do triangulo BPC e sempre 1/2, independente do
valor de AP=X que escolhermos. Sabemos que esta area pode ser expressa
assim :

1/2 = R3 *( semi-perimetro do traingulo BPC )

E quem e o semi-perimetro do traingulo BPC ? Vejamos :

PB = PT3  + T3B = X - R2 + 1 - R2 = 1 + X -2*R2
PC = PQ3 +  Q3C = (1-X) - R1 + 1 - R1 = 2 - X - 2*R1
BC = 1

Logo : semi-perimetro = (4 -2*R1 - 2*R2)/2 = 2 - R1 - R2. E daqui segue :

R3*(2 - R1 - R2) = 1/2   => 2 - ( 1/2*R3 ) = R1 + R2

Ou seja, a relacao acima entre os 3 raios independe do valor AP=X que
escolhermos.

Agora vamos a pergunta dois :

Existe  X  tal que R1:R2:R3 = 1:2:3 ?

Se existir um tal X entao R1 + R2 + R3 / 6 = R3/3. Substituindo R1 +
R2 por 2 - ( 1/2*R3), ficara :

2 - (1/2*R3) + R3 = 2*R3   => R3 = (2 - raiz_quad(2) ) / 2
( a outra raiz nao serve por ser maior que 1 )

E facil ver que este valor so ocorre quando AP=X=0, reduzindo-se um
dos circulos a um ponto. Assim, nao existe X > 0 que torne  R1:R2:R3 =
1:2:3.

Um Abraco a todos !
PSR, 61505090B21





> 2009/5/14 Carlos Nehab 
>
> Oi, Santa Rita,
>
> O problema do problema é efetivamente evitar o sistema que você mencionou,
> que é do terceiro grau...
> Aliás, os problemas de geometria ditos "quadráticos" são quase sempre
> triviais.
> Os bons problemas, em 90% dos casos, são quase sempre "cubicos".
>
> To atracado com o problema, tentando  uma soluao geométrica.  Guenta  a
> mão Denilson... :-)
>
> Abraços
> Nehab


> 2009/5/14 Denisson :
> Cerca de seis anos atrás eu tive um professor que era fascinado pelos
> problemas de sangaku. Alguns eu conseguia fazer, mas confesso que a
> maioria
> ficava à espera da resposta na próxima semana. Vou mostrar um em
> particular
> que eu não consegui fazer:
>
> Considere um quadrado ABCD de lado 1 e tome um ponto P qualquer entre A e
> D.
> Trace a reta BP e a reta PC. Inscreva um círculo em cada um dos 3
> triângulos
> ABP (de raio R2), BPC (de raio R3) e PCD (de raio R1). Encontre o raio dos
> 3
> círculos em função da medida AP. Em particular, existe alguma posição do
> ponto P sobre AD tal que R1:R2:R3 tome o valor 1:2:3?
>
> Se não surgirem respostas posto a solução daqui a alguns dias.
>
> --
> Denisson

=
Instru��es para entrar na lista, sair da lista e usar a lista em
http://www.mat.puc-rio.br/~obmlistas/obm-l.html
=


[obm-l] Re: [obm-l] sistema de equações modulares

2009-05-14 Por tôpico Paulo Santa Rita
Ola Vanderlei e demais
colegas desta lista ... OBM-L,

Nao sei se vou conseguir atender as suas expectativas ...  Pelo que
entendi, voce quer reduzir o espaco das solucoes. Supondo que voce
esta pensando em "x" e "y" como numeros reais, as conhecidas
propriedades entre modulos

| A - B | = | B - A )
| A | + | B | >= |A + B|

nos permitem, a principio, escrever :

|x+y|+|1-x| = 6 >= |x+y+1-x|  implica  |y+1| =< 6  implica -7 =< y =< 5
|x+y+1|+|1-y| = 4 >= |x+y+1+1-y|  implica  |x+2| =< 4 implica -6 =< x =< 2

ou seja, o espaco das solucoes restringe-se ao quadradinho definido
pelas duas inequacoes simultaneas acima. Isso, em si,  ja e uma
restricao importante. Resta portanto apenas descobrir quais pares
(x,y) interiores a este quadradinho nos interessam. Para ver como e
possivel  discrimina-los, considere que :

|x+y| + |1-x| = |x+y| + |x-1| = 6  >= |2x+y-1|  => -6 =< 2x+y-1 =< 6
=> -2x-5 =< y =< -2x + 7
|x+y+1| + |1-y| = |x+y+1| + |y-1| = 4 >= |x+2y|  => -4 =< x+2y =< 4 =>
-x/2 - 2 =< y =< -x/2 + 2

A interseccao entre essas 4 inequacoes simultaneas e a solucao.

Um Abraco a todos !
PSR, 51405091430

2009/5/14 Vandelei Nemitz :
> Bom dia pessoal..será que alguém consegue resolver sem analisar todos os
> casos?
>
> |x + y| + |1 - x| = 6 e |x + y + 1| + |1 - y| = 4
>
> Eu fiz analisando todas as possibilidades de sinais, mas é muito trabalhoso.
>
> obrigado!
>
> Vanderlei

=
Instru��es para entrar na lista, sair da lista e usar a lista em
http://www.mat.puc-rio.br/~obmlistas/obm-l.html
=


Re: [obm-l] Problemas de Sangaku

2009-05-14 Por tôpico Paulo Santa Rita
Ola Denisson e demais
colegas desta lista ... OBM-L,

Eu acho que voce queria dizer : "trace a reta BP e a reta PC", certo ?
Se for assim, a sua questao e  simples, pois, fazendo  AP=X,  e facil
ver que o triangulo ABP tem catetos 1 e X e o triangulo PCD tem
catetos 1 e 1-X. Isto implica que suas hipotenusas estao determinadas
pelo teorema de pitagoras. Ora, estas hipotenusas sao precisamente
dois dos lados do triangulo BPC, sendo o terceiro lado igual a 1.
Conhecemos portanto todos os lados dos tres triangulos  ( em funcao de
AP=X.).  Lancando mao da expressao :

R = ( (p(p-a)(p-b)(p-c) )^0.5 / p

onde "p" e o semi-perimetro do triangulo de lados "a", "b" e "c",
obtemos os raios Ri dos circulos inscritos nos triangulos. Deste como
conhecemos R1, R2 e R3 em funcao de X. Com as expressos dos raios (
funcao de X ) que obtivemos acima,  fazemos :

R1 = R2/2
R2/2 = R3/3

Caso o sistema acima admita uma ou varias solucoes, trata-se da
resposta a sua segunda pergunta. E logico que pode haver um caminho
mais elegante, partindo, por exemplo, de uma construcao geometrica
auxiliar. Fica a sugestao... Confesso, entretanto, que o que me levou
a lhe responder, apresentando uma solucao "forca bruta" como esta,
foi sobretudo outro motivo ...

O que sao estes "Problemas de  Sangaku" ? Nunca ouvi alguem falar sobre isso.

Um abraco a todos !
PSR, 51405091108

> 2009/5/14 Denisson :
> Cerca de seis anos atrás eu tive um professor que era fascinado pelos
> problemas de sangaku. Alguns eu conseguia fazer, mas confesso que a maioria
> ficava à espera da resposta na próxima semana. Vou mostrar um em particular
> que eu não consegui fazer:
>
> Considere um quadrado ABCD de lado 1 e tome um ponto P qualquer entre A e D.
> Trace a reta BP e a reta BC. Inscreva um círculo em cada um dos 3 triângulos
> ABP (de raio R2), BPC (de raio R3) e PCD (de raio R1). Encontre o raio dos 3
> círculos em função da medida AP. Em particular, existe alguma posição do
> ponto P sobre AD tal que R1:R2:R3 tome o valor 1:2:3?
>
> Se não surgirem respostas posto a solução daqui a alguns dias.
>
> --
> Denisson

=
Instru��es para entrar na lista, sair da lista e usar a lista em
http://www.mat.puc-rio.br/~obmlistas/obm-l.html
=


Re: [obm-l] formulas para numeros primos

2009-05-13 Por tôpico Paulo Santa Rita
Ola Bouskela,

Em off eu ja lhe respondi. Se quiser publicar aqui a parte academica
da resposta, pode publicar.

PROBLEMA : Sejam dados A_1, A_2, ..., A_n inteiros positivos com A_i >
A_k sempre que i > k. Considere o produto P de todas as diferencas da
forma ( A_i  -  A_k ), onde i > k. Fixado N > 3, encontre o D tal que
D divide P, independe dos A_i dados.

Um Abraco a Todos
PSR,41305090D11

2009/5/12 Albert Bouskela :
> Olá Santa Rita e demais colegas!
>
> Olá a quem, como eu, gosta de Filosofia (veja o PS) e tem a sorte - a mesma 
> sorte que tenho - de ser (muito bem) casado com uma escritora!
>
> Bem, Santa Rita, vou discordar (só um pouquinho) das suas críticas às minhas 
> críticas. Lá vai:
>
> Sou um crente fervoroso do seguinte teorema (a ser demonstrado): "Toda e 
> qualquer descoberta feita na Matemática teve, tem ou terá alguma aplicação 
> prática".
>
> Assim, defendo que um livro sobre Primos não pode prescindir de falar sobre 
> Criptografia Assimétrica, porque esta é, atualmente, a principal aplicação 
> dos Primos. Afinal, não podemos ensinar um estudante a somar, sem explicar 
> que, tendo ele 4 laranjas, ao comprar mais 2, passará a ter 6 laranjas. Desde 
> os primórdios, foi esse o leitmotiv do desenvolvimento da Matemática (e de 
> TODAS as Ciências!).
>
> Veja que o Lobachevsky, o Bolyai, o Riemann... quando inventaram as 
> Geometrias Não-euclidianas não sabiam qual seria a aplicação de um troço tão 
> inusitado. Por isso, seus trabalhos restringiram-se à Matemática Pura e 
> tiveram pequena repercussão. Hoje, entretanto, estas Geometrias estão na 
> ordem do dia, devido à sua aplicação na Teoria da Relatividade. É claro que 
> um trabalho atual sobre elas teria, necessariamente, que abordar como é a sua 
> formulação no âmbito da Física Relativista de Einstein (até mesmo pelo seu 
> contexto histórico).
>
> O livro do Eric não está apresentando um conceito novo da Teoria do Números. 
> Os Primos nasceram junto com os primeiros passos da Matemática. Em sendo 
> assim, para que servem?
>
> Albert Bouskela
> bousk...@msn.com
>
>
>> -Original Message-
>> From: owner-ob...@mat.puc-rio.br [mailto:owner-ob...@mat.puc-rio.br]
>> On Behalf Of Paulo Santa Rita
>> Sent: Tuesday, May 12, 2009 2:29 PM
>> To: obm-l@mat.puc-rio.br
>> Subject: Re: [obm-l] formulas para numeros primos
>>
>>  Oi Eric e demais colegas
>>  desta lista ... OBM-L,
>>
>> ( ESTOU REENVIANDO A MENSAGEM PORQUE ELA FOI INCOMPLETA )
>>
>>  Li com mais atencao o seu trabalho. Li tambem - agora - a(s)
>>  critica(s) que voce ja recebeu. Reitero que gostei dele.
>>
>>  E uma exposicao elementar e introdutoria sobre um tema especifico da
>>  Teoria dos numeros, nomeadamente a exposicao de algumas funcoes e
>>  tecnicas que geram numeros primos, algo que nao se ve com frequencia
>>  nas livrarias, mesmo nas especializadas. Neste sentido, o seu livro
>>  PREENCHE UMA LACUNA, so por isso, e importante e valioso.
>>
>> Eu tinha uma conviccao muito forte de que nao encontraria novidades,
>> no que acertei. Afinal, nao e uma tese. E um livro elementar e
>> introdutorio, nao podendo portanto ser apreciado por este angulo. Isso
>> nao desmerece em nenhum sentido o seu esforco.
>>
>> Voce pede apreciacoes. Vou dizer o que, EM MINHA OPINIAO, pode ser
>> melhorado. Basicamente duas coisas.
>>
>> 1) O livro repete resultados ja apresentados em capitulos anteriores.
>> Alguns leitores se aborrecem com isso. Eu suspeito que voce fez varios
>> capitulos separadamente e depois uniu-os neste livro. Assim, EM MINHA
>> OPINIAO, ele ficara melhor se voce retirar estas repeticoes.
>>
>> 2) Como o livro trata de formulas que geram numeros primos, voce nao
>> precisaria tratar diretamente de teoremas elementares e bem-conhecidos
>> da teoria dos numeros, tais como o teorema de Wilson e correlatos.
>>
>> Basicamente e isso. Retirando isso, EM MINHA OPINIAO, o livro vai
>> preencher uma lacuna, ser de leitura agradavel e bastante claro (
>> mesmo porque e um livro elementar ). Prossiga, aperfeicoando-o.
>>
>> Agora, se voce me permite,  uma palavra sobre as criticas que voce
>> recebeu ou venha a receber :
>>
>> 1) Nenhuma teoria matematica pode ser justificada ou validada por uma
>> eventual aplicacao
>> pratica que ela teve, tenha ou venha a ter. Assim, a exposicao ou
>> pesquisa de um assunto matematico nao pode ser obstado em qualquer
>> sentido por qualquer argumento desta natureza.
>>
>> 2) Numa sociedade livre e democratica, formada por pessoas
>> inteligentes, qualque forma de padronizacao das

Re: [obm-l] formulas para numeros primos

2009-05-12 Por tôpico Paulo Santa Rita
 Oi Eric e demais colegas
 desta lista ... OBM-L,

( ESTOU REENVIANDO A MENSAGEM PORQUE ELA FOI INCOMPLETA )

 Li com mais atencao o seu trabalho. Li tambem - agora - a(s)
 critica(s) que voce ja recebeu. Reitero que gostei dele.

 E uma exposicao elementar e introdutoria sobre um tema especifico da
 Teoria dos numeros, nomeadamente a exposicao de algumas funcoes e
 tecnicas que geram numeros primos, algo que nao se ve com frequencia
 nas livrarias, mesmo nas especializadas. Neste sentido, o seu livro
 PREENCHE UMA LACUNA, so por isso, e importante e valioso.

Eu tinha uma conviccao muito forte de que nao encontraria novidades,
no que acertei. Afinal, nao e uma tese. E um livro elementar e
introdutorio, nao podendo portanto ser apreciado por este angulo. Isso
nao desmerece em nenhum sentido o seu esforco.

Voce pede apreciacoes. Vou dizer o que, EM MINHA OPINIAO, pode ser
melhorado. Basicamente duas coisas.

1) O livro repete resultados ja apresentados em capitulos anteriores.
Alguns leitores se aborrecem com isso. Eu suspeito que voce fez varios
capitulos separadamente e depois uniu-os neste livro. Assim, EM MINHA
OPINIAO, ele ficara melhor se voce retirar estas repeticoes.

2) Como o livro trata de formulas que geram numeros primos, voce nao
precisaria tratar diretamente de teoremas elementares e bem-conhecidos
da teoria dos numeros, tais como o teorema de Wilson e correlatos.

Basicamente e isso. Retirando isso, EM MINHA OPINIAO, o livro vai
preencher uma lacuna, ser de leitura agradavel e bastante claro (
mesmo porque e um livro elementar ). Prossiga, aperfeicoando-o.

Agora, se voce me permite,  uma palavra sobre as criticas que voce
recebeu ou venha a receber :

1) Nenhuma teoria matematica pode ser justificada ou validada por uma
eventual aplicacao
pratica que ela teve, tenha ou venha a ter. Assim, a exposicao ou
pesquisa de um assunto matematico nao pode ser obstado em qualquer
sentido por qualquer argumento desta natureza.

2) Numa sociedade livre e democratica, formada por pessoas
inteligentes, qualque forma de padronizacao das manifestacoes
intelectuais e uma forma de censura inadmissivel, cabendo
exclusivamente ao autor, em face do seu feeling editorial, escolher o
conteudo dos seus trabalhos.

3) Os antigos diziam que "quem esta na chuva e pra se molhar". Assim,
nao se importe muito com as criticas. Procure ver o que elas tem de
precedente e prossiga. Va em frente, Como dizia D'Alembert : " Avante,
que a fe vos vira depois ! "

E com os melhores
votos de paz profunda
para ti, sou

Paulo Santa Rita









2009/5/12 Paulo Santa Rita :
> Oi Eric e demais colegas
> desta lista ... OBM-L,
>
> Li com mais atencao o seu trabalho. Li tambem - agora - a(s)
> critica(s) que voce ja recebeu. Reitero que gostei dele.
>
> E uma exposicao elementar e introdutoria sobre um tema especifico da
> Teoria dos numeros, nomeadamente a exposicao de algumas funcoes e
> tecnicas que geram numeros primos, algo que nao se ve com frequencia
> nas livrarias, mesmo nas especializadas. Neste sentido, o seu livro
> preenche uma lacuna e, so por isso, e importante e valioso.
>
> Voce pede apreciacoes. Vou dizer o que, EM MINHA OPINIAO,
>
> 2009/5/11 Paulo Santa Rita :
>> Oi Eric e demais colegas
>> desta lista ... OBM-L,
>>
>> Li rapidamente o seu livro e gostei. Vou le-lo com mais vagar,
>> adiante. Por oportuno gostaria de saber se voce ja se ocupou com a
>> questao sobre a influencia dos inteiros gaussianos na distribuicao dos
>> primos.
>>
>> Nos sabemos que muitos numeros primos passam a ser compostos no caso
>> de considerarmos os inteiros gaussianos. Por exemplo : 5=(2+i)(2-i) =>
>> 5 nao e primo. Chamando de numero primo APENAS AQUELES INTEIROS
>> POSITIVOS QUE TAMBEM SAO
>> PRIMOS NO DOMINIO DOS INTEIROS GAUSSIANOS, como fica o teorema da
>> distribuicao dos numeros primos neste contexto ?
>>
>> Teorema da distribuicao dos numeros primos :
>>
>> Seja pi(n) o numero de numeros primos p tais que 2 =< p =< n. Entao :
>>
>> LIM (  pi(N) / ( N/Log(N) )  ) = 1
>>
>> Na expressao acima, Log(N) e o logaritmo natural de N ( base e=2,7... )
>>
>> Um abraco
>> PSR, 21105090B05
>>
>> Aproveito a oportunidade para lhe solicitar um esclarecimento, caso
>> voce ja tenha se ocupado
>> do que vou relatar :
>>
>> 2009/5/11 Eric Campos Bastos Guedes :
>>>
>>> Saudacoes aos colegas da lista
>>>
>>> Acabo de disponibilizar na internet meu trabalho
>>> sobre formulas para numeros primos. O endereco eh:
>>>
>>> http://www.docstoc.com/docs/5851750/F%C3%B3rmulas-para-N%C3%BAmeros-Primos
>>>
>>> gostaria da opiniao dos membros da lista a respeito.
>>>
>>>
>>> 

[obm-l] Tabua Magica

2009-05-11 Por tôpico Paulo Santa Rita
Ola a Todos !

Ha alguns anos atras um hungaro chamado Erno Rubik inventou um
quebra-cabeca tridimensional agora muto conhecido, batizado-o de "cubo
magico". Este quebra-cabeca e tambem conhecido como "cubo de rubik" e
todos devem estar sabendo ao que me refiro.

Seguindo este exemplo, eu inventei um quebra-cabeca que estou
batizando de "tabua magina", nome este motivado pela descricao do
desafio que passo a apresentar :

IMAGINE uma tabua de forma retangular, melhor imaginada se supormos
que a altura e maior que a largura. Vamos IMAGINAR 9 linhas
horizontais. Na primeira linha, a linha mais acima, estao suspensos 49
fios, igualmente espacados e cada um deles de comprimento igual a
altura da tabua. Estes fios sao numerados da esquerda para a direita
de 1 ate 49. As demais linhas imaginarias da tabua sao numeradas de
cima para baixo de 1 ate 8. Ao longo de cada uma destas 8 linhas
existem 7 furos, igualmente espacos, por onde podem passar 7 fios.

O desafio consiste em passar os fios pelos buracos, atendendo as
seguintes regras :

1) As linhas sao preenchidas ( os 49 fios passaram por seus 7 buracos
) na ordem de sua numeracao, vale dizer, primeiro preenchemos a linha
1, depois a 2 e assim sucessivamente, ate a linha 8.
2) Por cada burado de uma linha devem passar exatamente 7 fios
3) Dois fios que ja passaram por um mesmo buraco de uma linha nao
poderao passar pelo um mesmo buraco de qualquer das linhas posteriores

OBS1 : Note que ao passar 1 fio por um burado, ele surgira "nas
costas" da tabua magica, ou seja, apos preencher uma linha voce vai
precisar "virar a tabua" para poder preencher a proxima linha.

Como resolver este quebra-cabeca ?

Um Abraco a Todos !
PSR, 21105090E08
=
Instru��es para entrar na lista, sair da lista e usar a lista em
http://www.mat.puc-rio.br/~obmlistas/obm-l.html
=


Re: [obm-l] formulas para numeros primos

2009-05-11 Por tôpico Paulo Santa Rita
Oi Eric e demais colegas
desta lista ... OBM-L,

Li rapidamente o seu livro e gostei. Vou le-lo com mais vagar,
adiante. Por oportuno gostaria de saber se voce ja se ocupou com a
questao sobre a influencia dos inteiros gaussianos na distribuicao dos
primos.

Nos sabemos que muitos numeros primos passam a ser compostos no caso
de considerarmos os inteiros gaussianos. Por exemplo : 5=(2+i)(2-i) =>
5 nao e primo. Chamando de numero primo APENAS AQUELES INTEIROS
POSITIVOS QUE TAMBEM SAO
PRIMOS NO DOMINIO DOS INTEIROS GAUSSIANOS, como fica o teorema da
distribuicao dos numeros primos neste contexto ?

Teorema da distribuicao dos numeros primos :

Seja pi(n) o numero de numeros primos p tais que 2 =< p =< n. Entao :

LIM (  pi(N) / ( N/Log(N) )  ) = 1

Na expressao acima, Log(N) e o logaritmo natural de N ( base e=2,7... )

Um abraco
PSR, 21105090B05

Aproveito a oportunidade para lhe solicitar um esclarecimento, caso
voce ja tenha se ocupado
do que vou relatar :

2009/5/11 Eric Campos Bastos Guedes :
>
> Saudacoes aos colegas da lista
>
> Acabo de disponibilizar na internet meu trabalho
> sobre formulas para numeros primos. O endereco eh:
>
> http://www.docstoc.com/docs/5851750/F%C3%B3rmulas-para-N%C3%BAmeros-Primos
>
> gostaria da opiniao dos membros da lista a respeito.
>
>
> -
> [ eric campos bastos guedes - matemático e educador ]
> [ ERIC PRESIDENTE 2010 - Pela Democracia Direta! -- ]
> [ O maior especialista do mundo em fórmulas para primos ]
> [ sites: http://fomedejustica.blogspot.com/ --- ]
> [ http://www.orkut.com.br/Main#Community.aspx?cmm=20551500 ]
> [ http://portaldovoluntario.org.br/people/58657-eric-campos-bastos-guedes ]
> [ http://www.publit.com.br/index.php?author_id=255  ]
> -
>
>
>
>
> _
> Novo Internet Explorer 8. Baixe agora, é grátis!
> http://brasil.microsoft.com.br/IE8/mergulhe/?utm_source=MSN%3BHotmail&utm_medium=Tagline&utm_campaign=IE8
> =
> Instruções para entrar na lista, sair da lista e usar a lista em
> http://www.mat.puc-rio.br/~obmlistas/obm-l.html
> =
>

=
Instru��es para entrar na lista, sair da lista e usar a lista em
http://www.mat.puc-rio.br/~obmlistas/obm-l.html
=


[obm-l] Re: [obm-l] SÉRIES HARMÔNICAS!

2009-05-11 Por tôpico Paulo Santa Rita
Ola Jorge e demais colegas
desta lista ... OBM-L,

Nao entendi bem o excerto abaixo ... Voce que uma prova para esta desigualdade ?

> A propósito, como valer a desiguldade para qualquer n natural maior que 1?
> 1/n-1 + 1/n + 1/n+1 > 3/n

Existem muitas maneiras de fazer isso, vou usar a que me parece,
no momento, mais produtiva. Seja A_1, A_2, ..., A_(2p-1) uma
progressao aritmetica com um numero impar de termos. E claro que o
termo central e o termo de ordem P. E igualmente claro que podemos
exprimir todos os demais termos em funcao deste termo.

Exemplo 1

A_1, A_2 e A_3. Aqui P=2 e A_2 e o termo central. Logo : A_1 = A_2 -
r, A_3=A_2 + r
( "r" e a razao da PA )

Exemplo 2

A_1, A_2, A_3, A_4, A_5. Aqui P=3 e A_3 e o termo central. Logo :
A_1=A_3 - 2r, A_2=A_3 - r, A_4=A_3 + r   e   A_5 = A_3 + 2r

Considerando agora que a Media Harmonica (H) nunca e maior que a Media
Aritmetica, teremos que :

H = (2P-1) / ( (1/A_1) + (1/A_2) + ... + ( 1/A_(2p-1) ) ) =< ( A_1 +
A_2+...+ A_(2p-1) ) / (2P-1) = A

Na media A, ao exprimir cada A_i # A_p em funcao de A_p e "r", a soma
subsequente fara com que os "r"  desaparecam. Surgirao entao (2p-1)
termos A_p. Donde :

1/(A_1)  +  1/(A_2)  + ... + 1/(A_(p-1))  >= (2p-1) / A_p( resultado 1 )

No seu caso particular temos a PA N-1, N e N+1. Aqui,  P=2 e A_2 = N. Logo

1/n-1  +  1/n  +  1/n+1  >  3/n
que e o que queriamos demonstrar.

Agora, vou lhe mostrar porque tratei diretamente de um caso mais geral
( resultado 1 ) :

Considere os 9 termos da serie  harmonica 1 + (1/2) + ... + (1/9).
Neste caso, aplicando diretamente o resultado acima, teremos :

1 + (1/2) + ... + (1/9)  >= (2*5-1) / A_5 = 9/5 =1.8
(resultado 2)

Agora, ao inves de aplicar o resultado que obtivemos diretamente a
soma toda vamos dividi-la em grupos de 3 e aplica-la separadamente a
cada grupo. O que resultara ?

1 + (1/2) + (1/3) >= 2/2 = 2/2=1
(1/4) + (1/5) + (1/6) >= 2/5
(1/7) + (1/8) + (1/9) >= 2/8

1 + (1/2) + ... + (1/9) >= 1 + (2/5) + (2/8) = 66/40 = 33/20 = 1,65
 Surpreso ?

Portanto, a  aplicacao em casos com menor numero de termos e com soma
ulterior da uma cota inferior pior ... Com aperfeicoar isso ? Basta
lembra que a media geometrica fica "mais proxima" da media harmonica
que a media geometrica. Logo :

H = N / ( (1/A_1) + (1/A_2) + ... + ( 1/A_n ) =< (A_1*A_2*...*A_n)^(1/n)

(1/A_1) + (1/A_2) + ... + ( 1/A_n ) >= N/ (A_1*A_2*...*A_n)^(1/n)
No caso em que A_1, A_2 ... e 1, 2, ..., teremos :
1 + (1/2) + ... + ( 1/n ) >= N/( (N!)^(1/N) )

Aplicando no caso 1 + (1/2) + ... + (1/9) teremos :1 + (1/2) + ... +
(1/9)  >= 2.17...
Portanto, uma aproximacao melhor !

Uma outra de cotar a serie harmonica seria assim :

1/4   < 1/3 < 1/2
1/4 =< 1/4 < 1/2
1/8   < 1/5 < 1/4
1/8   < 1/6 < 1/4
1/8   < 1/7 < 1/4
1/8  =< 1/8 < 1/4
1/16 < 1/9 < 1/8
...

Ao somar tudo, do lado esquerdo teremos um "multiplo de (1/2) + (
eventualmente ) pequena fracao" e do lado direito teremos "multiplo de
1 + pequena fracao". esta tecnica tambem nos fornece cotas para a soma
da serie harmonica. Note que usamos potencias de (1/2) por uma mera
questao de tradicao, poderiamos ter usado potencias de (1/3), (1/5)
etc.


Para um dado inteiro P > 0, como determinar o menor N tal que 1+(1/2)
+ ... + (1/N) > P. E uma boa pergunta, um belo problema. O que voce
tem a dizer a respeito ?




> Afinal! Qual o número de parcelas necessárias para que a série harmônica
> atinja o número cem?  1 + 1/2 + 1/3 + 1/4...

Vou usar o seguinte :

1/3 > 1/4
1/4 >= 1/4
De onde sai  : 1/3 + 1/4 > 1/2

1/5 > 1/8
1/6 > 1/8
1/7 > 1/8
1/8 >= 1/8
De onde sai : 1/5 + 1/6 + 1/7 + 1/8 > 1/2

Estes exemplos evidenciam que ao somarmos 2^N parcelas a partir de
1/(2^N + 1) o resultado sera maior que 1/2. Portanto, fazemos :

N/2 > 100   => N > 200.

logo : 2 + 4 + 8 + .. + 2^(201) = 2((2^201  - 1)

Ou seja, precisamos somar 2 + 2(2^201 - 1) = 2^202 parcelas de 1+
(1/2) + ... + (1/N) + para obtermos uma soma superior a 100. Note que
nao estabelecemos a precisao desta cota, vale dizer, pode ocorrer que
com um numero "menor" ( e mesmo assim, muito grande ) de parcelas
consigamos ultrapassar 100.

Fica a questao que coloquei acima :

Dado P inteiro positivo. Qual o menor N tal que 1 + (1/2)  +...+ (1/N) > P ?


> A existência de infinitos primos gêmeos ainda é um problema em aberto. O
> teorema de Brun afirma que mesmo que existam infinitos termos nesta soma, a
> série resultante é ainda assim convergente, em contraste com a série dos
> inversos dos primos que é divergente. B2 = (1/3+1/5) + (1/5+1/7) +
> (1/11+1/13) + (1/17+1/19) + (1/29+1/31) +...~1,9021605823
>
> Incrível, não! As séries harmônicas estão cheias de coisas incríveis, muitas
> já descobertas, mas certamente muitas outras ainda à espera de um novo Euler
> que as desvende...

E verdade ! Ha algum tempo atras eu me ocupei da questao :

Qual o menor "r" real tal que a serie dos inversos dos primos, 1 +
(1/2)^r + (1/3)^r + (1/5)^r + (1/7)^r + ...,  converge ?

Infelizmente, eu so consig

Re: [obm-l] Triangulo oco - centro de gravidade

2009-05-10 Por tôpico Paulo Santa Rita
Oi Ponce e demais colegas
desta lista ... OBM-L,

EU PENSO que um possivel objetivo comum a todos nos e justamente o de
propor problemas interessantes, nao triviais, e/ou suas respectivas
solucoes, quando possiveis.  Procedendo assim PARECE-ME que ocorrerao
duas implicacoes obvias :

1) Serao naturalmente afastados aqueles(as) que vem esta lista como um
SOS para vestibulandos, concurseiros e assemelhados.
2) Serao naturalmente atraidos  aqueles(as) que aspiram ler e aprender
uma Matematica de mais alta qualidade.

E eu tenho certeza - ate por experiencia pessoal - que com a
colaboracao das inumeras pessoas talentosas que ja passaram por aqui,
podemos criar um centro de discussao de Matematica Elementar que nao
ficara devendo a nenhum outro centro do Mundo

Quando ao problema do Nehab, e bastante razoavel supor que o campo
gravitacional e constante ( pois estamos imaginando o triangulo
proximo a superficie da terra). Agora, basta aplicar o teorema de
Varignon.

Seja ABC um triangulo. Sem perda de generalidade podemos supor que os
seus vertices estao disposto tais que A esta em (0,0), B em (c,0) e C
em (X,Y). E natural supor que a densidade linear de massa e constante
em cada lado, donde decorre que os respectivos centros de massa dos
seus lados estarao nos pontos medios destes lados. Seja d esta
densidade liner. As forcas que atuam em cada ponto medio sao :

No lado AB, cdg, atuando em (c/2,0)   -  aqui,  g e a aceleracao da gravidade
No lado AC,  bdg, atuando em (X/2,Y/2)
No lado BC, adg, atuando em ( (c+X)/2 , Y/2 )

Assim VERTICALMENTE, temos uma forca de cdg em 0, uma forca bdg em Y/2
e uma forca adg em Y/2, vale dizer, uma forca, (bdg+adg) em Y/2. A
forca que equilibra tudo isso tem modulo, obviamente, igual "adg + bdg
+ cdg" atuando na posicao vertical m ( Varignon agora ) tal que :

m*(adg + bdg + cdg) = cdg*0 + (bdg + adg)*(Y/2)
m =( (a+b) / (a+b+c) )*(Y/2)   Resultado 1

HORIZONTALMENTE, temos uma forca cdg em c/2, uma forca bdg em (X/2) e
uma forca adg atuando em (c+X)/2. Para equilibrar isso devemos ter uma
forca de modulo  obviamente, igual "adg + bdg + cdg" e atuando na
posicao horizontal n ( Varignon agora ) tal que :

n*(adg + bdg + cdg)  = cdg*(c/2) + bdg*(X/2) + adg*( (c+X)/2)
n = (  (a+c)*(c/2) + (b+c)*(X/2)  ) / (a + b + c) Resultado 2

Os dados em Resultado 1 e 2 sao conhecidos, bastando construir o
sistema de coordenadas indicado ( traca-se por A uma perpendicular a
AB, Esta perpendicular e o eixo Y; o eixo X sera a reta que suporta o
lado AB). Os valores de "m" e "n" sao facilmente construtiveis. Logo:

O PONTO PROCURADO

Traca-se por m uma paralela ao eixo X e por n uma paralela ao eixo Y.
O ponto de encontro e o ponto procurado.

Um Abracao a Todos !
PSR,11005091409


pois isto afastara aqueles estudantes preguicosos ,  para que aqueles
que nos acompanham
2009/5/10 Rogerio Ponce :
> Ola'  Santa Rita,
> obrigado pelo elogio.
> Mas o "culpado" foi voce mesmo, ao trazer problemas interessantes para a 
> lista.
> Parabens!
>
> Alias, o Nehab e' outro provocador  - vide o probleminha que ele
> propos (e que eu reescrevo abaixo):
>
>
> Dado o desenho de uma armacao triangular, feita de arame extremamente
> fino (unidimensional), encontre o seu centro de gravidade,
> utilizando-se apenas de desenho geometrico.
> (ou seja, a solucao tem que ser tracada, nao podendo ser apenas
> expressa por meio de equacoes.)
>
> Abracos a todos,
> Rogerio Ponce
>
>
>
> 2009/5/9 Paulo Santa Rita :
>> Ola Ponce, Nehab, Luis Lopes
>> e demais colegas desta lista ... OBM-L,
>>
>> Ponce, a sua solucao, simples e bela, e tipica de uma Matematica de
>> qualidade. Parabens por ela !
>> Fico feliz por ter iniciado uma discussao que lhe interessou, trouxe o
>> Ralph, o Shine e levou outros Matematicos de qualidade a se
>> manifestarem.
>>
>> ENTRE MUITO OUTROS Matematicos de qualidade que outrora apareciam por
>> aqui e que ja ha algum tempo nao escrevem, sem duvida se incluem o
>> Nicolau e o Gugu. Oxala eles voltem a escrever brevemente !
>>
>> Uma questao que sempre me interessou, subsidiariamente, e a seguinte :
>>
>> Sabemos que o INCENTRO ( Centro do Circulo inscrito a um triangulo )
>> nao faz parte da reta de Euler, isto e, ele nao esta NECESSARIAMENTE
>> alinhado com os pontos notaveis que pertencem a esta reta. Assim em
>> geral, o incentro, o circuncentro e o ortocentro formam um pequeno
>> triangulo no interior de um triangulo dado. O que se pode falar sobre
>> esse pequeno triangulo ? Que relacao ele mantem com o triangulo
>> original ?
>>
>> Um abraco a Todos !
>> PSR,7090509132D
> =
> Instruções para entrar na lista, sair da lista e usar a

Re: [obm-l] DG: [Era: serie para ln(2)]

2009-05-09 Por tôpico Paulo Santa Rita
Ola Ponce, Nehab, Luis Lopes
e demais colegas desta lista ... OBM-L,

Ponce, a sua solucao, simples e bela, e tipica de uma Matematica de
qualidade. Parabens por ela !
Fico feliz por ter iniciado uma discussao que lhe interessou, trouxe o
Ralph, o Shine e levou outros Matematicos de qualidade a se
manifestarem.

ENTRE MUITO OUTROS Matematicos de qualidade que outrora apareciam por
aqui e que ja ha algum tempo nao escrevem, sem duvida se incluem o
Nicolau e o Gugu. Oxala eles voltem a escrever brevemente !

Uma questao que sempre me interessou, subsidiariamente, e a seguinte :

Sabemos que o INCENTRO ( Centro do Circulo inscrito a um triangulo )
nao faz parte da reta de Euler, isto e, ele nao esta NECESSARIAMENTE
alinhado com os pontos notaveis que pertencem a esta reta. Assim em
geral, o incentro, o circuncentro e o ortocentro formam um pequeno
triangulo no interior de um triangulo dado. O que se pode falar sobre
esse pequeno triangulo ? Que relacao ele mantem com o triangulo
original ?

Um abraco a Todos !
PSR,7090509132D







> Rogerio Ponce :
> Ola' Nehab, Santa Rita, Luis Lopes e pessoal da lista,
> estou gostando dessas histórias !
>
> ...menos, é claro, da intenção do Nehab em me incluir na lista dos
> "quase coroas", visto que ele já conhecia o Bourbaki de trás pra
> frente, há mais de 10 anos, quando o único conjunto que eu conhecia
> era o dos Beatles...
>
> Xiii me entreguei
>
> Mas voltando 'a vaca fria, vamos resolver o problema do Santa Rita, ou seja,
> vamos tentar encontrar algum triangulo tal que o seu perimetro seja
> igual a soma das suas medianas.
>
> Entao, considere um triangulo ABC, e seja D o ponto medio do lado BC.
> Pois agora imagine os vetores AB e AC, com origem em A.
>
> Repare que a soma desses 2 vetores vale exatamente o dobro da mediana AD.
>
> Por outro lado, a gente sabe que a soma de dois vetores quaisquer
> vale, no maximo, a soma dos dois modulos.
>
> Portanto, a mediana AD vale no maximo a metade da soma dos
> comprimentos AB e AC, ou seja,
> 2*AD <= AB + AC
>
> Repita essa desigualdade para as outras medianas, e some tudo.
> Fica facil concluir que:
>  A SOMA DAS MEDIANAS E' SEMPRE MENOR OU IGUAL AO PERIMETRO DO TRIANGULO.
>
> Alias, essa igualdade so' acontece se os angulos entre os vetores
> forem zero, o que significa que o triangulo tem que ser degenerado.
> E, de fato, isso acontece quando um dos lados do triangulo tem comprimento 
> zero.
>
> []'s
> Rogerio Ponce
>
>
>
>
> 2009/5/6 Carlos Nehab :
>> Caramba,
>>
>> Falam em antiguidades e mencionam logo meu nome.  Não sei porque... :-) .
>>
>> Você já mencionaram dois maiores monstros do passado em Geometrias
>> (imaginem... o quanto passado...o meu passado! hahaha).
>> O Virgilio de Athayde Pinheiro e o Célio Pinto de Almeida (que depois foi
>> dono da construtora que levava seu nome).
>> O primeiro, um sábio, um verdadeiro mestre, de corpo e alma (falava grego
>> fluentemente, era um poço infinito de conhecimento, inclusive sobre história
>> da Matemática, aspecto tão negligenciado hoje em dia (para os alunos fica a
>> horrível sensação que tudo em matemática sempre foi do mesmo jeito
>> semprecomo se matemática fose uma descoberta dos deuses e não dso
>> homens...).
>>
>> Tive o privilégio de ter sido aluno do Virgílio em Geometria Descritiva e
>> Perspectiva(s).
>> Do segundo élio) fui aluno de Desenho Geométrico (ai incluidas as Cônicas):
>> um monstro e um extraordinário professor.
>>
>> Mas havia um outro monstro sagrado, tímido e introspectivo, que foi
>> professor do IME e da UFF (Dep de Matemática) - Luiz Oswaldo - e tive
>> oportunidade de ser aluno dele em ambas as escolas.  No IME, de Geometria, e
>> na UFF de Teoria dos Números e de Geometria (foi através dele que conheci e
>> me extasiei com o livro do Niven - Irrational Numbers, já mencionado algumas
>> vezes por aqui).
>>
>> Eram do Luiz Oswaldo a grande maioria das questões de Geometria dos
>> concursos de admissão ao IME na década de 65 a75, inclusive as questões de
>> Geometria da prova de 72/73 onde tive o prazer de trabalhar com ele (eu já
>> dava aula lá) e participar de forma intensa no massacre da prova de Álgebra
>> daquele ano. ;-)
>>
>> Para quem não se lembra eu e o Ponce (um "quase coroa da lista") já
>> escrevemos por aqui "causos" engraçados sobre o Luiz Oswaldo, inclusive sua
>> ridícula e única gravata de seu sovina vestuário.
>>
>> Mas eu tenho os livros do Virgílio de Descritiva, os do Célio, de Cônicas e
>> de outras "cositas" deles.
>>
>> Quanto ao problema proposto pelo Santa Rita (perímetro e medianas) eu tb não
>> o havia visto ainda e de fato, como o Luis mencionou, não seria um problema
>> digamos clássico, pois não é muito comum, na bibliografia, a sistematização
>> de problemas contendo somas, diferenças etc.  (vide uma das bíblias em
>> Wernick, W. "Triangle Constructions with Three Located Points." Math. Mag.
>> 55, 227-230, 1982.) e diversos outros papers que vão completando a lista do
>> Wernick.  Eu 

Re: [obm-l] DG: [Era: serie para ln(2)]

2009-05-06 Por tôpico Paulo Santa Rita
Ola Luis e Nehab e demais
colegas desta lista ... OBM-L,

Bom, eu nao conheci qualquer dos grandes Mestres citados abaixo e,
muito menos, tive o prazer de estudar por qualquer dos livros deles.
Assim, se por omissao cometi alguma injustica, asseguro que foi
involuntária.

Eu so estudei em escolas publicas. Tive um pessimo 1 grau e o 2 grau
foi pior ainda. Lembro que na antiga 7 serie eu estava totalmente
decepcionado com a miseravel Matematica que ensinavam e estudava por
conta propria sobretudo Filosofia e Filologia( Lia W Durant,, As obras
completas do Freud, Os Pensadores, Goethe e Shakespeare etc ). Foi
mais ou menos no fim desta serie que me deparei com um livro que veio
modificar totalmente a minha vida ...

Comprei no Sebo um livrinho sem capa, sobre Geometria, do Prof Eduardo
Wagner ( e acho que falecido Prof Morgado tambem era autor ). A
leitura deste livro me mostrou algo que, naquela epoca, eu nem
suspeitava : que a Matematica podia ser bela, que poderiamos penetrar
no desconhecido a partir de algumas nocoes basicas e que, enfim, a
Matematica era uma ciencia digna da minha atencao.

Eu me lembro que fiquei tao impressionado com aquelas coisas que,
daquele momento em diante, eu nao conseguia pensar em mais nada. Perdi
todos os amigos, pois nem todo mundo gosta destas coisas e passei
sistematicamente a devorar tudo que eu podia estudar : Calculo,
Algebra, Teoria de  Galois etc. Li Hardy e Ramanujam e dai em diante.

E por esta razao que o Prof Eduardo Wagner e uma pessoa que sem duvida
tem um lugar muito especial na minha vida de Matematico. Inclusive a
minha primeira descoberta de infancia  em matematica, eu a batizei com
o seu nome como uma forma de agradecer o que ele representava para mim
naquele momento :

***INICIO***

( DESIGUALDADE WAGNER ) EM QUALQUER TRIANGULO, O SEMI-PERIMETRO NUNCA
E MENOR QUE A SOMA DOS PRODUTOS DE CADA LADO PELO COSSENO DO ANGULO
OPOSTO.

Para ver isso, seja ABC um triangulo qualquer, imaginado como se BC fosse a
base ( B a esquerda, C a direita ) e A o vertice.

1) Prolongando CA a partir de A, no sentido de C para A, de um segmento AD
igual AB.
2) Prolongando BA a partir de A, no sentido de B para A, de um segmento AE
igual a AC
3) Ligando D com E

Os triangulo ABC e ADE sao iguais (caso LAL), pois :

1) AD = AB (por construcao)
2) AE = AC (por construcao)
3) Angulo BAC = Angulo DAE (opostos pelo vertice)

Segue que DE=BC.

Trancando por A uma paralela a BC. Seja "r" esta paralela. Agora, seja F o
pe da perpendicular a "r" tracado por D. Seja G o pe da perpendicular a "r"
tracada por E.

Entao, claramente :

DE >= AD*cos(DAF) + AE*cos(EAG)

Mas :

1)DE = BC = a
2)AD = AB = c
3)AE = AC = b
4)Angulo DAF = Angulo ACB ( Angulos Correspondentes ) = Ang C
5)Angulo EAG = Angulo ABC ( Angulos Correspondentes ) = Ang B

Portando :

a >= c*cos(C) + b*cos(B)

Repetindo construcoes e raciocinios semelhantes para os demais vertices,
chegaremos a :

b >= a*cos(A) + c*cos(C)
c >= b*cos(B) + a*cos(A)

Somando estas tres desigualdades :

a + b + c >= 2*a*cos(A) + 2*b*cos(B) + 2*c*cos(C)
2p >= 2*a*cos(A) + 2*b*cos(B) + 2*c*cos(C)

Portanto :


DESIGUALDADE WAGNER :
p >= a*cos(A) + b*cos(B) + c*cos(C)

***FIM***

Estou falando isso porque , dado que ninguem duvida do verdadeiro e
belissimo talento do Prof Wagner e FOI ELE, POR E-MAIL, QUE ME FALOU
DO LIVRO DO LUIS LOPES, classificando tal livro como o que havia de
melhor em geometria do traingulo. Assim, caro Luis, quem me falou
sobre a excelencia do seu trabalho foi o Wagner e, concorde comigo, O
Prof Wagner sabe perfeitamente do que fala quando o tema e a Geometria
Euclidiana...

Assim, a minha referencia infantil mais forte foi este ilustre Prof
Eduardo Wagner. Ele acendeu o estopim, disparou o Processo. Por que ?
Porque o Prof Wagner e um artista, as solucoes que ele apresenta tem
um "que" de beleza, tal como o Lagrange fazia. Assim, o trabalho dele
encanta e com ele aprendi a nao so  matematica, mas sobretudo a buscar
a Matematica Bonita.

Sobre o problema, como ocorre com frequencia, ele simplesmente surgiu
na minha cabeca. Se ele for realmente inedito e dele pudermos extrair
alguma coisa valiosa, eu me sentirei estimulado a pensar nele. Vou
aguardar o carissimo Nehab se pronunciar. Note que eu gostaria de ver
UMA CONSTRUCAO de um triangulo com tal propriedade, vale dizer, se o
problema for indeterminado e houver muitos triangulos assim, gostaria
de ver a construcao de um deles.

Um Abracao a Todos !
PSR, 40605092050

Em Tempo : prometo que da proxima vez vou escrever sobre Matematica.
Esta mensagem so tem texto porque estava parecendo que eu cometera uma
imensa injustica esquecendo outros grandes mestres. Nao foi isso.
Apenas realmente nunca ouvi falar ou li os livros destes grandes
professores.




2009/5/6 Carlos Nehab :
> Caramba,
>
> Falam em antiguidades e mencionam logo meu nome.  Não sei porque... :-) .
>
> Você já mencionaram dois maiores monstros do passado em Geometrias
> (imaginem... o quanto passado...o meu passado

Re: [obm-l] serie para ln(2)

2009-05-06 Por tôpico Paulo Santa Rita
Ola Bernardo e demais colegas
desta lista ... OBM-L,

1 ) Muito boa a sua mensagem, mas note que nao foi isso que o Luis
Lopes perguntou e, portanto, nao foi sobre o que eu respondi. Pelo que
eu entendi, o Luis quer saber se a colocao arbitraria de colchetes vai
afetar o valor original da serie ( sua soma), vale dizer, NAO HA A
HIPOTESE DE MESMO NUMERO DE TERMOS e, pior ainda, OS SINAIS DOS TERMOS
SAO PRESERVADOS, ou seja, ele simplesmente introduz colchetes e outros
delimitadores, SEM ALTERAR OS SINAIS ORIGINAIS, o que afeta o
resultado ainda mais.

2 ) Nao, amigo, descupe. Devo ter me expressado mal. Quero dizer :

Dado 2P, construa o triangulo tal que :
Soma das medianas = 2P
perimetro = 2P

Eu nao pensei sobre a questao. Nem sei se e trivial ou trabalhosa. Ela
veio a minha cabeca quando respondia o Luis Lopes. Como o Luis e um
Mestre consumado no assunto, queria saber se tal questao ja constava
no livro dele ou se ele dispunha de tempo pra apresentar uma solucao,
caso exista.

Um Abracao
PSR, 40605091048



2009/5/6 Bernardo Freitas Paulo da Costa :
> Grande Paulo ! Mas eu gostaria de tentar dar um "palpitezinho"... e
> vou acabar dando dois :
>
> 2009/5/6 Paulo Santa Rita :
>> Ola Luis e demais colegas
>> desta lista ... OBM-L,
>>
>> 1) O valor de uma serie ( sua "soma" ) e formalmente definido como em
>> qualquer serie, vale dizer, como limite das suas reduzidas. Assim, se
>> :
>>
>> S = a1 + a2 + a3 + ...  = L
>>
>> Entao a sequencia Rn = a1 + ... + an e tal que Lim Rn = L. Neste
>> particular caso que voce cita o agrupamento nao vai causar problemas ,
>> conforme pode-se ver considerando as suas reduzidas. No caso geral,
>> PRESERVANDO-SE O SINAL DOS TERMOS, nao pode. Basta , mais uma vez,
>> considerar as reduzidas.
>>
>> A resposta mais geral seria, entao, assim:
>>
>> "Se e possivel provar que a sequencia das reduzidas original converge
>> para o mesmo valor que a nova sequencia das reduzidas ( aquelas
>> derivadas do agrupamento ) entao o agrupamento e valido"
>>
>> 2) Estas series sao melhor tratadas em espacos de sobolev
> Eu tenho muito pouca prática com Sobolev, mas se você pudesse
> detalhar... Eu vou detalhar o que eu pensei quando escrevi :
>
> Temos uma soma S, com termos a1, a2, a3, ... Faça a seguinte hipótese
> : a_n tende a zero. Então, agrupando os termos SEMPRE EM MESMO NUMERO,
> se a primeira soma existir, a agrupada também, e vale o mesmo valor. A
> idéia, é exatamente voltar à definição que o Paulo deu mais acima, e
> notar que um grupo equivale a somar N termos sucessivos. Logo, somar n
> grupos, equivale a tomar a nN-ésima parcial. Sabemos que a seqüência
> das parciais converge, logo a das "N"-parciais também, e para o mesmo
> valor, pois é uma subseqüência. Muito mais interessante é tentar a
> recíproca : se a soma dos agrupamentos converge, o que acontece com a
> soma original ? E é aí que a hipótese inicial tem todo o seu sentido:
> sabemos que a subseqüência S_{Nk} converge para S, e queremos estudar
> a seqüência completa S_n. Ora, como os termos tendem a zero, de S_{Nk
> + 1} = S_{Nk} + a_{Nk + 1}, tomando o limite no termo da direita (que
> existe !), vemos que lim S_{Nk + 1} existe também, e vale S + lim
> a_{Nk + 1} = S. Como, além disso, temos um número finito de restos
> modulo N, e todas as subseqüências S_n{Nk + a} convergem para S (faça
> uma depois da outra !!), a seqüência original também converge para S,
> pois a partir de um certo N_0, todos os S_{Nk} estão a epsilon de S,
> para um N_1, os S_{Nk + 1}, etc e tal, logo para o máximo dos N_a,
> todo mundo está a epsilon de S. Note que a finitude (que vem da
> regularidade dos parênteses) é crucial, bem como a hipótese que os
> termos tendem a zero !!
>
>> 3) No fim da minha mensagem eu citei uma solucao diferente da que
>> apresentei, usando polinomios ( que, inclusive, gneraliza a questao ).
>> Agora, repensando, surgem muitas outras maneiras de tratar o problema
>> ...
>>
>> 4) Agora, Luis, tive uma ideia que talvez signifique uma humilde
>> contribuicao ao seu monumental trabalho sobre triangulos. Veja se voce
>> ja fez ou se e uma nova questao :
>>
>> PROBLEMA : Dado 2P. Construa o triangulo tal que a soma das suas
>> medianas e o seu perimetro sejam 2P.
> Não entendi, você quer que a soma das medianas seja 2P, e o perímetro
> também seja 2P ? Ou a soma a+b+c + m_a+m_b+m_c = 2P ? Porque, se for
> esse último (que foi como eu interpretei "de chofre") parece que tem
> uma solução para 3a + 3a\sqrt{3}/2 = 2P ...
>
>> OBS : Para aqueles que estao acompanhando esta mensagem, saibam que
>> Luis Lopes e autor do maior e mais completo livro DO MU

Re: [obm-l] serie para ln(2)

2009-05-06 Por tôpico Paulo Santa Rita
Ola Luis e demais colegas
desta lista ... OBM-L,

1) O valor de uma serie ( sua "soma" ) e formalmente definido como em
qualquer serie, vale dizer, como limite das suas reduzidas. Assim, se
:

S = a1 + a2 + a3 + ...  = L

Entao a sequencia Rn = a1 + ... + an e tal que Lim Rn = L. Neste
particular caso que voce cita o agrupamento nao vai causar problemas ,
conforme pode-se ver considerando as suas reduzidas. No caso geral,
PRESERVANDO-SE O SINAL DOS TERMOS, nao pode. Basta , mais uma vez,
considerar as reduzidas.

A resposta mais geral seria, entao, assim:

"Se e possivel provar que a sequencia das reduzidas original converge
para o mesmo valor que a nova sequencia das reduzidas ( aquelas
derivadas do agrupamento ) entao o agrupamento e valido"

2) Estas series sao melhor tratadas em espacos de sobolev

3) No fim da minha mensagem eu citei uma solucao diferente da que
apresentei, usando polinomios ( que, inclusive, gneraliza a questao ).
Agora, repensando, surgem muitas outras maneiras de tratar o problema
...

4) Agora, Luis, tive uma ideia que talvez signifique uma humilde
contribuicao ao seu monumental trabalho sobre triangulos. Veja se voce
ja fez ou se e uma nova questao :

PROBLEMA : Dado 2P. Construa o triangulo tal que a soma das suas
medianas e o seu perimetro sejam 2P.

OBS : Para aqueles que estao acompanhando esta mensagem, saibam que
Luis Lopes e autor do maior e mais completo livro DO MUNDO sobre
construcao geometrica de triangulos. Pena que ( pelo que sei ) ainda
nao ha uma versao em portugues

Um Abracao
PSR, 40605090832







2009/5/5 Luís Lopes :
> Sauda,c~oes,
> Oi Paulo e para os outros três que responderam,
>
> Então de
>
> 1 - (1/2) + 1/3 - (1/4) + 1/5 - (1/6) + ... = Ln(2)
>
> posso fazer
>
> [1 - (1/2)] + [1/3 - (1/4)] + [1/5 - (1/6)] + ...  e obter
> o mesmo resultado?? Sempre, ou seja, posso botar [ ]
> à vontade em séries cond. convergentes?
>
> Ando sempre em águas turvas com estas manipulações de
> séries cond. conv.
>
> P.S.: Paulo, o Rousseau acabou de me "dizer" que encontrou
> uma solução para aquela conjectura. Mas não a tenho.
> Obrigado pelo seu email a respeito. Muito trabalho nele.
>
> []'s
> Luís
>
>
>> > From: paulo.santar...@gmail.com
>> > Date: Mon, 4 May 2009 18:22:28 -0300
>> > Subject: Re: [obm-l] serie para ln(2)
>> > To: obm-l@mat.puc-rio.br
>> >
>> > Ola Luis e demais colegas
>> > desta lista ... OBM-L,
>> >
>> > A expressao 1(2n(2n-1)) pode ser olhada assim ::
>> >
>> > 1/(2n(2n-1)) = ( 1/(2n-1) ) - ( 1/2n )
>> >
>> > Assim, para n=1, 2, 3, ...
>> >
>> > 1 - (1/2) + 1/3 - (1/4) + 1/5 - (1/6) + ... = Ln(2)
>> >
>> > De maneira geral, se A1, A2, A3, ... e uma PA, a expressao
>> >
>> > soma de 1/(Ai+1*Ai+2*Ai+3*...*Ai+k) onde i = 1, 2, 3, ... e k>=2
>> > Tambem permite uma "olhada especial" de onde deriva sua soma. Como fazer 
>> > isso ?
>> >
>> > Exemplo :
>> >
>> > 1/(Ai*Ai+1*Ai+2) = (1/(2(r^2)))*((1/Ai) - (2/(Ai+1)) + (1/Ai+2))
>> > Use o fato de que Ai= (Ai+1) - r e Ai+2 = (Ai+2) + r, onde "r" e
>> > a razao da PA
>> >
>> > Agora, considere o seguinte :
>> >
>> > Seja S = 1 - (1/2) + (1/3) - (1/4) + ... = Ln(2)
>> >
>> > Nos "olhar" esta serie assim : +-+-+-+- ... onde, sem tirar os termos
>> > de lugar, notamos que em cada posicao par ha um sinal "-" e em cada
>> > posicao impar ha um sinal "+". Representarei este fato com a notacao S
>> > (1,1), significando que apos 1 sinal par segue um impar.
>> >
>> > O simbolo S(2,3) significa que apos 2 sinais "+" sempre seguem 3
>> > sinais "-", assim :
>> >
>> > S(2,3) = 1 + (1/2)-(1/3)-(1/4)-(1/5)+(1/6)+(1/7) - (1/8)-(1/9)-(1/10)+...
>> >
>> > Eu afirmo que S(N,P) converge se N e P sao inteiros positivos. Como
>> > provar isso ?
>> >
>> > Um Abraco a Todos
>> > PSR, 20405091800
>> >
>> >
>> > 2009/5/4 Luís Lopes :
>> > > Sauda,c~oes,
>> > >
>> > > No meio de vários  ao thread CEGUEIRA FRACIONÁRIA!
>> > > encontrei a seguinte mensagem:
>> > >
>> > >> [obm-l] Mais um divertimento: 0 > 1/2 (???)
>> > >> Albert Bouskela
>> > >> Thu, 18 Dec 2008 10:15:24 -0800
>> > >> Amigos:
>> > >>
>> > >> Já que o divertimento anterior foi um sucesso de público e crítica, aí 
>> > >> vai
>> > >> o
>> > >> segundo:
>> > >
>> > > [...]
>> > >
>> > >
>> > >> E, assim, "demonstra-se" que 0 > 1/2 (???)
>> > >>
>> > >> Onde está o erro?
>> > >>
>> > >> Uma curiosidade:
>> > >> soma ( 1/2n(2n-1) , n = 1 ... +oo ) = 1/2 + 1/12 + 1/30 + ... = ln(2) =
>> > >> 0,69 >
>> > >> 1/2
>> > >
>> > > [...]
>> > >
>> > > Como demonstrar a curiosidade acima?
>> > >
>> > > []'s
>> > > Luís
>> > >
>> > >
>> > > 
>> > > Conheça os novos produtos Windows Live. Clique aqui!
>> >
>> > =
>> > Instru??es para entrar na lista, sair da lista e usar a lista em
>> > http://www.mat.puc-rio.br/~obmlistas/obm-l.html
>> > =
>>
>>
>>
>> Descubra uma nova internet. Internet Explorer 8. Mergulhe.
>

[obm-l] Re: [obm-l] QUESTÃO DO ITA 92

2009-05-05 Por tôpico Paulo Santa Rita
Ola Pessoal,

Correcao :

No item I) eu quis dizer :
"o determinante da matriz dos coeficientes da incognitas E IGUAL A ZERO"


2009/5/5 Paulo Santa Rita :
> Ola Vanderlei e demais
> colegas desta lista ... OBM-L,
>
> Nos podemos pensar em A como os coeficientes numeros das incoginitas
> de um sistema ( linear ) de tres equacoes a tres incognitas. Olhando
> assim :
>
> I ) Obviamente verdadeira, pois um sistema homogeneo so admite solucao
> diferente da trivial  ( solucao trivial : (0,0,0) ) se o determinante
> da matriz dos coeficientes das incognitas e diferente de zero.
>
> II) Obviamente falsa, pois basta tomar um Y tal que a segunda e
> terceira coluna da matriz dos coeficientes das incoginitas juntas com
> o Y forme uma matriz 3x3 com determinante diferente de zero. Neste
> caso, as caracteristicas da matriz principal e secundaria serao
> diferente e, pelo teorema de rouche, teremos um sistema impossivel.
>
> O Teorema de Rouche permite discutir um sistema linear considerando as
> caracteristicas da matriz principal ( matriz dos coeficientes das
> incoginitas, caracteristica = X ) e da matriz segundaria ( matriz
> principal + coluna dos termos independentes, caracteristica = Y ).
> Vale o seguinte :
>
> X=Y=N   => sistema possivel e determinado
> X=Y < N  => sistema possivel indeterminado
> X # Y   => sistema impossivel
>
> NOTA : caracteristica de uma matriz ( tambem chamada de outros nomes.
> Estou usando esta expressao porque voce parece ser estudante de nivel
> medio ) e a ordem da matriz de maior ordem com determinante diferente
> de zero contido na matriz sob consideracao.
>
> Um Abracao
> PSR< 3050509120F
>
>
> 2009/5/5 Vandelei Nemitz :
>> Seja A uma matriz 3 x 3 tal que detA = 0. Considere as afirmações:
>> I. Existe X 3 x 1 não nula tal que AX é identicamente nula.
>> II. Para todo Y 3 x 1, existe X 3 x 1 tal que AX = Y.
>>
>> pessoal, essas duas afirmações são tais que a primeira é verdadeira e a
>> segunda é falsa. Gostaria de alguma sugestão elegante para mostrar, uma vez
>> que a maneira que fiz ficou longa demais.
>>
>> Obrigado,
>>
>> Vanderlei
>>
>> OBS: A propósito, alguém tem a prova do ITA DE 1992 resolvida? Só falta essa
>> para minha coleção desde 1980.
>> Valeu
>

=
Instru��es para entrar na lista, sair da lista e usar a lista em
http://www.mat.puc-rio.br/~obmlistas/obm-l.html
=


[obm-l] Re: [obm-l] QUESTÃO DO ITA 92

2009-05-05 Por tôpico Paulo Santa Rita
Ola Vanderlei e demais
colegas desta lista ... OBM-L,

Nos podemos pensar em A como os coeficientes numeros das incoginitas
de um sistema ( linear ) de tres equacoes a tres incognitas. Olhando
assim :

I ) Obviamente verdadeira, pois um sistema homogeneo so admite solucao
diferente da trivial  ( solucao trivial : (0,0,0) ) se o determinante
da matriz dos coeficientes das incognitas e diferente de zero.

II) Obviamente falsa, pois basta tomar um Y tal que a segunda e
terceira coluna da matriz dos coeficientes das incoginitas juntas com
o Y forme uma matriz 3x3 com determinante diferente de zero. Neste
caso, as caracteristicas da matriz principal e secundaria serao
diferente e, pelo teorema de rouche, teremos um sistema impossivel.

O Teorema de Rouche permite discutir um sistema linear considerando as
caracteristicas da matriz principal ( matriz dos coeficientes das
incoginitas, caracteristica = X ) e da matriz segundaria ( matriz
principal + coluna dos termos independentes, caracteristica = Y ).
Vale o seguinte :

X=Y=N   => sistema possivel e determinado
X=Y < N  => sistema possivel indeterminado
X # Y   => sistema impossivel

NOTA : caracteristica de uma matriz ( tambem chamada de outros nomes.
Estou usando esta expressao porque voce parece ser estudante de nivel
medio ) e a ordem da matriz de maior ordem com determinante diferente
de zero contido na matriz sob consideracao.

Um Abracao
PSR< 3050509120F


2009/5/5 Vandelei Nemitz :
> Seja A uma matriz 3 x 3 tal que detA = 0. Considere as afirmações:
> I. Existe X 3 x 1 não nula tal que AX é identicamente nula.
> II. Para todo Y 3 x 1, existe X 3 x 1 tal que AX = Y.
>
> pessoal, essas duas afirmações são tais que a primeira é verdadeira e a
> segunda é falsa. Gostaria de alguma sugestão elegante para mostrar, uma vez
> que a maneira que fiz ficou longa demais.
>
> Obrigado,
>
> Vanderlei
>
> OBS: A propósito, alguém tem a prova do ITA DE 1992 resolvida? Só falta essa
> para minha coleção desde 1980.
> Valeu

=
Instru��es para entrar na lista, sair da lista e usar a lista em
http://www.mat.puc-rio.br/~obmlistas/obm-l.html
=


Re: [obm-l] serie para ln(2)

2009-05-04 Por tôpico Paulo Santa Rita
Ola Luis e demais colegas
desta lista ... OBM-L,

A expressao 1(2n(2n-1)) pode ser olhada assim ::

1/(2n(2n-1)) = ( 1/(2n-1) )  -  ( 1/2n )

Assim, para n=1, 2, 3, ...

1 - (1/2) + 1/3 - (1/4) + 1/5 - (1/6) + ... = Ln(2)

De maneira geral, se A1, A2, A3, ... e uma PA, a expressao

soma de 1/(Ai+1*Ai+2*Ai+3*...*Ai+k) onde i = 1, 2, 3, ... e   k>=2
Tambem permite uma "olhada especial" de onde deriva sua soma. Como fazer isso ?

Exemplo :

1/(Ai*Ai+1*Ai+2) = (1/(2(r^2)))*((1/Ai) - (2/(Ai+1)) + (1/Ai+2))
Use o fato de que Ai= (Ai+1) - r e Ai+2 = (Ai+2)  + r, onde "r" e
a razao da PA

Agora, considere o seguinte :

Seja S = 1 - (1/2) + (1/3) - (1/4) + ... = Ln(2)

Nos "olhar" esta serie assim : +-+-+-+- ... onde, sem tirar os termos
de lugar, notamos que em cada posicao par ha um sinal "-" e em cada
posicao impar ha um sinal "+". Representarei este fato com a notacao S
(1,1), significando que apos 1 sinal par segue um impar.

O simbolo S(2,3) significa que apos 2 sinais "+" sempre seguem 3
sinais "-", assim :

S(2,3) = 1 + (1/2)-(1/3)-(1/4)-(1/5)+(1/6)+(1/7) - (1/8)-(1/9)-(1/10)+...

Eu afirmo que S(N,P) converge se N e P sao inteiros positivos. Como
provar isso ?

Um Abraco a Todos
PSR, 20405091800


2009/5/4 Luís Lopes :
> Sauda,c~oes,
>
> No meio de vários  ao thread CEGUEIRA FRACIONÁRIA!
> encontrei a seguinte mensagem:
>
>> [obm-l] Mais um divertimento: 0 > 1/2 (???)
>> Albert Bouskela
>> Thu, 18 Dec 2008 10:15:24 -0800
>> Amigos:
>>
>> Já que o divertimento anterior foi um sucesso de público e crítica, aí vai
>> o
>> segundo:
>
> [...]
>
>
>> E, assim, "demonstra-se" que 0 > 1/2 (???)
>>
>> Onde está o erro?
>>
>> Uma curiosidade:
>> soma ( 1/2n(2n-1) , n = 1 ... +oo ) = 1/2 + 1/12 + 1/30 + ... = ln(2) =
>> 0,69 >
>> 1/2
>
> [...]
>
> Como demonstrar a curiosidade acima?
>
> []'s
> Luís
>
>
> 
> Conheça os novos produtos Windows Live. Clique aqui!

=
Instru��es para entrar na lista, sair da lista e usar a lista em
http://www.mat.puc-rio.br/~obmlistas/obm-l.html
=


Re: [obm-l] Representacao do Multinomio de Leibniz

2009-05-03 Por tôpico Paulo Santa Rita
Ola Nehab e demais colegas
desta lista ... OBM-L,

Voce gostou do Conto ? Fico Feliz ! Ser casado com uma escritora traz
algumas vantagens ... Aqui vai uma implicacao do sonho :

Eu precisaria, previamente, ter caracterizados as folheacoes ou faces
ocultas do triangulo de Pascal que descrevi no sonho. Usando a notacao
que la introduzi as coisas ficam mais faceis e diretas. Mas vou seguir
um atalho aqui.

Estarei imaginando o Triangulo de Pascal ( doravante chamado de TP )
com a seguinte disposicao :

Bi(0,0)
Bi(1,0),Bi(1,1)
Bi(2,0),Bi(2,1),Bi(2,2)
. . .

As colunas sao entendidas como numeradas da esquerda para a direita a
partir de zero. Verifique que se tomarmos 3 elementos consecutivos
Ai+1, Ai e Ai+1 da coluna 2 teremos :

Ai+1 - 2Ai + Ai-1 = 1^2 = 1

Exemplo : Bi(2,2) - 2Bi(3,2) + Bi(4,2) = 1 - 2*3 + 6 = 1

Se tomarmos 4 elementos consecutivos Ai+2, Ai+1, Ai e Ai-1 da coluna 3
termos que :

Ai+2 - 3Ai+1 + 3Ai _ Ai-1 = 1^3 = 1

E, de maneira geral, se tomarmos N+1 elementos consecutivos da coluna
N, usamos os
coeficientes numericos do desenvolvimento de (a+b) ^N, com os sinais
alternativamente trocados, teremos que a soma do tipo acima
exemplificada resulta em 1 :

( nao vou provar estas coisas aqui porque isso e muito mais coisas
deriva naturalmente das diversas faces ocultas do TP )

Esse numero 1, um valor constante em todo TP, sera chamado de NIC* (
ele e muito importante ). Assim, o TP e um triangulo aritmetico com
NIC = 1

O fato de no TP o NIC ser 1 esconde muitas coisas ... De fato, a
expressao geral para o NIN de um triangulo aritmetico e um polinomio
de coeficientes interessantes ( que depende das faces, conforme ja
falei ) na variavel NIC.

No TP, ao introduzirmos meios aritmeticos em todos o triangulo, vale
dizer, usar os coeficientes de (a+b) ^(N/2), N = ... -2,-1,0,1,2,...,
teremos que :

Ai+1 - 2Ai + Ai-1 = (1/2)^2 na coluna 2
Ai+2 - 3Ai+1 + 3 Ai - Ai-1 = (1/2)^3 na coluna 3
e assim sucessivamente

Se, no TP, introuzirmos os "termos aritmeticos, vale dizer, colocarmos
entre as linhas os coeficientes de (a+b)^(N/3), N = ... -2, -1, 0, 1,
2, ... teremos que :

Ai+1 - 2Ai + Ai-1 = (1/3)^2 na coluna 2
Ai+2 - 3Ai+1 + 3 Ai - Ai-1 = (1/3)^3 na coluna 3

Agora, va introduzindo meior, tercos, quartos etc aritmeticos no TP e
use este resultado para obter algo inedito, vale dizer, a expressao de

1 + (1/2)^3 + (1/3) ^3 + ...

Como uma soma de coeficiente binomiais ( conforme ja propus aqui ) sem
nenhuma NENHUMA potencia negativa.

Isso, que por si so e inedito e que levaria muito matematico correr
para publicar, e uma mera e simples aplicacao do "sonho de uma noite
de verao"

*O NIC e o nucleo dessa teoria. Nao e tao simples descobrir a
expressao dele. Um primeiro passo e descobri as faces ocultas de ja
falei. O Termo NIC deriva de NICOLAU SALDANHA. Eu descobri e
desenvolvi estas coisas pouco antes de ingressar nesta lista, ha cerca
de 10 anos atras. O Nicolau, alem de criar este maravilhoso espaco de
discussao, me recebeu ( como recebe a todos ) muito bem e foi o meu
modelo de Inicial de Matematico. Assim, nada mais justo que dar a um
elemento importante da minha teoria o nome do amigo e mestre que no
inicio me guiou e que desde sempre mereceu e merce a minha mais
elevada estima e consideracao.. Assim, e uma forma de um Matematico (
Eu, Paulo Santa Rita ) homenagear outro Matematico ( Nicolau Saldanha
).

Um Abraco a Todos
PSR, 10305091629







2009/5/3 Carlos Nehab :
> Oi, Paulo,
>
> Simplemente delicioso o texto e o conteúdo, mas... implore a sua esposa para
> não acordá-lo quando dormir sobre o teclado...
> Sonhe mais, por favor...
>
> Abraços,
> Nehab
>
> Paulo Santa Rita escreveu:
>
> Ola Benedito e demais
> colegas desta lista ... OBM-L,
> ( escreverei sem acentos )
>
>
>
>
> From: "benedito" 
> para  paulo.santar...@gmail.com
> data  2 de maio de 2009 09:16
> assunto   Re: [obm-l] Representacao do Multinomio de Leibniz
>   
>   
> Paulo,
>
> Desculpe-me a intimidade explícita na mensagem.
> Na verdade, estava passando esta beleza de raciocíonio para um Amigo, também
> Professor de >Matemática na minha cidade, que também se chama Paulo, para os
> íntimos Paulinho.
> Por engano, repassei a mensagem para obm-lista.
> Desculpe-me.
> Benedito
>
>
> Tudo bem, nao fiquei chateado.
>
> Voce gostou ? Vou supor que sim. Aperte os cintos porque vamos
> decolar. Vamos ver os elementos iniciais de "um sonho de uma noite de
> verao".
>
> Eu estava em casa. Eram cerca de duas horas da madrugada. Nao sei
> exatamente o dia, mas sei que   estava feliz, trabalhando no Maxima (
> http://maxima.sourceforge.net/ ) sobre o glorioso Debian/GNU Linux (
> http://www.br.debian.org/ ). Havia descoberto um fato interessante
> sobre sequencias de inteiros que sao expressas por duas ou mais
> sentencas, tais como a famosa sequ

[obm-l] Re: [obm-l] CEGUEIRA FRACIONÁRIA!

2009-05-03 Por tôpico Paulo Santa Rita
Ola Jorge, Albert e demais
colegas desta lista ... OBM-L,

Talvez esteja ocorrendo alguma confusao aqui ... E possivel fazer o seguinte :

1/3  > 1/4
1/4 >= 1/4
somando as duas desgigualdades  : 1/3 + 1/4 > 1/4 + 1/4 = 1/2

1/5 > 1/8
1/6 > 1/8
1/7 > 1/8
1/8 >= 1/8
somando as 4 desigualdades : 1/5 + 1/6 + 1/7 + 1/8 > 1/8 + 1/8 + 1/8 + 1/8 = 1/2

Prosseguindo assim vamos concluir que :

1 + 1/2 + 1/3 + ... + 1/n + ... > 1 + 1/2 + 1/2 + ...
Como a segunda serie, obviamente, diverge, pelo criterio de comparacao
segue que a primeira serie ( serie harmonica ) tambem diverge.

E entao jorge, e isso o que voce queria dizer ?

Eu conheco um problema sobre series, bonitinho e nao-trivial. Ele e assim :

Seja S=A1 + A2 + A3 + ... uma serie ( de numeros reias )
condicionalmente convergente. Sabemos, pelo teorema de Riemann, que
com um arranjo inteligente dos indices podemos fazer com que esta
serie convirja para um real "r" qualquer. Seja b:N -> N uma bijecao.
Caracterize as bijecoes b tais que Ab(1) + Ab(2) + Ab(3) + ...
converge

OBS : aqui, Ab(n) deve ser entendido com A con indice b(n)

Um Abracao a Todos
PSR, 10305090E24


2009/5/1 Jorge Luis Rodrigues e Silva Luis :
> Ok! Paulo e demais colegas! Em breve discutiremos matemática avançada com
> uma pitada de análise ou teoria da medida, mas vamos devagarinho, pois
> graças às discussões triviais aprendemos a prova da iguldade 0,999...=1, sem
> dúvida o tema mais discutido na lista. Afinal! Qual é maior: 1,001 ou 0,900?
> Há uma coisa que devemos aceitar como certa. Não nos sentimos à vontade
> lidando com frações. Dificilmente, um candidato olímpico saberia provar a
> desiguldade 1/2*3/4*5/6...99/100 < 1/1000? Outro pesadelo
> fracionário é pedir aos olímpicos para repartirem 9 maçãs entre 12 crianças,
> de modo que nenhuma maçã seja dividida em mais de 4 partes. Mas se acham que
> estou blefando, tentem apresentar aos alunos o conceito de fração imprópria:
> que sentido atribuir, por exemplo, à fração 5/2? Entre as frações 1/5 e 1/3
> temos 16 divisões iguais. Em qual das divisões se encontra a fração 1/4?
> (Campeã Olimpica!)
>
> A, B e C dividiram todo o conteúdo de uma garrafa de suco em três copos
> iguais, enchendo metade do copo de A, um terço do copo de B e um quarto do
> copo de C. Como cada um queria um copo cheio de suco, eles abriram outras
> garrafas iguais à primeira até encher completamente os copos. Quantas
> garrafas a mais eles tiveram que abrir? Se o suco de uma garrafa tivesse
> sido dividido igualmente entre eles, que fração de cada copo conteria suco?
>
> A propósito! Para obtermos 0, no visor da calculadora devemos
> efetuar 1/3*3 ou 3*1/3? (Essa é do colega Felipe Takiyama)
>
> Afinal! Como provar a desiguldade 1+1/2+1/3+1/4+...>1+1/2+1/2+1/2+...?
>
> Abraços!
>
> 
> Turbine seu Messenger com emoticons! Clique já, é GRÁTIS!

=
Instru��es para entrar na lista, sair da lista e usar a lista em
http://www.mat.puc-rio.br/~obmlistas/obm-l.html
=


Re: [obm-l] Representacao do Multinomio de Leibniz

2009-05-02 Por tôpico Paulo Santa Rita
binacoes nao seria capaz de expandir a representacao com a mesma
facilidade e uniformidade.

Com esta representacao, que no meu sonho era chamada de PIRAMIDE DE
LEIBNITZ, a expansao de (a+b+c)^N era um PIRAMIDE TRIANGULAR REGULAR,
na qual cada face era um UM PARTICULAR TRIANGULO DE PASCAL.  Assim,
por exemplo, para sabermos os coeficientes de (a+b+c)^3, bastava
seccionar a piramide de Leibniz pelo plano X+Y+Z=3. As interseccoes
assim obtidas eram simultaneamente os lugares e os valores dos
coeficientes dos monomios que constituem a expansao de (a+b+c)^3

Como exemplo, o ponto V=(1,1,1) e o "lugar" do monomio
(a^1)*(b^1)*(c^1) e o coeficiente  deste monomio e o numero [1,1,1] =
(1+1+1) ! / ( 1! * 1! * 1! ) = 6 . O ponto R=(2,0,1) e o "lugar" do
monomio (a^2)*(b^0)*(c^1)=(a^2)*(b^1) e o coeficiente  deste monomio e
o numero [2,0,1]=[2,1]=(2+1)! / (2! * 1!) = 3. E assim sucessivamente.

Para mim, neste ponto, ficou claro como representar a expansao de (X1
+ X2 + ... + Xm) ^N. Bastava, obvio, ir para o R^M e seccionar a
correspondente PIRAMIDE DE LEIBNIZ com o plano X+Y + ... ( M variaveis
) = N. As solucoes inteiras e nao-negativas desta equacao sao os
"lugares" e os valores dos coeficientes da expansao.

A VERDADEIRA RIQUEZA ESTA NO INTERIOR

Mas, no inicio do sonho, uma voz havia me dito que tudo aquilo era
para que eu pudesse ver as faces ocultas do traingulo de Pascal.
Lembrei-me disso, no sonho. E eis que aquela mesma voz fixou os meus
olhos num ponto V=(1,1,1), lugar de "abc" na expansao de (a+b+c)^3 e
no qual devemos por o coeficiente [1,1,1]=6.

Sob este ponto, ao aproximar os meus olhos, vi as letras
V.I.T.R.I.O.L. Minha formacao Iniciatica me permitil deduzir
imediatamente que aquilo era uma abreviacao para : "Visita Interiora
Terrae Retificando que  Invinies Occultum Lapidem" E, fiat luz, a luz
se fez. Numa fracao de segundos, tudo ficou claro. Entendi tudo :

O ponto V=(1,1,1) nao pertence a qualquer da faces da Piramide de
Leibniz, pois ele nao contem um ou mais zeros. Ao seccionar esta
piramide pelo plano X+Y+Z=4 com o objetivo de encontrar os lugares e
coeficientes de (a+b+c)^4, acharemos  os pontos I=(1,2,1) , T=(1,1,2)
e R=(2,1,1). Ora, os pontos V, I, e T definem UM PLANO PARALELO ao
plano YoZ, os pontos V, I e R definem UM PLANO PARALELO ao plano XoY
e, finalmente, os pontos V, T e R definem UM PALNO PARALELO ao plano
XoZ, ou seja, a partir de V=(1,1,1) surgira, no interior da piramide
de Leibniz, UMA NOVA PIRAMIDE CUJAS FACES SAO PARALELAS AS FACES DA
PIRAMIDE DE LEIBNIZ.

Ficou claro para mim naquele ponto do sonho que para todo N=3K,
K=1,2,3, ... a interseccao de X+Y+Z=N com a piramide de Leibniz faria
surgir um ponto (K,K,K) que daria origem a uma nova piramide, mais
interna, cujas faces seriam paralelas as faces das piramides mais
externas.

A intuicao infantil, o sonho, tinham confirmado as minhas suspeitas.
Eu tinha VISTO ou ADVINHADO as faces ocultas do triangulo de Pascal.
Agora era tudo simples e maravilhoso. Por que ? Por que apos ver o
objeto, bastaria eu caracterizar estas "folheacoes discretas" e
proceder como Arquimedes fez ...

( A area do circulo é pi*(r^2) = ( (2pi*r)*r ) /2 = area de um
triangulo retangulo que tem para altura o raio do circulo e para base
o comprimento da circunferencia => volume da esfera = volume de um
cone que tem para altura o raio e para base a area da esfera  =>
(4/3)pi*(r^3) = (1/3)r* S   => S=4pi*(r^2) )

Ou seja, na validade de uma relacao qualquer entre numeros binomiais
do triangulo de Pascal, supor que isto seja um caso particular das
piramides de Leibniz e extrapolar a relacao para os numeros
multinomiais quaisquer, dado a imensa simetria do objeto que
contemplamos.

Assim, o meu trabalho naquele momento era caracterizar as faces
ocultas e estudar como poderia aplicar suas propriedades, que esperava
serem notaveis, na solucao de problemas que sem tal visualizacao ficam
muito dificeis.

Mas, que balanco e esse ? Diante de tao maravilhosa visao algo me
arrancava brutalmente daquele paraiso. Seria Satanas, que nao gosta
que o homem aprecie as coisas de Deus ? ...
Que nada, era a minha mulher me acordando  e me tirando de cima do
teclado. Eu havia dormido no escritorio la de casa e, ja tendo
amanhecido, precisava me arrumar para ir trabalhar ...

Um Abracao a Todos !
PSR, 70205091830

2009/4/29 benedito :
> Show de bola, Paulinho.
> Benedito
>
>
> - Original Message - From: "Paulo Santa Rita"
> 
> To: 
> Sent: Wednesday, April 29, 2009 10:54 AM
> Subject: [obm-l] Representacao do Multinomio de Leibniz
>
>
>> Ola Pessoal,
>>
>> O Binomio de Newton e um assunto tipico da Matematica do ensino medio.
>> Ele da origem a questoes interessantes, algumas ja discutidas aqui
>> nesta. Um aspecto "curioso" deste tema
>> e que podemos olhar a expansao como disposta ao longo de uma reta,
>&

[obm-l] Re: [obm-l] demonstrações

2009-04-30 Por tôpico Paulo Santa Rita
Ola Johnson e demais colegas
desta lista ... )BM-L,

Em primeiro lugar, parabens pela sua escolha ! O livro ao qual voce se
refere, EM MINHA OPINIAO, e a melhor introducao a Analise produzida
por um brasileiro ... Prossiga nele. E no caso de alguma duvida,
publique-a aqui, pois nesta lista ha muitas pessoas que apreciam a
Analise Matematica.

Em segundo lugar, uma dificuldade inicial e absolutamente normal e um
caminho valido, mesmo louvavel, e justamente o pelo qual parece que
voce encetou, vale dizer, ler e reler tantas vezes quantas forem
necessarias ao um entendimento pleno. Tenha certeza que preguicoso nao
aprende matematica avancada ... e com o passar do tempo, voce
prosseguindo num estudo sistematico, tudo vai ficando mais familiar e
mais facil.

Mas e muito importante que apos uma leitura voce seja capaz de
responder a pergunta : exatamente e com precisao, o que eu nao estou
entendendo ? Nao adiantA ler, nao entender e nao ser capaz de
identificar QUE PARTE DA DEMONSTRACAO NAO ENTENDEU. Por exemplo, eu
quase apostaria que a dificuldade que voce sente na demonstracao
mencionada e que voce ainda nao percebeu que os conceitos de funcao
injetiva, bijetiva e inversa SAO  ANTERIORES a definicao de numero
natural.

Rigorosamente falando, nao existe "tecnicas de demonstracao". A
demonstracao e a maneira como nos provamos, de forma irrefutavel,
aquilo que vemos, independente dos meios pelos quais chegamos a ver
aquilo que desejamos demonstrar. O que existe e um "tipico modo de
pensar" relativo a uma area do conhecimento. Por exemplo : um Analista
costuma usar, muito, a ideia de LIMITE, pois a maioria dos conceitos
com que ele rotineiramente trabalha ( diferenciabilidade,
integrabilidade, etc ) foram, originalmente, definidas usando este
conceito; um algebrista, por outro lado,  vai procurar caracterizar as
coisa pela ideia de GRUPO, um conceito que na Algebra desempenha papel
equivalente ao de LIMITE em Analise. E assim sucessivamente.

EU ACREDITO  que e altamente saudavel a pessoa estudar ao menos dois
ramos "relativamente distantes", pois a grandes ideias costumam
surgir, justamente, da convergencia de mais de um ramo. No seu caso,
por exemplo, tente se concentrar em Algebra e Analise.

Em sintese, o que voce deve fazer e justamente o que esta fazendo e
propor aqui as demonstracoes que nao entendeu MENCIONANDO
EXPLICITAMENTE o ponto onde esta a sua duvida.

Um abraco
PSR,53004091230

2009/4/30 johnson nascimento :
> Olá amigos!
> Eu venho aqui na humildade pedir dicas de como melhorar minhas tecnicas em
> demonstrações. Sendo sincero com voces, tenho muita mais muita dificuldade
> em entender as demonstrações dos livros e se em cada demonstração eu parar
> quase 5,6 ou ate 15 dias para entender eu não termino o livro.
> Eu estou lendo o livro do Elon Lages um curso de analise. O fato é que eu ja
> compreendo tudo de forma intuitiva e sei fazer todos os exercicios mais de
> uns anos para ca eu estou interessado em adquirir tecnicas de demonstração
> pois para mim é o que tem de mais lindo na matematica.
>
> Ja conseguir entender como funciona o metodo axiomatico as custas de muito
> esforço já li livros de logica e tenho um livro da teoria dos conjuntos que
> ja li e é show mais, o fato é que não consigo entender as demonstrações de
> forma imediata.
> Eu agora estou tentando enteder a demonstração de um teorema no comecinho do
> livro 1 que fala sobre uma bijeção b: A -> I tal que I = {p pertence N : 1<
> p 
> O que devo fazer para realmente ficar bom nisso e não perder tanto tempo
> conseguindo vizualizar a demonstração quase que imediatamente?
> Obrigado
> 
> Veja quais são os assuntos do momento no Yahoo! + Buscados: Top 10 -
> Celebridades - Música - Esportes

=
Instru��es para entrar na lista, sair da lista e usar a lista em
http://www.mat.puc-rio.br/~obmlistas/obm-l.html
=


Re: [obm-l] Quanto Apostar ?

2009-04-29 Por tôpico Paulo Santa Rita
Ola Bernardo e demais colegas
desta lista ... OBM-L,

Parabens pela sua mensagem ! Muito boa ! Ela se coaduna perfeitamente
ao espirito original desta lista !

Quanto ao "B", eu usei "B^2" porque assim era no problema original :
eu apenas traduzi para uma linguagem elementar, adeguada para ser
publicada aqui. O fato de nunca haver encontro significa para mim que
as orbitas nunca atingem um ponto de equilibrio, mas isso e outra
historia ...

O caminho que eu segui foi diferente. Apos caracterizar - como voce
fez - os intervalos In eu passei a caracterizacao do formato das
moedas cuspidas. Descobri que a N-esima moeda cuspida tem o formato :

Mn = (Fn)^2  -  2*Fn*Fn+1*X + ((Fn+1)^2)*(X^2)

onde Fn e o N-esimo termo da sequencia de Fibonacci. Daqui,
considerando que nesta sequencia (F1)^2 + (F2)^2 + ... + (Fn)^2 =
(Fn)*(Fn+1), a soma M1 + ... + Mn  fica calculavel
e passivel de ser comparda com os valores presentes em In.

Vamos trabalhar para subir o nivel das discussoes.

Um abracao pra voce, tambem.
PSR, 42904091432

2009/4/29 Bernardo Freitas Paulo da Costa :
> 2009/4/28 Paulo Santa Rita :
>> Ola Bernardo e demais
>> colegas desta lista ... OBM-L,
>> ( escreverei sem acentos)
>>
>> Voce gostou do problema ? Que bom ! Fico contente por isso. Vou ficar
>> aguardando que voce publique aqui nesta nossa lista a sua solucao.
> Bom, aí vai uma idéia do problema. Está bem (beem) desorganizado,
> mas eu acho que é a melhor coisa a fazer para mostrar como a gente
> pode pensar no problema :)
>
> Comece vendo que, para a máquina te devolver alguma coisa, você tem
> que apostar suficientemente alto. Afinal de contas, se você der só
> 0.1, a máquina calcula B = 0.9 > X, e paf, você perdeu. Resultado,
> aposte pelo menos 0.5 + um pouquinho (repare que, da forma como o
> Paulo escreveu, dar 0.5 exatamente faz B = 0.5 que não é estritamente
> menor do que X = 0.5).
>
> O Paulo ajudou bastante escrevendo o algoritmo pra nós todos de forma
> extremamente clara, o que permite montar uma recorrência :
> a_{n+1} = x_n
> x_{n+1} = a_n - x_n
>
> e a coisa continua se b_n = a_n - x_n < x_n, ou seja, a_n < 2 * x_n
> (repare que aqui temos de novo o primeiro resultado do 0.5 !)
>
> Bom, recorrências de segunda ordem cheiram sempre a Fibonacci,
> principalmente quando os coeficientes são sempre 1 ou -1. Bom, daí eu
> montei a matriz de mudança de índices :
> 0 1
> 1 -1
> e calculei algumas potências:
> 1 -1
> -1 2
>
> -1 2
> 2 -3
>
> 2 -3
> -3 5
>
> Bom, aqui estava claro que a solução era uma sequência de Fibonacci,
> com sinais trocados. Um jeito de provar é fazer por indução, mas um
> modo muito mais legal é ver que essa sequência em questão também
> satisfaz uma recorrência de segunda ordem (multiplicar por um (-1)^n
> só muda o sinal das raízes !) e portanto, se duas recorrências de 2a
> ordem coincidem em dois termos, elas coincidem *sempre*. Ou seja,
> provamos sem precisar fazer contas. Legal. Os coeficientes da matriz
> de passagem para n termos adiante é, portanto :
> G_{n-1}  G_n
> G_n        G_{n+1}
> onde G_n = -(-1)^n F_n, F_n a sequência de Fibonnaci clássica F_0 = 0,
> F_1 = 1, F_2 = 1, F_3 = 2 e assim por diante, e a gente tem (por
> exemplo, pra verificar o (-1)^n) G_3 = 2 = F_3, portanto coincide para
> n ímpar, por isso o -(-1)^n.
>
>> Esse problema surgiu como uma questao secundaria na abordagem de um
>> tema bastante distante do tipo habitual de problemas tratados aqui. A
>> roupagem original, formal, era muito sisuda. Foi entao que eu lhe dei
>> esta apresentacao contextualizada em uma maquina de apostas.
>>
>> Vou falar um pouquinho sobre o problema :
>>
>> Seja 1, 1, 2, 3, 5, ..., Fn, ... a sequencia de Fibonacci. Para n par
>> considere o intervalo In=(Fn/Fn+1, Fn-1/Fn). Se n for impar, considere
>> In=(Fn-1/Fn,Fn/Fn+1). Usando as propriedades conhecidas desta
>> sequencia, e facil ver que I1 C I2 C I3 C ... C In C In+1 C ..., onde
>> "C" significa ESTA CONTIDO. Alem disso, e possivel provar o seguinte :
> Daí, eu pensei nos intervalos encaixados do Paulo (mas "a posteriori")
> e resolvendo a recorrência e impondo condição de continuar (a_n <
> 2x_n), a gente obtém
> (a_n, x_n) = (-1)^n (F_{n-1} - F_n * x , F_{n+1} x - F_n)
> logo
> (-1)^n (F_{n-1} - F_n * x ) < (-1)^n 2(F_{n+1}x - F_n)
> (-1)^n (F_{n-1} + 2 F_n) < (-1)^n (2F_{n+1} + F_n) x
> e lembrando da definição dos fibonacci :
> (-1)^n (F_n + F_{n+1}) < (-1)^n (F_{n+1} + F_{n+2}) x
> (-1)^n F_{n+2} < (-1)^n F_{n+3}x
>
> que dá, pra n=0, realmente 1 < 2x, e depois para n=1, 3x < 2. Ufa, deu
> certo! E ainda mais, coincide com o que faz o Paulo :
> para n par, é x > F_{n+2}/F_{n+3

[obm-l] Representacao do Multinomio de Leibniz

2009-04-29 Por tôpico Paulo Santa Rita
Ola Pessoal,

O Binomio de Newton e um assunto tipico da Matematica do ensino medio.
Ele da origem a questoes interessantes, algumas ja discutidas aqui
nesta. Um aspecto "curioso" deste tema
e que podemos olhar a expansao como disposta ao longo de uma reta,
numa ordem implicita. Assim :

(a+b)^N = a^n + N*(a^(n-1))*b + ... + N*a*(b^(n-1)) + b^n

E nos falamos com naturalidade no "primeiro termo da expansao", no
"segundo termo" e assim sucessivamente, firmando-nos nos expoente de
"b" ( ou de "a") que funcionam como um indice. Inclusive os livros
falam em algo como, "excontre o decimo termo da expansao de
(2x-3y)^15", implicitamente admitindo este tipo de ordenacao.

E na expancao, digamos, de um trinomio do tipo (2x-3y+y)^15 ? Quem e o
"decimo" termo ? Aqui, NA AUSENCIA DE UMA REPRESENTACAO CONSISTENTE,
uma tal questao e INDETERMINADA, pois precisamos acrescentar mais
algumas informacoes.

Seria possivel uma representacao consistente ? Uma maneira de olhar as
coisas que preservasse a visao habitual e lhe acrescentasse alguma
novidade ? Eu lembro que a ordem habitual no Binomio de Newton segue o
triangulo de Pascal ...

Bi(0,0)
Bi(1,0),Bi(1,1)
Bi(2,0),Bi(2,1),Bi(2,2)
...

Onde Bi(N,P)=N!/( (P!)*( (N-P)! ) )

Portanto, usando o triangulo de Pascal ( preservando sua principais
leis e propriedades ) e possivel encontrar uma representacao
consistente, um "lugar" onde colocar os termos da expansao de (X1 + X2
+  + Xm)^N ?

Note que uma tal construcao significaria, em parte ( existe uma outra
parte, mais dificil ), ver o famoso triangulo pascalino apenas como a
ponta de um iceberg, descortinando parte da superestrutura que lhe da
suporte ...

Entao : como e a parte imersa do iceberg ?

Um Abraco a Todos !
PSR, 42904091050
=
Instru��es para entrar na lista, sair da lista e usar a lista em
http://www.mat.puc-rio.br/~obmlistas/obm-l.html
=


[obm-l] Re: [obm-l] Re: [obm-l] [obm-l] Questões de Mat. B ásica

2009-04-29 Por tôpico Paulo Santa Rita
Ola Bruno e demais colegas
desta lista ... OBM-L,

A mensagem do Bruno e muito boa. Este espaco e uma LISTA DE DISCUSSAO
DE PROBLEMAS DE MATEMATICA OLIMPICA, nao e lugar para se propor
problemas de vestibulares ou concursos publicos. Digo isso, em
primeiro lugar, porque esse era o
objetivo original deste ambiente, conforme pode se ver na pagina da
OBM. Se o Prof
Nicolau nao alterou este objetivo, ele continua o mesmo ... Alem
disso, estudantes de
concursos e vestibulares tem inumeros outros espacos na Internet para
colocarem e discutirem seus problemas especificos, ao contrario dos
estudantes que se preparam
para Olimpiadas, com muito poucas opcoes.

Ha alguns anos, estudantes de olimpiadas de diversas partes do Mundo
assistiam as nossas discussoes. Eu receibia mensagens de alunos de
paises da America do Sul, dos EUA e da Europa interessados nos nossos
problemas, discussoes e solucoes. Me lembro que na traducao dos
problemas russos  :

http://www.mat.puc-rio.br/~nicolau/psr/

Eu precisei disponibilizar a traducao na pagina do Prof Nicolau,
tantos e tao diversificados eram os pedidos.

E o que estamos vendo agora ? A nossa tao estimada lista cheia de
problemas triviais, altamente distantes do ideal olimpico e verdadeira
fonte de solucoes para alunos preguicosos que nao querem pensar. Isso
afugenta os alunos serios, os Prof's competentes e muitas
outras pessoas que poderiam estar colocando aqui belas questoes e
belas solucoes, ajudando assim aquele nosso amigo de um estado
distante, que gostaria de se preparar para as
Olimpiadas de Matematica e que nao dispoe de locais de treinamento
proximo as suas casas.

A maneira mais sabia de combater estas coisas, eu penso, e nao
responder a estas questoes, desestimulando assim aqueles que estao,
conscientes ou nao, desvirtuando este espaco de seu belo ideal
original.

Um abraco a Todos
PSR, 42904090841

EM TEMPO : O Euler nos ensinou a calcular a soma dos inversos dos
quadrados dos numeros naturais. Nomeadamente ele mostrou que :

1 + (1/2)^2 + (1/3)^2 + ... = (pi)^2 /6

Mas tambem e verdade que ele tentou somar os inversos dos cubos dos
numeros naturais sem sucesso. Parece mesmo que esta soma ainda hoje e
um "problema em aberto". Pois bem.  Expresse

T = 1 + (1/2)^3 + (1/3)^3 + ...

Como uma soma de numeros binomiais na qual NENHUM dos numeros binomias
aparece em denominador ou elevado a potencias diferentes de 1.


2009/4/29 Bruno França dos Reis :
> Luciano, teoricamente esta lista tem por objetivo a discussão de problemas
> olímpicos (afinal de contas, veja o nome da lista), e não a resolução de
> lista de exercícios. Questões mais abertas, que exigem mais reflexão do que
> simplesmente cálculo bobo, são sempre bem recebidas pela maioria.
> Infelizmente, nos últimos tempos a lista tem se transformado nisso. Há
> pessoas que só fazem isso por aqui, colocam suas listas de exercícios para
> que os outros resolvam, sem nem sequer colocar uma mensagem (começa com a
> lista de exercícios e assina em baixo, com um apelido).
> Finalmente, muitos dos problemas colocados aqui já foram discutidos, e estão
> nos arquivos da lista. Claro que se o intuito for promover uma nova
> abordagem a um problema antigo, isso é fantástico. Agora, só pra saber a
> resposta, ou para discutir a mesma coisa, seria preferível consultar os
> arquivos.
> Bruno
>
> --
> Bruno FRANÇA DOS REIS
>
> msn: brunoreis...@hotmail.com
> skype: brunoreis666
> tel: +33 (0)6 28 43 42 16
>
> http://brunoreis.com
> http://blog.brunoreis.com
>
> GPG Key: http://brunoreis.com/bruno-public.key
>
> e^(pi*i)+1=0
>
>
> 2009/4/29 Luciano de Siqueira Pimentel 
>>
>> 1) Numa certa cidade, foi adotado o seguinte sistema de rodízio de carros:
>> duas vezes por semana, de segunda a sexta, cada carro fica proibido de
>> circular, de acordo com o final de sua placa (alg. das unidades). O número
>> médio de finais de placa proibidos diferentes para cada dia de proibição é:
>> A) 4
>> B) 1
>> C) 3
>> D) 2
>> E) indefinido
>>
>> 2) Um lojista sabe que, para não ter prejuízo, o preço de venda de seus
>> produtos deve ser no mínimo 44% superior ao preço de custo. Porém ele
>> prepara a tabela de preços de venda acrescentando 80% ao preço de custo,
>> porque sabe que o cliente gosta de obter desconto no momento da compra. Qual
>> o maior desconto que ele pode conceder ao cliente, sobre o preço de tabela,
>> de modo a não ter prejuízo?
>> A) 10%
>> B) 15%
>> C) 20%
>> D) 25%
>> E) 36%
>>
>> 3) O número de soluções reais da equação x^2 = 2^x é:
>> A) 0
>> B) 1
>> C) 2
>> D) 3
>> E) 4
>>
>> P.S.: Nessa questão aí eu só achei 2 soluções: x=2 ou x=4
>>
>> Gostaria de saber mais ou menos como funciona a lista. Estou me preparando
>> para o vest. do ITA, portanto gostaria de participar da lista mandando
>> outros tipos de exercícios (de Matemática, é claro). Eu poderia fazer isso
>> ou seria muito inconveniente?
>> Abraços!
>>
>>
>
>

=
Instru��es para entrar na list

Re: [obm-l] Quanto Apostar ?

2009-04-28 Por tôpico Paulo Santa Rita
Ola Bernardo e demais
colegas desta lista ... OBM-L,
( escreverei sem acentos)

Voce gostou do problema ? Que bom ! Fico contente por isso. Vou ficar
aguardando que voce publique aqui nesta nossa lista a sua solucao.

Esse problema surgiu como uma questao secundaria na abordagem de um
tema bastante distante do tipo habitual de problemas tratados aqui. A
roupagem original, formal, era muito sisuda. Foi entao que eu lhe dei
esta apresentacao contextualizada em uma maquina de apostas.

Vou falar um pouquinho sobre o problema :

Seja 1, 1, 2, 3, 5, ..., Fn, ... a sequencia de Fibonacci. Para n par
considere o intervalo In=(Fn/Fn+1, Fn-1/Fn). Se n for impar, considere
In=(Fn-1/Fn,Fn/Fn+1). Usando as propriedades conhecidas desta
sequencia, e facil ver que I1 C I2 C I3 C ... C In C In+1 C ..., onde
"C" significa ESTA CONTIDO. Alem disso, e possivel provar o seguinte :

Se X esta em In entao a maquima cospe ao menos N moedas

Assim, podemos obrigar a maquina a cuspir tantas moedas quanto
quisermos, bastando tomar um intervalo In suficientemente fino. Isto
significa que X vai ficando cada vez mais limitado e o numero de
moedas cuspidas vai crescendo ...

Um Abraco a Todos
PSR,32804091458

2009/4/28 Bernardo Freitas Paulo da Costa :
> Bom, eu fiquei contente de adivinhar qual era a magica do problema,
> mas fiquei encucado com a soluçao... porque eu acho que a sua maquina
> ganha sempre se for B^2... mas perde se for B. Seria legal que ela
> devolvesse phi^2 / (1 - phi) = 1 > phi ao todo, em vez de phi^4/(1 -
> phi^2) = phi^3 < phi ... Mas muito bom o problema (e mais uma
> excelente ocasiao de provar que um programinha pode ajudar a estudar
> uma conjectura :))
>
> 2009/4/28 Paulo Santa Rita :
>> Ola a todos !
>>
>> IMAGINEM um pais no qual para todo real X, 0 < X < 1, cunham moedas de
>> valor X. Neste pais ha uma maquina de apostas que opera recebendo, a
>> principio, uma moeda de valor X (a aposta) , podendo devolver zero,
>> uma ou diversas moedas, segundo o algoritmo :
>>
>> Passo 1) Faz A = 1
>> Passo 2) Calcula  B = A - X
>> Passo 3) Se B < X, faz :
>> * Entrega ao apostador (cospe) uma moeda valendo B^2
>> * Faz : A = X
>> * Faz : X = B
>> * Volta a executar o algoritmo a partir do passo 2
>> Senao ( Se B  >= X) , a maquina PARA.
>>
>> Para qual(is) valor(es) de X e vantajoso apostar  ?
>>
>> Um Abraco a Todos !
>> PSR, 22704092032
>> =
>> Instruções para entrar na lista, sair da lista e usar a lista em
>> http://www.mat.puc-rio.br/~obmlistas/obm-l.html
>> =
>>
>
>
>
> --
> Bernardo Freitas Paulo da Costa
>
> =
> Instruções para entrar na lista, sair da lista e usar a lista em
> http://www.mat.puc-rio.br/~obmlistas/obm-l.html
> =
>

=
Instru��es para entrar na lista, sair da lista e usar a lista em
http://www.mat.puc-rio.br/~obmlistas/obm-l.html
=


[obm-l] Quanto Apostar ?

2009-04-27 Por tôpico Paulo Santa Rita
Ola a todos !

IMAGINEM um pais no qual para todo real X, 0 < X < 1, cunham moedas de
valor X. Neste pais ha uma maquina de apostas que opera recebendo, a
principio, uma moeda de valor X (a aposta) , podendo devolver zero,
uma ou diversas moedas, segundo o algoritmo :

Passo 1) Faz A = 1
Passo 2) Calcula  B = A - X
Passo 3) Se B < X, faz :
* Entrega ao apostador (cospe) uma moeda valendo B^2
* Faz : A = X
* Faz : X = B
* Volta a executar o algoritmo a partir do passo 2
Senao ( Se B  >= X) , a maquina PARA.

Para qual(is) valor(es) de X e vantajoso apostar  ?

Um Abraco a Todos !
PSR, 22704092032
=
Instru��es para entrar na lista, sair da lista e usar a lista em
http://www.mat.puc-rio.br/~obmlistas/obm-l.html
=


Re: [obm-l] conjectura com numeros de Fibonacci

2009-04-21 Por tôpico Paulo Santa Rita
Ola carissimo Luis Lopes e demais
colegas desta lista ... OBM-L,
( escreverei sem acentos )

Vamos IMAGINAR que os possiveis totais acumulados apos o N-esimo lancamento
da moeda estao dispostos ao longo de uma coluna, numerados de cima para baixo,
de 1 ate 2^N. Represetaremos por Tn o total de resultados possiveis distintos;
por Tni um particular resultado.

Exemplos

1) coluna 1 : 0,1
T1=2, T11=0 e T12=1

2) coluna 2 : 0,1,0.5,2
T2=4, T21=0, T22=1, T23=0.5 e T24=2

3) coluna 3 : 0, 1, 0.5, 2, 0.25, 1.5, 1, 3
T3=7, T31=0,T32=1, T33=0.5, T34=2, T35=0.25, T36=1.5, T37=1 e T38=3

Note que estou convencionando que os Tni com i impar promanam da fortuna
acumulada anteriormente pela multiplicacao por 0.5; com i par, pela adicao
de 1.

Agora vou introduzir uma representacao para os possiveis totais acumulados

Representarei por "A" o ato de multiplicar por 0.5; O ato de somar 1 sera
representado por "B". Os exemplos anteriores ficarao assim :

Exemplos

1) coluna 1 : A,B
T1=2, T11=A e T12=B

2) coluna 2 : AA,AB,BA,BB
T2=4, T21=AA, T22=AB, T23=BA e T24=BB

3) coluna 3 : AAA,AAB,ABA,ABB,BAA,BAB,BBA,BBB
T3=7, T31=AAA,T32=AAB,...,T37=BBA e T38=BBB

E facilver que a N-esima coluna sera representada por TODOS os arranjos de
comprimento N que podemos fazer dispondo de 2 letras ... E igualmente
claro que metade dos arranjos da N-esima coluna iniciam com "A", a outra metade
iniciando com "B". Ora, e obvio que podemos descartar os A's iniciais
dos arranjos
iniciados com A, pois eles equivalem a multiplicar por 0.5 o valor inicial zero.

Exemplo

AABB = 0*(0.5)*(0.5) + 1 + 1 = 1 + 1 = BB
ABABB=(((0*(0.5)+1)*(0.5))+1)+1=1*(0.5)+1+1=BABB

Assim, se tomarmos todos os arranjos da coluna N iniciados com A ( a
metade superior
da coluna N ) e descartarmos o primeiro A de cada um, restara,
claramente, TODOS OS
ARRANJOS DA COLUNA n-1, cujo numero de elementos distintos ja
convencionamos designar
por Tn-1. portanto :

Tn = Tn-1 + VALORES INEDITOS DA METADE INFERIOR DA COLUNA N (1)

Os valores ineditos surgem na metade inferior da coluna N, sao todos
eles arranjos
iniciados com a letra B. Todavia, e claro que nem todo arranjo
iniciado com a letra
B representa um valor inetido. Por exemplo, na coluna 3 temos que T37=BBA=1 e um
valor que nao e inedito.

Vamos portanto introduzir um novo conceito. Para ver a motivacao para
ele considere
os exemplos abaixo :

Exemplos

1) BBA = (1+1)*(0.5)=2*(0.5)=1  => BBA=B
2) BBBA=(1+1+1)*(0.5) = 1*(0.5)+1 = 1.5 => BBBA=BAB
3) BABBA=((0.5)+2)*(0.5)=(0.25)+1 = 1.25  => BABBA=BAAB

Pensando um pouco sobre os exemplos acima e facil perceber que quando A ESQUERDA
DE ALGUM "A" HA DOIS OU MAIS "B" CONSECUTIVOS, o valor numerico
representado pelo
arranjo pode ser representado por um arranjo de comprimento menor. Isso motiva a
seguinte definicao :

DEFINICAO : Um arranjo e dito ser IRREDUTIVEL se a esquerda de qualquer de suas
letras A nao existe duas ou mais letras B consecutivas. Um arranjo que nao e
IRREDUTIVEL e dito ser REDUTIVEL

Os arranjos redutiveis, cujo valor numerico intrinseco pode ser
representado por um
arranjo de comprimento menor, nao nos interessam, pois, pela relacao
(1) deduzida acima,
eles ja foram computados. Interessa-nos os arranjos irredutiveis, pois
sao deles
que promanam valores ineditos. Ora, os arranjos irredutiveis iniciam sempre com
um unico B e a esqueda de qualquer A que porventura nele exista ha, no
maximo, um B.
Sao portanto exemplos de arranjos irretudiveis :

Exemplos :

1) BABABA, BAAA, BABAB, BAAABAABA, BB

Os arranjos irredutiveis permitem uma representacao mista interessante
: todos as
letras B que estao a direita da letra A mais a direita representam um
numero natural,
nomeadamente igual ao numero de letras B que lá existam; a parte
restante do arranjo
(a esquerda da letra A mais a direita, inclusive esta letra A )
representa um numero
binario decimal.

Exemplos

1) BABABBB = BABA + BBB = BABA + 3 = ((1*(0.5)+1)*0.5) + 3 = 3 + [0.11],
onde a parte entre colchetes e um numero real em sua representacao na base dois.

2) BAABA = (1*((0.5)^2)+1)*(0.5) = [0.101]

Assim, toda arranjo irredutivel R pode ser colocado na forma R = I +
[D], onde "I" e a
PARTE INTEIRA e "D" a PARTE DECIMAL - em base 2- da representacao
mista. Considerando
esta representacao mista, fica facil ver que :


LEMA 1 - Todo arranjo irredutivel representa um valor inedito, vale
dizer, um valor que
nao surgiu em qualquer das colunas anteriores

LEMA 2 - Dois arranjos irredutiveis distintos de mesmo comprimento
representam valores
distintos

Os fenomenos 1) e 2) acima nos permitem melhorar a relacao (1)
deduzida acima, colocando-a
como :

Tn = Tn-1 + TOTAL DE ARRANJOS IRREDUTIVEIS da coluna N (2)

Vejam que agora este despretencioso problema nos conduzia a
consideracao de um belo
problema de Analise Combinatoria, nomeadamente o calculo do numero de
arranjos irredutiveis
de comprimento N :

PROBLEMA : "Usando duas letras, quantos arranjos distintos - de
comprimento N - podemos
fa

[obm-l] RE: [obm-l] Números (em especial para o Ralph)

2009-04-16 Por tôpico Paulo Santa Rita
Ola Marcone e demais
colegas desta lista ... OBM-L,

Para quem quer partir do zero, o livro abaixo e interessante :

1) 
http://www.impa.br/opencms/pt/publicacoes/colecao_matematica_universitaria/livro_introducao_a_teoria_dos_numeros/index.html

Veja tambem :
2) http://www.mat.unb.br/~maierr/tnotas.pdf

O livro abaixo seria um curso mais avancado, para voce estudar quando
ja tiver aprendido as coisas basicas dos link's 1) ou 2) acima :

3) http://www.uv.es/ivorra/Libros/Numeros.pdf

Um abraco a todos
PSR, 51604090944


> From: owner-ob...@mat.puc-rio.br [mailto:owner-ob...@mat.puc-rio.br] On
> Behalf Of marcone augusto araújo borges
> Sent: Saturday, April 04, 2009 12:44 AM
> To: obm-l@mat.puc-rio.br
> Subject: [obm-l] RE: [obm-l] Números (em especial para o Ralph)
>
> Peço ao Albert ou outro interessado em teoria dos numeros  para me indicar
> livros acessíveis a um iniciante,escrito em potugues,sobre o assunto,que me
> é de grande interesse.Ficarei muito grato a quem praticar tal
> gentileza.Aguardo.Obrigado.

=
Instru��es para entrar na lista, sair da lista e usar a lista em
http://www.mat.puc-rio.br/~obmlistas/obm-l.html
=


[obm-l] Re: [obm-l] Problema de combinatória

2009-04-15 Por tôpico Paulo Santa Rita
Ola Tadeu e demais colegas
desta lista ... OBM-L,
( escreverei sem acentos )

O problema esta um pouco ambiguo. Vou dizer o motivo mais adiante.

Podemos escolher 5 cores de um total de 9 de Binom(9,5) maneiras.
Fixadas cinco cores, elas fornecem Binom(5,2) combinacoes de duas
cores. Podemos escolher 4 destas combinacoes de Binom(Binom(5,2),4)
maneiras. Portanto, a numero de maneiras de "pintar" o tetraedro sera
:

R=Binom(9,5)*Binom(Binom(5,2),4)

eu acho que o problema esta um tanto ambiguo porque nao sabemos, a
priori, se as bolinhas dos vertices sao indistinguiveis. Alem disso,
existe uma unica maneira de pintar uma bolinha com 1 cor, mas existem
diversas maneiras de pintar um bolinha com duas cores ...

Usando duas cores, de quantas maneiras seria possivel colorir uma
esfera de maneira que cada cor cor ocupasse CONTINUAMENTE 50% da
superficie esferica ?

um abraco a todos !
PSR,21304091120


2009/4/9 Walter Tadeu Nogueira da Silveira :
> Amigos,
>
> Uma professora disse que sonhou (veja só!) o seguinte problema.
>
> Suponha que nos vértices de um tetraedro haja uma bolinha que tenha que ser
> pintada de duas cores diferentes. Há 9 cores disponíveis, mas de cada vez
> são selecionadas cinco cores que tem que ser utilizadas no tetraedro. De
> quantas formas isso é possível?
>
> Que sonho!
>
> Abraços e Boa Páscoa!
>
> --
> Walter Tadeu Nogueira da Silveira
>
>

=
Instru��es para entrar na lista, sair da lista e usar a lista em
http://www.mat.puc-rio.br/~obmlistas/obm-l.html
=


Re: [obm-l] Fwd: Por favor me ajudem nessa

2009-04-15 Por tôpico Paulo Santa Rita
Ola Marcelo e demais colegas
desta lista ... OBM-L,

Voce deve estar fazendo alguma confusao. O problema ou e impossivel ou
e muito simples. Explico. Quando a reta "r" corta o angulo AOB nos
pontos C e D,  supondo que OC < OA e OD < OB teremos  que a igualdade
entre os angulos ( correspondentes )  OA e OD > OB entao a igualdade entre os angulos
 DC=DO. Ao
tracar as bissetrizes CE e OE teremos que  o triangulo
CEO e isosceles  =>  CE=OE. Por outro lado, os angulos :
>
> Honestamente, já tentei de tudo, acredito que haja algum teorema do baú da
> vovó, por favor me ajudem!E se ouver esse teorema, me digam qual é.
> Obrigado!
>
> Seja o ângulo AOB formado por duas semi-retas, tal que AOB é agudo. Traça-se
> uma reta r que intercepte o ângulo AOB nos pontos C e D respectivamente
> sobre as semi-retas AO e AB, de tal forma que 0 < AOB < ACD. Traça-se as
> bissetrizes de AOB e de ACD que se interceptarão num ponto E, e traça-se
> outro segmento, DE. O ângulo CDE será denominado Beta, e o seu adjacente,
> BDE, de Alfa. Pode-se afirmar que a relação entre Alfa e Beta é:
>
> Resposta: São iguais.
>
>
> --
> "Matemática é o alfabeto com o qual Deus escreveu o Universo"
> Galileu Galilei
>
>
>
> --
> "Matemática é o alfabeto com o qual Deus escreveu o Universo"
> Galileu Galilei
>

=
Instru��es para entrar na lista, sair da lista e usar a lista em
http://www.mat.puc-rio.br/~obmlistas/obm-l.html
=


Re: [obm-l] Combinatoria Pre-IME

2009-04-15 Por tôpico Paulo Santa Rita
Ola Silas e demais colegas
desta lista ... OBM-L,

O rapaz pode esclarecer as duvidas dele aqui conosco. Basta voce
inscrever ele na lista.

Quanto ao seu problema, eis aqui um raciocinio valido :

OBS : Nao e dito quantas cadeiras ha em uma fila ... vou supor que na
fila ha exatamente 7 cadeiras..  Se nao for assim, a solucao deve
tomar outro rumo.

Vamos IMAGINAR  que as cadeiras estao numeradas de 1 ate 7. Quais as
posicoes que as mocas podem ocupar ? As seguintes : (1,3, 5), (1,3,6),
(1, 3, 7), (1,4,6), (1,4,7), (1,5,7), (2,4,6), (2,,4,7), (2,5,7) e (
3,5,7). Escolhida uma destas 10 possibilidades, podemos permutar as
mocas de 3!=6. As quatro posicoes restantes podem ser ocupadas pelos
rapazes de 4!=24 modos. Assim, o total T  de maneiras de dispor estas
pessoas, atendidas as exigencias do problema, e :

T=10* 3! * 4! = 10*6*24 = 1440

Um abraco a todos !
PSR, 21304091802








2009/4/13 Silas Gruta :
> Boa tarde a todos,
>
> Tenho um aluno, cujo sonho é se formar pelo
> IME, extraordinariamente aplicado, uma verdadeira raridade numa escola
> pública! Faço o que posso para ajudá-lo, embora preparar alunos para o IME
> não seja, nem de perto, a minha especialidade. Bem, ele me apresentou um
> problema retirado de uma apostila de um curso Pré-IME/ITA de São Paulo, mas
> confesso que não estou conseguindo resolvê-lo mesmo depois de 13 dias de
> tetativas infrutíferas! Agradeço se puderem dar uma ajuda:
>
> Três moças e quatro rapazes estão num teatro e desejam, sentar-se, os sete,
> lado a lado, na mesma fila. Determine o número de maneiras pelas quais eles
> podem distribuir-se nos assentos de modo que:
>   a) duas moças nunca fiquem sentadas juntas; RESPOSTA: 1440
>   b) ...
> A pergunta (b) também é bem difícil, mas, se for o caso, apresento outro
> dia.
>
> Obrigado!
> --
> Silas Gruta

=
Instru��es para entrar na lista, sair da lista e usar a lista em
http://www.mat.puc-rio.br/~obmlistas/obm-l.html
=


[obm-l] Re: [obm-l] demonstração Geom Plana

2009-04-13 Por tôpico Paulo Santa Rita
Olá Thelio de demais
colegas desta lista ... OBM-L,
(escreverei sem acentos)

As medianas de um triangulo qualquer se encontram no centro de
gravidade do triangulo, tambem chamado de baricentro. Esse baricentro,
portanto, divide cada mediana em duas partes, a saber  : a primeira
parte, que vai do vertice onde se origina a mediana ate o baricentro,
a segunda, que vai do baricentro até o ponto medio do lado oposto. E
bem sabido que a primeira parte tem medida igual ao dobro da segunda
... Assim, chamando de "Ma" a medida da mediana que termina no ponto
medio do lado "a", segue que :

(2/3)*(Ma) + (2/3)*(Mb) > c( pela desigualdade triangular )
(2/3)*(Ma) + (2/3)*(Mc) > b( idem )
(2/3)*(Mb) + (2/3)*(Mc) > a( idem )

Somando tudo : Ma + Mb + Mc > (3/4)*(a+b+c)

Um abraco a todos
PSR, 21304091430

2009/3/13 Thelio Gama :
> Caros professores
> gostaria de uma ajuda na seguinte demonstração:
> "Mostre que a soma das três medianas de um triângulo é maior do que os 3/4
> do perímetro"
> Tentei resolver por desigualdade triangular, mas não consegui.
> Obrigado
> Thelio

=
Instru��es para entrar na lista, sair da lista e usar a lista em
http://www.mat.puc-rio.br/~obmlistas/obm-l.html
=


Re: [obm-l] Problema!!

2009-04-13 Por tôpico Paulo Santa Rita
Ola Antonio e demais
colegas desta lista ... OBM-L,
(escreverei sem acentos)

E bem conhecido que " um numero natural  pode ser escrito como soma de
dois quadrados se, e somente se, na sua decomposicao em fatores primos
os fatores da forma 4N+3 tenham expoente par ". Como 96=(2^5)*3, ve-se
que o fator primo 3 ( que e da forma 4N+3 ) nao tem expoente par.
Logo, o numero 96 nao pode ser representado como soma de dois
quadrados.

Este tema da representacao de numeros como soma de dois quadrados e
bem conhecido e, em geral,  abordado nos cursos iniciais sobre teoria
dos numeros.

um abraco a todos
PSR, 21304091042




Um n

2009/3/24 Antonio Manuel Castro del Rio :
> Ola, boa noite.
> Preciso de ajuda para resolver um problema.
>
>   COMO FAZER 96 VIRAR UMA SOMA DE DOIS QUADRADOS?
>
> Desde já, obrigado
>    Antonio del Rio
>
>

=
Instru��es para entrar na lista, sair da lista e usar a lista em
http://www.mat.puc-rio.br/~obmlistas/obm-l.html
=


[obm-l] Re: [obm-l] Re: [obm-l] Re: [obm-l] Uma difícil de Combinatória

2009-04-07 Por tôpico Paulo Santa Rita
Ola Marcelo e demais
colegas desta lista ... OBM-L,
( escreverei sem acentos )


Neste caso as caixas seriam distinguiveis. O raciocinio original que
voce empregou seria válido.

Um problema de alguma forma proximo ao que voce propos, porem nao tao
simples, pode ser formulado assim : IMAGINE 10 pequenas bolas, duas a
duas indistinguiveis. Dispondo de 4 cores e suponto que cada bola sera
pintada de uma unica cor, quantos colares distintos podemos fazer ?

SUGESTAO : IMAGINE uma pintura qualquer das bolas. Essa pintura
corresponde a uma solucao da equacao A+B+C+D=10. Todavia, com esta
particular pintura, em geral, sera possivel fazer diversos colares ...

Um Abraco a Todos  !
Paulo Santa Rita
30704091200


2009/4/7 Marcelo Gomes :
> Olá Professor Paulo, bom dia.
>
> Muito obrigado por sua preciosa explicação, entendi..bem a utilização da
> quádrupla para se obter as soluções não negativas, onde a ordem não é
> relevante (este foi o meu erro).
>
> Muito Obrigado, abração, Marcelo.
>
> Queria lhe perguntar uma outra dúvida.
>
> Se as caixas fossem numeradas de 1 a 10 ou em outras palavras, importa
> 2009/4/6 Paulo Santa Rita 
>>
>> Ola Marcelo e demais colegas
>> desta lista ... OBM-L,
>> ( escreverei sem acentos )
>>
>> Sejam :
>>
>> A -> caixas na cor azul
>> B -> caixas na cor amarelo
>> C -> caixas na cor verde
>> D -> caixas na cor vermelho.
>>
>> Uma solucao de A+B+C+D=10 na qual so figurem numeros inteiros
>> nao-negativos pode ser interpretada como uma maneira de pintar as
>> caixas. Assim, a 4-upla (A,B,C,D)=(3,2,0,5) significa que tres caixas
>> foram pintadas de azul, duas caixas foram pintadas de amarela, nenhuma
>> caixa foi pintada verde e cinco caixas foram pintadas de vermelho.
>>
>> O total de solucoes inteiras nao-negativas de A+B+C+D=10 nos da,
>> portanto, o numero de maneiras possiveis de pintarmos as 10 caixas com
>> as quatro cores disponiveis   - claro, supondo-se que duas caixas
>> ainda nao pintadas sao indistinguíveis !
>>
>> Isto posto, fica facil ver que considerando agora as solucoes de
>> A+B+C+D=10 nas quais A > 0 ( "A" e positivo ), vale dizer, todas as
>> solucoes de A+B+C+D=9, teremos todas as maneiras de pintar as caixas
>> nas quais  AO MENOS UMA CAIXA FOI PINTADAS DE AZUL.
>>
>> Existe um algoritmo direto, mesmo uma "formula", para o total de
>> solucoes inteiras e nao negativas de uma equacao diofantina da forma
>> X1 + ... + Xn = M, o que responde a sua questao. A formula e :
>>
>> Binom(N+M-1,M)
>>
>> No seu caso : N=4 e M=9. Logo : Binom(4+9-1,9)=220
>>
>> Um Abraco a Todos !
>> Paulo Santa Rita
>> 20604092020
>>
>>
>>
>>
>>
>>
>> 2009/4/6 Marcelo Gomes :
>> > Pessoal esta questão caiu em uma avaliação que fiz e o gabarito foi  bem
>> > diferente do que ei fiz. Por favor se alguém tiver um tempinho me dê uma
>> > mão, ok  ?
>> >
>> > Questão: Sejam 10 caixas de madeira, exatamente iguais. Queremos pintar
>> > cada
>> > uma delas com uma cor dentre quatro cores disponíveis: Azul, amarelo,
>> > verde
>> > e vermelho. De quantos modos podemos pintar as caixas, sabendo que pelo
>> > menos uma das caixas deve ser pintada de azul ?
>> >
>> > Minha resolução:
>> >
>> > Busquei encontrar o número ma´ximo de possibilidades para pintar as
>> > caixas.
>> > Então pensei da seguinte maneira: Na primeira caixa poderiam entra 4
>> > cores,
>> > e na segunda 4 e na terceira 4 e.assim até a décima caixa. Então o
>> > número máximo de possibilidades de se pintar as 10 caixas pela minha
>> > conta
>> > seria 4^10.
>> >
>> > Em seguida busquei encontrar o número de possibilidades onde a cor azul
>> > não
>> > aparecesse. Então analisei que na rimeira caixa poderiam entrar 3 cores,
>> > na
>> > segunda 3 cores, na terceira 3 corese na décima 3 cores. Então pela
>> > minha conta eu teria 3^10 onde o azul não aparece.
>> >
>> > Então como preciso ter pelo menos uma caixa azul, fiz:
>> >
>> > 4^10 - 3^10 = achei 989.527 maneiras
>> >
>> > Bem pessoal pelo gabarito eu errei e muito!  O gabarito deu 220
>> > modos.
>> >
>> > Não entendi nada!
>> >
>> > Peço que vocês me ajudem por favor, para compreender o enorme erro que
>> > fiz.
>> >
>> > Abração a todos.
>> >
>> > Marcelo.
>> >
>> >
>> >
>>
>> =
>> Instruções para entrar na lista, sair da lista e usar a lista em
>> http://www.mat.puc-rio.br/~obmlistas/obm-l.html
>> =
>
>

=
Instru��es para entrar na lista, sair da lista e usar a lista em
http://www.mat.puc-rio.br/~obmlistas/obm-l.html
=


[obm-l] Re: [obm-l] Uma difícil de Combinatória

2009-04-06 Por tôpico Paulo Santa Rita
Ola Marcelo e demais colegas
desta lista ... OBM-L,
( escreverei sem acentos )

Sejam :

A -> caixas na cor azul
B -> caixas na cor amarelo
C -> caixas na cor verde
D -> caixas na cor vermelho.

Uma solucao de A+B+C+D=10 na qual so figurem numeros inteiros
nao-negativos pode ser interpretada como uma maneira de pintar as
caixas. Assim, a 4-upla (A,B,C,D)=(3,2,0,5) significa que tres caixas
foram pintadas de azul, duas caixas foram pintadas de amarela, nenhuma
caixa foi pintada verde e cinco caixas foram pintadas de vermelho.

O total de solucoes inteiras nao-negativas de A+B+C+D=10 nos da,
portanto, o numero de maneiras possiveis de pintarmos as 10 caixas com
as quatro cores disponiveis   - claro, supondo-se que duas caixas
ainda nao pintadas sao indistinguíveis !

Isto posto, fica facil ver que considerando agora as solucoes de
A+B+C+D=10 nas quais A > 0 ( "A" e positivo ), vale dizer, todas as
solucoes de A+B+C+D=9, teremos todas as maneiras de pintar as caixas
nas quais  AO MENOS UMA CAIXA FOI PINTADAS DE AZUL.

Existe um algoritmo direto, mesmo uma "formula", para o total de
solucoes inteiras e nao negativas de uma equacao diofantina da forma
X1 + ... + Xn = M, o que responde a sua questao. A formula e :

Binom(N+M-1,M)

No seu caso : N=4 e M=9. Logo : Binom(4+9-1,9)=220

Um Abraco a Todos !
Paulo Santa Rita
20604092020






2009/4/6 Marcelo Gomes :
> Pessoal esta questão caiu em uma avaliação que fiz e o gabarito foi  bem
> diferente do que ei fiz. Por favor se alguém tiver um tempinho me dê uma
> mão, ok  ?
>
> Questão: Sejam 10 caixas de madeira, exatamente iguais. Queremos pintar cada
> uma delas com uma cor dentre quatro cores disponíveis: Azul, amarelo, verde
> e vermelho. De quantos modos podemos pintar as caixas, sabendo que pelo
> menos uma das caixas deve ser pintada de azul ?
>
> Minha resolução:
>
> Busquei encontrar o número ma´ximo de possibilidades para pintar as caixas.
> Então pensei da seguinte maneira: Na primeira caixa poderiam entra 4 cores,
> e na segunda 4 e na terceira 4 e.assim até a décima caixa. Então o
> número máximo de possibilidades de se pintar as 10 caixas pela minha conta
> seria 4^10.
>
> Em seguida busquei encontrar o número de possibilidades onde a cor azul não
> aparecesse. Então analisei que na rimeira caixa poderiam entrar 3 cores, na
> segunda 3 cores, na terceira 3 corese na décima 3 cores. Então pela
> minha conta eu teria 3^10 onde o azul não aparece.
>
> Então como preciso ter pelo menos uma caixa azul, fiz:
>
> 4^10 - 3^10 = achei 989.527 maneiras
>
> Bem pessoal pelo gabarito eu errei e muito!  O gabarito deu 220 modos.
>
> Não entendi nada!
>
> Peço que vocês me ajudem por favor, para compreender o enorme erro que fiz.
>
> Abração a todos.
>
> Marcelo.
>
>
>

=
Instru��es para entrar na lista, sair da lista e usar a lista em
http://www.mat.puc-rio.br/~obmlistas/obm-l.html
=


[obm-l] Re: [obm-l] livro de Análise V1 do Elon pagina 29

2009-04-04 Por tôpico Paulo Santa Rita
Ola Prof Roberto,
Alguem ( nao foi eu ) disponibilizou na internet as solucoes destesexercicios. 
Veja no link abaixo :
http://www.ebah.com.br/elon-capitulo-1-exercicios-resolvidos-por-paulo-santa-rita-zip-a4600.html
Um AbracaoPaulo Santa Rita

2009/4/2 prof.roberio :>> Questão 6) Se A, X está 
contido em E são tais que A ∩ X = Ø e A U X = E,> prove que X = Complementar de 
A.>> Questão 7 ) Se A está contido em B, então B ∩(A U C )  = (B∩C) U A para 
todo> conjunto c. Por outro lado, se existir C de modo que a igualdade acima 
seja> satisfeita, então A está contido em B.>> 
=> 
Instruções para entrar na lista, sair da lista e usar a lista em> 
http://www.mat.puc-rio.br/~obmlistas/obm-l.html> 
=
=
Instru��es para entrar na lista, sair da lista e usar a lista em
http://www.mat.puc-rio.br/~obmlistas/obm-l.html
=


Re: [obm-l] sequências Elon / Análise 1

2009-01-15 Por tôpico Paulo Santa Rita
1) E claro que para todo N temos que Xn =< Yn, pois a media geometricanunca e 
maior que a media aritmetica. Desta desigualdade pontualdecorre imediatamente o 
seguinte :
Xn+1 = (Xn*Yn)^(1/2)  >=  (Xn*Xn)^(1/2)=Xn  => (Xn) e uma 
sequencianao-decrescenteYn+1 =(Xn+Yn)/2  =<  (Yn+Yn)/2 = Yn  => (Yn) e uma 
sequencia nao-crescente
E tambem o seguinte :
Yn >= Xn para todo N, Xn >= X1 para todo N  => X1 =< (Yn) =< Y1 paratodo N => 
(Yn) e limitada.Xn <= Yn para todo N, Yn =< Y1 para todo N  => X1 =< (Xn) =< Y1 
paratodo N => (Xn) e limitada
E concluimos :
(Xn) e (Yn) sao monotonas e limitadas => (Xn) e (Yn) sao convergentes.
Sejam A = lim Xn= sup{X1, X2, ... } e B = lim Yn = inf{Y1, Y2, ...}
Nao pode ser B < A porque sendo B um infimo isto implicaria aexistencia de um 
Yp < A e a monotonicidade de (Yn) implicaria que Yn = p. Ora, 
tomando um E > 0 tal que A-E > Yp teriamosque Xn > Yp para todo N 
suficientemente grande => Xn > Yn para Nsuficientemente grande ... ABSURDO !
Nao pode ser B > A porque teriamos Yn+1 =(Xn+Yn)/2 >= B , para todo N=> Xn >= B 
+ (B-Yn), para todo N. Para N suficientemente grante temosque B-Yn tende a zero 
pois LIM Yn = B => para N suficientemente grandeXn > A ... ABSURDO !
Assim, nao podendo ser A < B ou A > B segue que A=B, como queriamos demonstrar !

2)  Sem perda de generalidade vou supor que a >= b > 0. Os detalhesdos casos em 
que a=0 ou/e b=0 sao triviais e fica como exercicio.
Xn=(a^n+b^n)^(1/n) = [a^n(1+(b/a)^n]^(1/n)=a[(1+(b/a)^(1/n))^(1/n)Lim Xn = 
a*LIM[(1+(b/a)^(1/n))^(1/n)
Note agora que a >= b   => (b/a) =< 1  => (b/a)^N <= 1 =>1+(b/a)^N =< 1+1   
=> 1  =< 1+(b/a)^N =< 2=>  1 =<[1+(b/a)^N]^(1/N) =< 2^(1/N)Aplicando o 
teorema do confronto ( teorema do sandwich ) temos que LIM [1+(b/a)^N]^(1/N) = 
1.  Logo :
LIm Xn= a*1 = a = max{a,b}
Fica com Deus !PSR, 51501092019

OBS : Da pra tornar mais claro os dois ultimos argumentos, sendotalvez mais 
prolixo. Isso fica como exercicio.2009/1/15 Murilo Krell 
:> prezados amigos da lista,>> Poderiam me ajudar com 
algumas questões de séries?>> 1) dados a,b pertencente a R+ defina 
indutivamente as sequências (xn)> e (yn) pondo x1=(a.b)^(1/2) e y1 = (a+b)/2 e 
xn+1=(xn.yn)^1/2 e yn+1=> (xn+yn)/2. Prove  que xn e yn convergem para o mesmo 
limite.>> 2) seja a >=0, b>=0, prove que lim(a^n + b^n)^(1/n) = max { a, b}>>> 
abs,> Murilo>> 
=> 
Instruções para entrar na lista, sair da lista e usar a lista em> 
http://www.mat.puc-rio.br/~obmlistas/obm-l.html> 
=>
=
Instru��es para entrar na lista, sair da lista e usar a lista em
http://www.mat.puc-rio.br/~obmlistas/obm-l.html
=


Re: [obm-l] dúvida simples - valor de aderência

2009-01-15 Por tôpico Paulo Santa Rita
Ola Carlos,
Ele nao deu DOIS termos : ele definiu uma sequencia DESTACANDO duas desuas 
subsequencias. A sequencia esta bem definida e comporta umainfinidade de 
subsequencias. Agora, no que concerne diretamente com aquestao, ha um resultado 
classico e basico da analise real que podeser enunciado da seguinte maneira :
"Se uma sequencia converge, entao TODAS as suas sub-sequencias tambemconvergem 
PARA O MESMO VALOR."
Entenda bem. Se uma seqquencia converge, entao 1) todas as suassubsequencia 
convergem 2) todas as subsequencias convergem para omesmo valor a que a 
sequencia converge.
Uma implicacao obvia e imediata e a seguinte : se sabemos que umasequencia 
converge, para sabermos para que valor ela converge bastacalcular o limite de 
qualquer uma de suas subsequencias.
Voltando ao seu problema, vemos que a subsequencia formada pelostermos impares 
diverge. Logo, a sequencia nao converge. A subsequenciaformada pelos termos 
pares converge para ZERO. Logo, zero e um valorde aderencia da sequencia. Eu 
afirmo que trata-se do UNICIO valor deaderencia. Para ver isso rapida e 
claramente, seja "r" # 0 um realqualquer
1) se r < 0 entao "r" nao pode ser valor de aderencia porque toda asequencia, 
por definicao, tem termos positivos e sabemos - por umaaplicacao direta do 
teorema da permanencia do sinal - que se umasequencia converge para um valor 
negativo, a partir de um certo pontotodos os seus termos devem ser negativos. 
Assim, nenhum r < 0 pode servalor de aderencia desta sequencia
2) se r > 0, tome E > 0 tal que r-E > 0. Seja N1 um natural tal quen>N1, X2n < 
r-E ( isto e possivel porque X2n -> 0 ) e seja N2 outronatural tal que n> N2, 
X2n-1 > r+E (isto e possivel porque X2n+1 tendeao infinito ). Para N3 = 
max{N1,N2} temos Xn nao esta em I=(r-E,r+E)para todo n > N3 ( pois os termos 
impares estarao a direita de "I" eos termos pares estarao a esquerda de "I" ). 
Isto mostra que apenas umnumero finito de termos esta neste invervalo I, vale 
dizer, "r" nao evalor de aderencia de Xn.
Note que estou aqui usando o fato de que se "r" e o valor de aderenciade uma 
sequencia (Xn) entao para todo E > 0 o intervalo (r-E,r+E)contem uma infinidade 
de termos da sequencia. Eu diria que esteresultado e obvio ululante, mas pode 
ser provado com rigor. Voce querfazer isso ?
Um AbracaoPSR, 51501091845



2009/1/15 Carlos Silva da Costa :> No livro do 
Elon (pequeno), tem uma questão assim:>> quais os valores de aderência da 
sequeência (xn) tal que x2n-1=n e x2n=1/n?> Está sequência converge?> o valor 
de aderência é zero, até ai tudo bem.>> Agora a sequência converge?,> qual é 
minha dúvida ele me deus dois termos dela, tal que x2n-1 -> oo  e x2n> vai para 
zero porém é divergente (harmonica), a análise que tem que ser> feita é essa 
mesma?>> []'s> Carlos
=
Instru��es para entrar na lista, sair da lista e usar a lista em
http://www.mat.puc-rio.br/~obmlistas/obm-l.html
=


Re: [obm-l] Re: [obm-l] avalição de resolução Análise

2009-01-13 Por tôpico Paulo Santa Rita
mpo eu participo mais
A todos, com os melhoresvotos de paz profunda,souPSR, 31301091812




G1(N) = N




2009/1/13 Marcelo Salhab Brogliato :> Olá Paulo,> por acaso 
esta "técnica" para montar a seqüência T é chamada de> Diagonalização, ou algo 
parecido?>> abraços,> Salhab>>>> 2009/1/13 Paulo Santa Rita 
>>>> Ola Murilo,>> Por que a sequencia g:N->N nao 
pertence a "lista (enumeracao)>> desequencias" ? Acho que faltou tornar isto 
MAIS CLARO. Alem disso,faltou>> enunciar claramente  que "suponhamos que as 
sequencias denumeros naturais>> seja enumeravel". Eis aqui uma demonstracao :>> 
Seja S o conjunto das sequencias de numeros naturais. SUPONHAMOS queeste>> 
conjunto seja enumeravel. Seja entao (s1, s2, ..., sn,...) umaenumeracao>> 
qualquer dos elementos deste conjunto. Vamos mostrar queexiste uma sequencia>> 
T de numeros naturais que nao esta na enumeracaoanterior :>> Facamos :>> 
T(1)=n1, tal que n1 # s1(1)T(2)=n2, tal que n2 # s2(2)...T(i)=ni, tal que>> n1 
# si(i)>> Assim definida, T e uma sequencia de numeros naturais e,>> po!
rtanto,necessita estar na enumeracao que fizemos, mas esta sequencia T>> 
naoesta na enumeracao pois ela e diferente de qualquer sequencia>> sn,n=1,2,... 
precisamente no ponto "n". o que e um absurdo, poisestavamos>> supondo que o 
conjunto S e enumeravel e que (s1, s2, ... )seria uma>> enumeracao dos seus 
elementos, abrigando portando TODAS assequencias de>> numeros naturais.>> 
Assim, a nossa tese e insustentavel e somos obrigados a admitir que>> oconjunto 
das sequencias de numeros naturais nao e enumeravel.>> EXERCICIO DE ANALISE : 
Mostre que QUALQUER CONJUNTO INFINITO pode>> serexpresso como uma uniao 
enumeravel de conjunto infinitos, dois a>> doisdisjuntos.>> Um AbracoPSR, 
31301090847>>>>>> 2009/1/12 Murilo Krell :> Pessoal,> 
continuando na>> labuta com a análise, fiz um exercício e queria colocar minha> 
resolução>> para um julgamento, acho que é a melhor forma de aprender.> (estou 
tentando>> deixar a construção de soluções e o formalismo a!
purado, por> favor,>> sugestões são muito bem vindas)>> Enu!
nciado: Prove que o conjunto das>> sequências de números natureais> (n1>>> resolução:>> Sendo X(N,N) o conjunto de todas as 
sequências crescentes de>> números> naturais. vamos mostrar que nenhuma função 
F; N-> X (N,N) pode ser>>> sobrejetiva.> Indicando por fm o valor de f no ponto 
m pertencente a N>>>> Isto significa que fm pertence a X(N,N), ou seja, é uma 
sequência crescente>>> de naturais. Assim, para cada n pertencente a N, fm(n) é 
um número>> natural.>> Temos:>> f1:= ( f1(1) < f1(2 )< f1(3) << f1(n) < ... 
) =>> F1(N)> f2:= ( f2(1) < f2(2 )< f2(3) << f2(n) < ... ) = F2(N)> .> .> 
fm:=>> ( fm(1) < fm(2 )< f!>> m(3) << fm(n) < ... ) = Fm(N)> .> .>> Agora, 
vamos "construir"uma>> sequência crescente g: N -> N que não esteja na> imagem 
de f.> Como N é>> infinito e ordenado, para n=1, coloque g(1) = f1(2) > f1(1)> 
No conjunto>> f2(N) coloque g(2) como sendo f2(1), ou seja, para g(n) vamos> 
tomar g(n) =>> fn(n-1)> assim f!
ormamos uma nova sequência g que não pertence a lista de>> sequências> fn.>> 
Assim nenhuma lista enumerável pode esgotar todas as>> funções em X (N,N)>> 
abraços e muito obrigado,> Murilo>>>> 
=>> 
Instru�ões para entrar na lista, sair da lista e usar a lista em>> 
http://www.mat.puc-rio.br/~obmlistas/obm-l.html>> 
=>>
=
Instru��es para entrar na lista, sair da lista e usar a lista em
http://www.mat.puc-rio.br/~obmlistas/obm-l.html
=


Re: [obm-l] avalição de resolução Análise

2009-01-13 Por tôpico Paulo Santa Rita
Ola Murilo,
No seu enunciado esta SEQUENCIAS DE NUMEROS NATURAIS e nao SEQUENCIASCRESCENTES 
DE NUMEROS NATURAIS. Mas se voce que que a sequenciaconstruida seja crescente e 
facil :
Faca t(1) # s1(1)para "i" natural tal que i > 1, faca t(i) = K, onde K e um 
natural talque K > { t(i-1), si(i) }. Isto garante que a sequencia T e 
crescentee que e diferente da sequencia Si precisamente no ponto "i" [pois 
t(i)> si(i) ], vale dizer, a sequencia T nao consta da enumeracao 
onde,erradamente, estamos supondo que estao TODAS as sequencias. Nistoconsiste 
o absurdo.
Este raciocinio e "conhecidissimo" e foi elaborado pela primeira vezpelo 
Cantor. E o "raciocinio diagonal" que o Cantor usou para provarque o conjunto 
dos numeros reais e nao-enumeravel.
Quanto a "construir explicitamente" nao e possivel, pois as enumracoessao 
arbitrarias e temos que raciocinar em cima DE QUALQUER ENUMERACAOQUE SEJA 
FEITA. Alem disso, aqui penetramos em uma discussao secular,vale dizer, se 
podemos ou nao podemos admitir em matematica objetoscuja construcao exige um 
numero de passos infinitos. Neste particulareu penso como o Hilbert : ninguem 
nos tirara do paraiso que Cantorcriou para nos !
Um AbracoPSR, 31301091116
2009/1/13 Murilo Krell :> Oi Paulo,>> muito obrigado 
pela solução,>> porém uma dúvida que eu fiquei é, não é preciso construir 
explicitamente a a> sequência que não vai constar na lista?,>> grande abraço e 
obrigado novamente,>> Murilo>> 2009/1/13 Paulo Santa Rita 
>>>> Ola Murilo,>> Por que a sequencia g:N->N nao 
pertence a "lista (enumeracao)>> desequencias" ? Acho que faltou tornar isto 
MAIS CLARO. Alem disso,faltou>> enunciar claramente  que "suponhamos que as 
sequencias denumeros naturais>> seja enumeravel". Eis aqui uma demonstracao :>> 
Seja S o conjunto das sequencias de numeros naturais. SUPONHAMOS queeste>> 
conjunto seja enumeravel. Seja entao (s1, s2, ..., sn,...) umaenumeracao>> 
qualquer dos elementos deste conjunto. Vamos mostrar queexiste uma sequencia>> 
T de numeros naturais que nao esta na enumeracaoanterior :>> Facamos :>> 
T(1)=n1, tal que n1 # s1(1)T(2)=n2, tal que n2 # s2(2)...T(i)=ni, tal que>> n1 !
# si(i)>> Assim definida, T e uma sequencia de numeros naturais e,>> 
portanto,necessita estar na enumeracao que fizemos, mas esta sequencia T>> 
naoesta na enumeracao pois ela e diferente de qualquer sequencia>> sn,n=1,2,... 
precisamente no ponto "n". o que e um absurdo, poisestavamos>> supondo que o 
conjunto S e enumeravel e que (s1, s2, ... )seria uma>> enumeracao dos seus 
elementos, abrigando portando TODAS assequencias de>> numeros naturais.>> 
Assim, a nossa tese e insustentavel e somos obrigados a admitir que>> oconjunto 
das sequencias de numeros naturais nao e enumeravel.>> EXERCICIO DE ANALISE : 
Mostre que QUALQUER CONJUNTO INFINITO pode>> serexpresso como uma uniao 
enumeravel de conjunto infinitos, dois a>> doisdisjuntos.>> Um AbracoPSR, 
31301090847>>>>>> 2009/1/12 Murilo Krell :> Pessoal,> 
continuando na>> labuta com a análise, fiz um exercício e queria colocar minha> 
resolução>> para um julgamento, acho que é a melhor forma de aprender.>!
 (estou tentando>> deixar a construção de soluções e o fo!
rmalismo apurado, por> favor,>> sugestões são muito bem vindas)>> Enunciado: 
Prove que o conjunto das>> sequências de números natureais> (n1>>> resolução:>> Sendo X(N,N) o conjunto de todas as sequências 
crescentes de>> números> naturais. vamos mostrar que nenhuma função F; N-> X 
(N,N) pode ser>>> sobrejetiva.> Indicando por fm o valor de f no ponto m 
pertencente a N>>>> Isto significa que fm pertence a X(N,N), ou seja, é uma 
sequência crescente>>> de naturais. Assim, para cada n pertencente a N, fm(n) é 
um número>> natural.>> Temos:>> f1:= ( f1(1) < f1(2 )< f1(3) << f1(n) < ... 
) =>> F1(N)> f2:= ( f2(1) < f2(2 )< f2(3) << f2(n) < ... ) = F2(N)> .> .> 
fm:=>> ( fm(1) < fm(2 )< f!>> m(3) << fm(n) < ... ) = Fm(N)> .> .>> Agora, 
vamos "construir"uma>> sequência crescente g: N -> N que não esteja na> imagem 
de f.> Como N é>> infinito e ordenado, para n=1, coloque g(1) = f1(2) > f1(1)> 
No conjunto>> f2(N) coloque g(2) como!
 sendo f2(1), ou seja, para g(n) vamos> tomar g(n) =>> fn(n-1)> assim formamos 
uma nova sequência g que não pertence a lista de>> sequências> fn.>> Assim 
nenhuma lista enumerável pode esgotar todas as>> funções em X (N,N)>> abraços e 
muito obrigado,> Murilo>>>> 
=>> 
Instru�ões para entrar na lista, sair da lista e usar a lista em>> 
http://www.mat.puc-rio.br/~obmlistas/obm-l.html>> 
=>>
=
Instru��es para entrar na lista, sair da lista e usar a lista em
http://www.mat.puc-rio.br/~obmlistas/obm-l.html
=


Re: [obm-l] avalição de resolução Análise

2009-01-13 Por tôpico Paulo Santa Rita
Ola Murilo,
Por que a sequencia g:N->N nao pertence a "lista (enumeracao) desequencias" ? 
Acho que faltou tornar isto MAIS CLARO. Alem disso,faltou enunciar claramente  
que "suponhamos que as sequencias denumeros naturais seja enumeravel". Eis aqui 
uma demonstracao :
Seja S o conjunto das sequencias de numeros naturais. SUPONHAMOS queeste 
conjunto seja enumeravel. Seja entao (s1, s2, ..., sn,...) umaenumeracao 
qualquer dos elementos deste conjunto. Vamos mostrar queexiste uma sequencia T 
de numeros naturais que nao esta na enumeracaoanterior :
Facamos :
T(1)=n1, tal que n1 # s1(1)T(2)=n2, tal que n2 # s2(2)...T(i)=ni, tal que n1 # 
si(i)
Assim definida, T e uma sequencia de numeros naturais e, portanto,necessita 
estar na enumeracao que fizemos, mas esta sequencia T naoesta na enumeracao 
pois ela e diferente de qualquer sequencia sn,n=1,2,... precisamente no ponto 
"n". o que e um absurdo, poisestavamos supondo que o conjunto S e enumeravel e 
que (s1, s2, ... )seria uma enumeracao dos seus elementos, abrigando portando 
TODAS assequencias de numeros naturais.
Assim, a nossa tese e insustentavel e somos obrigados a admitir que oconjunto 
das sequencias de numeros naturais nao e enumeravel.
EXERCICIO DE ANALISE : Mostre que QUALQUER CONJUNTO INFINITO pode serexpresso 
como uma uniao enumeravel de conjunto infinitos, dois a doisdisjuntos.
Um AbracoPSR, 31301090847


2009/1/12 Murilo Krell :> Pessoal,> continuando na 
labuta com a análise, fiz um exercício e queria colocar minha> resolução para 
um julgamento, acho que é a melhor forma de aprender.> (estou tentando deixar a 
construção de soluções e o formalismo apurado, por> favor, sugestões são muito 
bem vindas)>> Enunciado: Prove que o conjunto das sequências de números 
natureais> (n1> resolução:>> Sendo X(N,N) o conjunto 
de todas as sequências crescentes de números> naturais. vamos mostrar que 
nenhuma função F; N-> X (N,N) pode ser> sobrejetiva.> Indicando por fm o valor 
de f no ponto m pertencente a N>> Isto significa que fm pertence a X(N,N), ou 
seja, é uma sequência crescente> de naturais. Assim, para cada n pertencente a 
N, fm(n) é um número natural.>> Temos:>> f1:= ( f1(1) < f1(2 )< f1(3) << 
f1(n) < ... ) = F1(N)> f2:= ( f2(1) < f2(2 )< f2(3) << f2(n) < ... ) = 
F2(N)> .> .> fm:= ( fm(1) < fm(2 )< f!
m(3) << fm(n) < ... ) = Fm(N)> .> .>> Agora, vamos "construir"uma sequência 
crescente g: N -> N que não esteja na> imagem de f.> Como N é infinito e 
ordenado, para n=1, coloque g(1) = f1(2) > f1(1)> No conjunto f2(N) coloque 
g(2) como sendo f2(1), ou seja, para g(n) vamos> tomar g(n) = fn(n-1)> assim 
formamos uma nova sequência g que não pertence a lista de sequências> fn.>> 
Assim nenhuma lista enumerável pode esgotar todas as funções em X (N,N)>> 
abraços e muito obrigado,> Murilo>>
=
Instru��es para entrar na lista, sair da lista e usar a lista em
http://www.mat.puc-rio.br/~obmlistas/obm-l.html
=


Re: [obm-l] Arquivos fontes - Material do IME

2009-01-09 Por tôpico Paulo Santa Rita
Ola Sérgio,
Parabéns ! Este seu trabalho é digno de louvor em diversos sentidos.Tenha 
certeza que ele já é e será útil em diversos níveis e paradiversos tipos de 
estudantes. Se precisar de ajuda na solução dealguma questão, conte comigo.

Um AbraçãoPSR, 60901091709
2009/1/9 Sergio Lima Netto :>> Caros colegas da lista,>> 
Estou disponibilizando no site> www.lps.ufrj.br/~sergioln/ime> TODOS os 
arquivos-fonte de LaTeX> (.tex para os textos e .eps para as figuras)> do 
material com as provas de matematica do> vestibular do IME.>> O uso 
pessoal/individual deste material eh livre> (para edicao, correcao, 
complementacao, divulgacao etc.)> O uso institucional deve ser solicitado por 
email.>> Considero que este foi um grande projeto, feito com muito> carinho. 
Gostaria muito de ve-lo continuado,> expandido, melhorado, divulgado, ou seja, 
usado,> por varias outras pessoas.>> Abraco,> sergio>> 
=> 
Instruções para entrar na lista, sair da lista e usar a lista em> 
http://www.mat.puc-rio.br/~obmlistas/obm-l.html> 
=>
=
Instru��es para entrar na lista, sair da lista e usar a lista em
http://www.mat.puc-rio.br/~obmlistas/obm-l.html
=


Re: [obm-l] off topic: polinomio de taylor

2008-11-21 Por tôpico Paulo Santa Rita
Olá Hermann e demais
colegas desta lista ... OBM-L,
( escreverei sem acentos)

O livro "Um Curso de Calculo", Volume 1, do Prof Hamilton Guidorizzi,
tem um capitulo - o cap. 15 - inteiramente dedicado ao polinomio de
Taylor. EM MINHA OPINIAO, este livro e, "ao menos", um dos melhores
dentre todos os livros de calculo escritos por brasileiros ...
muitissimo melhor que algumas porcarias estrangeiras adotadas em
conhecidas e boas Universidades e Faculdades do Brasil. E e muito
dificil nao entender o que e o polinomio de Taylor estudando por ele,
pois o autor e altamente didatico.

Parece que foi ou ainda e adotado no ITA.

EM MINHA OPINIAO, se voce quer estudar calculo seriamente e criar
alicerces seguros para um posterior aprofundamento, vale a pena ter os
4 volumes e estudar por ele. ME PARECE que a sua fraqueza esta nos
exercicios, em pouca quantidade e triviais. Mas exercicios voce pega
em outros, ja classicos e bem conhecidos.

Um Abracao
PSR, 62111081917





2008/11/19 Hermann <[EMAIL PROTECTED]>:
> Boa noite, gostaria de falar sobre 3 assuntos:
>
> 1) Mais uma vez agradecer a todos que participam dessa lista, tirar dúvidas
> ou ler as dúvidas dos outros ensina bastante.
>
> 2) Estou achando que há algo de errado com o servidor pois não estou
> recebendo mensagens. (só consigo lendo no site)
>
> 3) Meu problema -
> (Preciso de ajuda para encontrar um texto (se possível em português) que
> explique Polinômios de Taylor. Nos livros que tenho de cálculo, o assunto é
> passado muito superficialmente.)
>
> Obrigado
> Hermann

=
Instruções para entrar na lista, sair da lista e usar a lista em
http://www.mat.puc-rio.br/~obmlistas/obm-l.html
=


Re: [obm-l] FATORIAL DE ZERO

2008-06-18 Por tôpico Paulo Santa Rita
Ola Jorge e demais colegas
desta lista ... OBM-L,

Nao ha o que justificar ...  0! = 1 e um POSTULADO : tao "POSTULADO"
quanto o quinto postulado de Euclides. E - assim como o famoso
postulado euclidiano tambem foi - ele e ainda hoje um dos alicerces da
nossa maneira de contar, pois, se o negarmos, as consequencias que dai
advem parecem nao corresponder com a realidade com que estamos
acostumados a lidar

Mas nada pode tolher a nossa liberdade de imaginacao.

Quando o Lobachevski negou o quinto postulado de Euclides e afirmou
que por um ponto fora de uma reta era possível traçar não uma, mas
várias retas paralelas a reta inicial dada, ele chamou os
desenvolvimento desta LOUCA HIPOTESE de GEOMETRIA IMAGINARIA
simplesmente porque achava que a realidade se conformava com a
geometria de Euclides, nao com a Geometria que ela estava descobrindo.
Entretanto, com o passar do tempo, ficamos sabendo que a realidade e
muito provavelmente NAO-EUCLIDIANA mais provavel que a realidade se

> Jorge Paulino wrote:
>  Provavelmente esse tópico já foi criado em algum
> momento. Mesmo assim, como sou novo por aqui, gostaria de alguma
> contribuição.
>
> Sem recorrer à função gama, usando como recurso
> apenas a interpretação através da problemas de contagem, como justificar que
> 0!=1??
>
> Eu conheço apenas a interpretação vinculada ao
> número de subconjuntos. Como Cn,p é igual ao número de subconjuntos de p
> elementos de um conjunto de n elementos,  então Cn,0 = 1 indica o número de 
> subconjuntos de 0 elementos, a saber, o
> vazio.
>
> Porém, se C8,3 indica o número de comissões
> de 3 pessoas num grupo de 8, como aceitar que o número de comissões de zero
> pessoas é igual C8,0=1?
>
> Se A5,3 fornece o número de senhas de 3 letras
> distintas a partir de um universo de 5, como aceitar que deste mesmo universo 
> é
> possível obter uma senha de zero letras, isto é, A5,0 = 1?
>
>  Grato,
>  Jorge

=
Instruções para entrar na lista, sair da lista e usar a lista em
http://www.mat.puc-rio.br/~obmlistas/obm-l.html
=


[obm-l] Exercicios de Analise 7

2008-04-25 Por tôpico Paulo Santa Rita
teremos :

| Xn+1 | =< (C^(N-N0) )*| Xn0+1| => | Xn+1 | =< (C^N )*( | Xn0+1| / (C^N0) )

Fazendo | Xn0+1| / (C^(N0-1) = K, temos que | Xn+1 | < K*(C^N )
No Livro em referencia o autor demonstra que LIM C^N = 0, quando C <
1. Assim, dado E
> 0 : Existe N1 tal que n>N1 => | C^N - 0 | < E / K. Daqui, tomando 
> N2=max{N0,N1} temos
que n>N2 => |Xn+1| < E => | Xn+1 - 0 | < E => LIM Xn=0.

***

De | X n+1 / Xn | >= C > 1 tiramos que | Xn+1 | >= C*|Xn| para todo
n>N0. Daqui segue
que Pi[ N0+1,N : |Xn+1| ] >= Pi[ N0+1,N : C*| Xn | ] para todo n>N0.
Eliminando os fatores
comuns aos dois lados da ultima desigualdade, teremos :

| Xn+1 | >= (C^(N-N0) )*| Xn0+1| => | Xn+1 | >= (C^N )*( | Xn0+1| / (C^N0) ).

Fazendo | Xn0+1| / (C^(N0+1) ) = K, temos que | Xn+1 | > K*(C^N )
No Livro em referencia o autor demonstra que LIM C^N = +INF, quando
C>1. Assim, dado
qualquer A > 0 : Existe N1 tal que n>N1 => C^N > A/K. Daqui, tomando
N2=max{N0,N1}
temos que n>N2 => |Xn+1| > A. => LIM | Xn | = +INF.

REOBTENDO O EXEMPLO 21

Seja Xn=(a^N) / N, onde a > 1. Entao : | Xn+1 / Xn | = a*(N / (N+1)).
Seja N0 o menor
natural tal que N0 > 1/(a-1). Segue :

N0>1/(a-1) => a(N0/(N0+1)) > 1. Fazendo C= a(N0/(N0+1)) teremos que N
> N0 implica
que N*N0 + N > N0*N + N0 => N*(N0+1) > N0(N+1) => (N/(N+1)) > (N0/(N0+1)) =>
(a*N/(N+1)) > (a*N0/(N0+1)) = C

Assim, para todo N > N0 => | Xn+1 / Xn | = a*(N / (N+1)) >
(a*N0/(N0+1)) = C > 1. Daqui
concluimos que LIM |Xn| = +INF. Como, neste caso particular, |Xn| =
Xn, seguira do
resultado demonstrado na solucao do exercicio 4.25 que LIM Xn=+INF.

REOBTENDO O EXEMPLO 22

Seja Xn=N! / (a^N), onde a > 0. Entao | Xn+1 / Xn | = (N+1) / a. Seja
N0 o menor natural
tal que N0 >= a. Entao (N0+1) / a > 1. Facamos C=(N0+1) / a. E claro
que se N > N0
entao (N+1)/a > (N0+1)/a=C. Portanto, N > N0 => | Xn+1 / Xn | =(N+1)/a
>(N0+1)/a=C>1.

Daqui concluimos que LIM |Xn| = +INF. Como, neste caso particular,
|Xn|=Xn, seguira do
resultado demonstrado na solucao do exercicio 4.25 que LIM Xn=+INF

LIMITE N! / (N^N) = 0

| Xn+1 / Xn | = ( (N+1)! / ( (N+1)^(N+1) ) )*( (N^N) / N! ) = (1 +
(1/N))^(-N). Portanto :
LIM | Xn+1 / Xn | = 1/e < 1/2. Seja E tal que (1/e) + E =1/2. Facamos C=1/2.

Como E > 0 existe um natural N0 tal que N > N0 => | |Xn+1/Xn| - (1/e)
| < E, vale dizer,
N>N0 => |Xn+1/Xn| < (1/e) + E = C = 1/2 < 1 . Pela demonstracao do
exercicio 4.25 isto
estabelece que LIM Xn =0, como queriamos demonstrar.

FIM

Um Abracao a Todos
Paulo Santa Rita
6,0D1A,190408

=
Instruções para entrar na lista, sair da lista e usar a lista em
http://www.mat.puc-rio.br/~obmlistas/obm-l.html
=


[obm-l] Exercicios de Analise 6

2008-04-18 Por tôpico Paulo Santa Rita
xn |, as propriedades dos
numeros reais nos
permitem escrever : Pi[1,P : |xi+2 - xi+1| ] =< Pi[1,P : c*|xi+1 - xi|
]. Daqui, apos
eliminarmos os fatores positivos comuns aos dois lados da
desigualdade, chegaremos a :

| xp+2 - xp+1 | =< (c^P)*| x2 - x1 |

Aplicando o somatorio, P variando de 1 a N -1 :

Sp[1,N-1:| xp+2 - xp+1 | ] =< Sp[1,N-1:(c^P)*| x2 - x1 | ] =>
|x2 - x1| + Sp[1,N-1:| xp+2 - xp+1 | ] =< |x2 - x1| + Sp[1,N-1:(c^P)*|
x2 - x1 | ]
vn =< (1+ c + c^2 +...+c^(N-1) )*|x2 - x1|

Fazendo d=|x2 - x1| e notando que 1+c+...+c^(N-1) e uma progressao geometrica de
razao 0 =< c < 1 e que portando 1+c+...+c^N < 1/(1-c) para todo N, teremos :
vn =< d / (1-c) para todo N => | vn | =< d/(1-c) => { vn } e limitada
=> { xn } e de
variacao limitada,

ITEM C

(IDA, =>) Seja { Xn } uma sequencia de variacao limitada e Vn como
definida acima, vale
dizer, Vn= Si[1,N:| Xi+1 - Xi | ]. Sabemos que { Vn } e limitada,
existindo portanto um L
real positivo tal que | Vn | =< L, para todo n. Isto posto, definimos :

Y1= X1 e Yn = Xn + Vn-1 se n > 1
Z1 =0 e Zn = Vn-1 se n > 1

Agora, e facil ver que :

1) Yn - Zn = Xn, para todo n
2) Zn e limitada porque, por hipotese, Vn e limitada. Yn tambem e
limitada por ser a soma
de duas sequencias limitadas : { Xn } e limitada porque, pelo ITEM A,
e convergente e
{Vn} e limitada porque, por hipotese, {Xn} e de variacao limitada.
Assim, tanto {Yn} quanto
{Zn} sao limitadas
3) Zn e nao-decrescente porque e soma de modulos de numeros reais. Quanto a Yn,
basta ver que : Yn+1 - Yn = (Xn+1 - Xn) + |Xn+1 - Xn|. Se Xn+1 >= Xn
entao teremos que
|Xn+1 - Xn| = Xn+1 - Xn => Yn+1 - Yn = 2(Xn+1 - Xn) >= 0. Se Xn+1 < Xn
entao teremos
que |Xn+1 - Xn| = Xn - Xn+1 => Yn+1 - Yn = 0 => Yn+1 = Yn. Portanto,
sob qualquer
hipotese, Yn+1 >= Yn => Yn e nao-decrescente.

Os itens 1), 2) e 3) estabelecem a implicacao direta.

(VOLTA, <=) Seja Xn = Yn - Zn onde {Yn} e {Zn} sao limitadas e nao-decrescente.
Queremos mostrar que { Xn } e de variacao limitada.

Seja Di = | Xi+1 - Xi |. Entao Vn = Si [1,N:Di ]. E facil ver o seguinte :

Di = |Yi+1 -Zi+1 - Yi + Zi| = |Yi+1 - Yi + Zi - Zi+1| =< |Yi+1 - Yi| +
|Zi - Zi+1|
Di =< |Yi+1 - Yi| + | Zi+1 - Zi| = (Yi+1 - Yi) + (Zi+1 - Zi)

Portanto ( agrupando os Y's e Z's do somatorio ) :

Vn = Si[1,N:Di] =< (Yn+1 - Y1) + (Zn+1 - Z1)
Como {Yn} e {Zn} sao limitadas, existem reais positivos L1 e L2 tais
que |Yn| =< L1 para
todo n e |Zn| =< L2. para todo n. Alem disso, as sequencias sao tambem
naodecrescentes,
vale dizer :

-L1 =< Y1 =< Y2 =< ... =< Yn =< ... =< L1 => Yn - Y1=< L1 -(-L1) =
2*L1, para todo n
-L2 =< Z1 =< Z2 =< ... =< Zn =< ... =< L2 => Zn - Z1 =< L2 -(-L2) =
2*L2, para todo n

Logo :

| Vn | =< 2*(L1+L2) para todo n => { Vn } e limitada => {Xn} e de
variacao limitada, como
queriamos demonstrar.

ITEM D

Seja r um real qualquer. Definimos : Xn = r + ( ( (-1)^N ) / N ). E
obvio ululante que esta
sequencia e convergente e LIM Xn = r. Entretanto :

Di = | Xi+1 - Xi | = | ( ( (-1)^(i+1) ) /(i+1) ) - ( ( (-1)^i ) / i )
| > 2/(i + 1)

Portando :

Vn = Si[1,N : Di ] > 2*( (1/2) + (1/3) + ... + (1/(N+1)) )

Como a serie harmonica e divergente, vale dizer, torna-se e se mantem maior que
qualquer grandeza dada, segue que Vn nao e limitada. Assim, {Xn} e um exemplo de
sequencia convergente que nao e de variacao limitada.


FIM


Um Abraco a Todos !
Paulo Santa Rita
6,0B21,120408

=
Instruções para entrar na lista, sair da lista e usar a lista em
http://www.mat.puc-rio.br/~obmlistas/obm-l.html
=


[obm-l] Exercicios de Analise 5

2008-04-08 Por tôpico Paulo Santa Rita
; Xp2 > Xp3 > ... > Xpi > ...

E portanto a sequencia Xn admite tambem uma sequencia monotona (
monotona decrescente ). Ora, P sera finito ou infinito, nao havendo
uma terceira possibilidade. Assim, todo sequencia admite uma
subsequencia monotona, como queriamos demonstrar.


( EXERCICIO 4.16 )

Precisamos mostrar que "a" e o limite de alguma subsequencia  de Xn.

Dado um natural qualquer M1.

Como LIM an = a, existe um natural N1 tal que n > N1 implica que | an
- a | < 1 / M1  => an E (a - (1/M1),a + (1/M1) ) = I1. Tomando um K1 >
N1 temos que ak1 E  I1  =>
a - (1/M1) <   ak1  < a + (1/M1).

Considerando que e facil ver que F1=min{ ak1 - (a - (1/M1) ), ( a +
(1/M1) ) - ak1 } e positivo e a menor das distancias de ak1 aos
extremos de I1, segue que (ak1 - F1, ak1 + F1 ) esta contido em I1.
Ora, sendo ak1  valor de aderencia de Xn e portanto existindo uma
subsequencia Xni de Xn que converge para ak1, correspondendo ao F1
fixado existe um Ni0 tal que todos os Ni > Ni0 da subsequencia Xni sao
tais que Xni E (ak1 - F1, ak1 + F1 )  =>  Xni E I1.

Tomando um Xni E I1, facamos Xp1=Xni. Seja M2 um natural tal que 1/M2
< | a - Xp1 |.

Como LIM an= a, existe um natural N2 tal que n > N2 => | an - a | < 1
/ M2 =>
an E (a - (1/M2),a + (1/M2) ) = I2. Tomando um K2 > N2 temos que ak2 E I2  =>
a - (1/M2) <   ak2  < a + (1/M2)

Considerando que e facil ver que F2=min{ ak2 - (a - (1/M2) ), ( a +
(1/M2) ) - ak2 } e positivo e a menor das distancias de ak2 aos
extremos de I2, segue que (ak2 - F2, ak2 + F2 ) esta contido em I2.
Ora, sendo ak2  valor de aderencia de Xn e portanto existindo uma
subsequencia Xni de Xn que converge para ak2, correspondendo ao F2
fixado existe um Ni0 tal que todos os Ni > Ni0 da subsequencia Xni sao
tais que Xni E (ak2 - F2, ak2 + F2 )  =>  Xni E I2.

Tomando um Xni E I2, facamos Xp2=Xni. Seja M3 um natural tal que 1/M3
< |a - Xp2|.

Repetindo o mesmo raciocinio desenvolvido antes, encontraremos um Xp3,
depois um Xp4 e assim sucessivamente. Mas o que e importante perceber
e que a construcao que fizemos e tal que a subsequencia Xpi cumpre :

(1/M1) > |a - Xp1 | > (1/M2) >  | a - Xp2 | >  ...  > (1/Mi) > | a -
Xpi | > ... ou seja :

| a - Xpi | < 1/Mi com Mi natural e (1/Mi+1)  < |a - Xpi|  para i = 1,
2, 3,  ...

Assim, dado E > 0, tomando um Mj da sequencia dos Mi tal que 1/Mj < E temos que
| a - Xpi | < (1/Mi) < E para todo i > j, vale dizer, a subsequencia
Xpi converge para "a", ou seja, "a" e valor de aderencia de Xn, como
queriamos demonstrar.


Um Abraco a Todos
Paulo Santa Rita
3,0D2C,080408

=
Instruções para entrar na lista, sair da lista e usar a lista em
http://www.mat.puc-rio.br/~obmlistas/obm-l.html
=


Re: [obm-l] lim (n --> oo) e^(-n) (1 + n + (n^2)/2!...+(n^n)/n!)

2008-04-08 Por tôpico Paulo Santa Rita
Ola carissimo Artur e demais
colegas desta lista ... OBM-L,

Artur, aqui vai uma ideia que passo pra voce analisar :

Seja Xn = 1 + N + ( (N^2)/2!) + ( (N^3)/3!) + ... + ((N^N)/N!). Entao
e^N = Xn + RL, onde
RL e o RESTO DE LAGRANGE. Segue daqui o seguinte :

Xn/(e^N) = 1 - ((RL)/(e^N))  => LIM Xn/(e^N) = 1 - LIM ((RL)/(e^N))

Um Estudo do LIM ((RL)/(e^N)) mais as propriedade de Y(X)=e^X resolve a questao.

EM TEMPO : Seria INTERESSANTE um estudo geral dos limites de expressoes como
esta, onde temos um quociente entre uma funcao, considerada somente em
seus valores
naturais, e a sua serie de Taylor, tambem considerada somente em
"pontos naturais".

Um Abracao a Todos
Paulo Santa Rita
3,080A,080408

2008/4/7 Artur Costa Steiner <[EMAIL PROTECTED]>:
> Gostei do argumento!
>  Vou pensar na questao do "meio da serie". De imediato, nao sei.
>  Abracos
>  Artur
>
>  -Mensagem original-
>  De: [EMAIL PROTECTED] [mailto:[EMAIL PROTECTED]
>  nome de Bernardo Freitas Paulo da Costa
>  Enviada em: sábado, 5 de abril de 2008 03:48
>  Para: obm-l@mat.puc-rio.br
>  Assunto: Re: [obm-l] lim (n --> oo) e^(-n) (1 + n +
>  (n^2)/2!...+(n^n)/n!)
>
>
>  O difícil desse argumento é a famosa "convergência uniforme". Eu acho
>  (como uma certa metade das pessoas que responderam aqui) que não está
>  certo, um pouco pelo fato de "parecer meio roubado" pegar o limite
>  assim. Usando umas coisas que eu escrevi na minha mensagem um pouco
>  acima (ou abaixo, dependendo do que você usa), note que os termos
>  "importantes" da soma estão "no meio" da série... e a gente truncou !
>  Vou tentar explicar (além do mais, acabei de ver que isso dá quase uma
>  prova de que a resposta é 1/2 ) :
>
>  Os termos a_k = n^k/k! (que é o que a gente soma) são crescentes até o
>  n^(n-1)/(n-1)! = n^n/n!. Antes, o quociente entre o a_(n-k) e
>  a_(n-k+1) é (n-k)/n = 1 - k/n. Depois, o quociente entre a_(n+k+1) e
>  a_(n+k) é n/(n+k+1) = 1 - (k+1)/(n+k+1).
>
>  Se a gente fosse arrumar isso num triângulo como na minha primeira
>  mensagem, depois de dividir por n^n/n!, fica :
>
>
>  (1 - 1/n )(1 - 2/n )(1 - 3/n)(1 - 4/n)...(1/n)
>  + ...
>  + (1 - 1/n )(1 - 2/n )(1 - 3/n)(1 - 4/n)
>  + (1 - 1/n )(1 - 2/n )(1 - 3/n)
>  + (1 - 1/n )(1 - 2/n )
>  + (1 - 1/n )
>  + 1 + 1 ( os "termos do meio" n^n/n! = n^(n-1)/(n-1)! )
>  + (1 - 1/(n+1) )
>  + (1 - 1/(n+1) )(1 - 2/(n+2) )
>  + (1 - 1/(n+1) )(1 - 2/(n+2) )(1 - 3/(n+3) )
>  + (1 - 1/(n+1) )(1 - 2/(n+2) )(1 - 3/(n+3) )(1 - 4/(n+4) )
>  + ...
>
>  Note que quando n -> inf, os termos de cada lado do 1+1 convergem pra
>  1 (de forma não uniforme) mas ainda mais, os "de baixo" convergem para
>  os de cima mais rápido ainda (não tenho muito tempo para detalhar
>  isso). Ou seja, se pararmos de somar no termo k = n^alpha com alpha
>  perto de 1 mas menor do que 1, temos uma convergência dos termos
>  "depois" aos termos "antes" do meio e com isso "dá pra ver" que na
>  verdade a gente só tem metade da série que realmente contribui para
>  e^n até os n primeiros termos (é aí que entra a tal história da
>  convergência uniforme, a gente precisa cada vez mais de termos para a
>  soma dar certo : com n fixo, a gente consegue majorar a soma por uma
>  PG de razão n/(n+1) a partir do primeiro termo "depois", mas veja que
>  tanto a razão quanto o termo que a gente majora tendem pro lugar
>  errado com n -> infinito). Faça as contas com n = 1,2,3, para ver como
>  os termos se comportam nessa série, é bem legal (e se você tiver maple
>  / mathematica / matlab / scilab / octave / ... veja com n = 20 e em
>  torno!)
>
>  Artur : você acha que dá pra tentar formalizar essa idéia do "meio da série" 
> ?
>  Rogério : passo a bola pra você me convencer !
>
>  Abraços,
>  --
>  Bernardo Freitas Paulo da Costa
>
>  2008/4/5 Rogerio Ponce <[EMAIL PROTECTED]>:
>  > Oi Marcelo,
>  >  quando voces quiserem repetir a dose, e' so' avisar - e juro que o
>  >  Nehab tambem vai :-)
>  >
>  >  Bem, voltando 'a vaca fria, vou explicar um pouquinho melhor o que eu 
> vejo.
>  >
>  >  A questao original e' calcular
>  >
>  >  lim (n --> oo) e^(-n) (1 + n +  (n^2)/2!...+(n^n)/n!)
>  >
>  >  Repare que o segundo fator corresponde aos "n" primeiros termos da
>  >  expansao de Taylor para "e^n".
>  >
>  >  O que eu sustento e' que, quando "n" vai para infinito, o segundo
>  >  fator passa a ser EXATAMENTE a expansao de Taylor para "e^n". E neste
>  >  ponto, tanto faz que o "n" desse tal fator

Re: [obm-l] Exercicios de Analise 4

2008-04-05 Por tôpico Paulo Santa Rita
Ola Marcelo e demais
colegas desta lista ... OBM-L,

Em primeiro lugar, obrigado : palavras de incentivo  nos motivam a prosseguir.

A ideia me parece muito boa por tres razoes :

1) Todas as solucoes ficariam armazenadas em um "mesmo lugar", o que
facilitaria consultas.
2) Outras pessoas poderiam aperfeicoar e/ou corrigir as provas
3) Esta nossa lista receberia o link a medida que eu fosse postando as solucoes.

Um problema que ocorre e que eu so posso ir colocando as solucoes na
medida em que vou arranjando tempo livre, o que significa que eu posso
ficar, as vezes, varias semanas sem publicar nada. Se estas ausencias
nao ijmplicarem algum problema maior, podemos tocar este projeto ( o
Arthur Steiner, que gosta muito de Analise, pode querer ajudar. E bom
perguntar a ele ), sem problemas

Eu nao tenho nenhum interesse financeiro, holofotes naome atraem e so
desejo contribuir para o progresso e elevacao desta maravilhosa
ciencia : sou um Matematico, do dedao do pe ate os cabelos da cabeca.

Alem disso, tenho uma divida de gratidao tanto para com o Nicolau, que
aqui me recebeu com lhaneza, respeito e dignidade bem como com outros
professores, com os quais aprendi muito (  Ralph, Gugu, Eduardo Wagner
e o falecido Morgado, so para citar alguns ). Assim, esta lista estara
sempre sob os meus cuidados, mesmo que muitas vezes, por falta de
tempo, eu nao possa dar a ela a devida atencao.

Um Abracao
Paulo santa Rita
7,0B23,050408



Muitos estudantes de outros paises da America do Sul e mesmo de outros
paises costumam nos acompanhar. Vi isso quando traduzi os problemas
russos e recebia em off muitos pedidos.



2008/4/4 Marcelo Salhab Brogliato <[EMAIL PROTECTED]>:
> Olá Paulo,
>
> gostaria de parabenizá-lo pelas soluções. Tem o interesse de postar estas
> soluções diretamente em uma wiki?
> Você utilizaria os benefícios do Latex, de forma prática e que fica
> disponível para toda a comunidade, não sendo necessário procurar nos
> arquivos da lista. E para enviar para a lista, basta postar o link. ;)
>
> Pensei em criarmos alguma coisa assim:
> == Análise na reta - Elon ==
>
> * Capítulo 1
> ** Exercício 1
> ** Exercício 2
> ** :
> * Capítulo 2
> ** ...
>
> e assim por diante.
> Se quiser, crio para você e mando o link por pvt.
>
> Um grande abraço,
> Salhab
>
>
> 2008/4/3 Paulo Santa Rita <[EMAIL PROTECTED]>:
> >
> >
> >
> > Ola Pessoal,
> >
> > Tenho publicado algumas solucoes de exercicios de Analise retirados do
> > excelente Livro :
> >
> > Curso de Analise - Volume 1 - Projeto Euclides - IMPA
> > 11 edicao - 2 impressao
> > Autor : Elon Lages Lima
> >
> > Eu nao publiquei nenhuma solucao dos capitulos 1, 2 e 3 porque sao
> > assuntos que, em geral, o estudante ja viu em cursos anteriores,
> > principalmente na Graduacao.  Alem disso, eles sao bastante simples.
> > Entretanto, algumas pessoas  me escreveram "em off" e pediram que eu
> > publicasse 2 ou 3 solucoes desses capitulos. Assim, selecionei 3
> > exercicios do capitulo 1 e estou publicando agora. Escolhi os que
> > achei mais interessantes ou/e desafiadore. Seguem as solucoes :
> >
> >
> >
> >
> > NOTACAO : A letra "lambda" sera representada nestes exercicios por
> > "m". Os simbolos de uniao e intersecao serao representados
> > respectivamente pelos prefixos UNI e INTER. Os simbolos ">" e "<"
> > representarao, respectivamente, "contem" e "esta contido". O Simbolo
> > de "pertence a" sera representado pela letra "E" e "f_a" representa a
> > letra "f" com indice "a". A barra "/" representara a expressao "tal
> > que"
> >
> >
> >
> > ( EXERCICIO 1.14)
> >
> > NOTACAO : Seja f : A -> B uma funcao. Se Y < B, f(-1)(Y) sera o
> > conjunto de todos os elementos x E A tais que f(x) E Y. No caso de Y
> > ser unitario, tal como em Y={b}, f(-1)(Y) sera representado por
> > f(-1)(b)
> >
> > ITEM A :
> >
> > Seja X < A, a E X. Existe b E f(X) / b = f(a)  =>  a E f(-1)(b)  => a
> > E f(-1)( f(a) ). Como f(a) = b E f(X) => f(-1)( f(X) ) > f(-1)(f(a))
> > => a E f(-1)( f(X) ). Assim, a E X => a E f(-1)( f(X) ). Isto
> > estabelece que X < f(-1)(f(X)), qualquer que seja o X < A, tal como
> > queriamos demonstrar.
> >
> > ADVERTENCIA : No meu livro o enunciado esta errado, pois la pede-se
> > para demonstrar que f(-1)(f(X)) < X para todo X < A. Isto e
> > evidentemente impossivel. Basta considerar a funcao f:{1,2,3} ->{4,5,6
> > } tal que f(1)=f(2)=f(3)=4. Tomado X={1,2} temos 

Re: [obm-l] Inteiros!!!

2008-04-04 Por tôpico Paulo Santa Rita
Ola Pedro e demais
colegas desta lista ... OBM-L,

Obviamente que todo par (X,Y) da forma (X,-X) e solucao, pois :
X^3 + (-X)^3 = 0 = (X+(-X))/2.  Em particular, (0,0) e solucao.

Se, porem, X+Y # 0, teremos :
X^3 + Y^3 = (X+Y)*(X^2 -XY + Y^2) = (X+Y)/2. => X^2 - XY + Y^2 = 1/2
=> (X-Y)^2 = - (X^2 +Y^2) .
A possibilidade aqui, logicamente, e :  X-Y=0 e X^2+Y^2 = 0. Mas isso
da (X,Y)=(0,0)
o que contraria a hipotese X+Y # 0

Assim, todas as solucoes inteiras sao {(X,-X) / X e inteiro }

Um Abracao a Todos
Paulo Santa Rita
6,0A2D,040408

2008/4/4 Pedro Júnior <[EMAIL PROTECTED]>:
> 02. Ache todos os pares tais de números inteiros (x, y) tais que:
> x^3 + y^3 = (x + y)^2

=
Instruções para entrar na lista, sair da lista e usar a lista em
http://www.mat.puc-rio.br/~obmlistas/obm-l.html
=


Re: [obm-l] Exercicios de Analise 4

2008-04-04 Por tôpico Paulo Santa Rita
Ola Joao !

Tranquilo. Fique a vontade : o meu interesse e que aqui nesta lista
seja praticado Matematica de Qualidade. Assim, um bom exemplo e fazer
as questoes do Projeto Euclides, IMPA. Se achar valido, apenas cite
que trata-se de solucao de um membro da LISTA DE DISCUSSAO DE
PROBLEMAS DE MATEMATICA OLIMPICA da PUC-RIO.

Um Abracao
Paulo Santa Rita
6,0911,040408

2008/4/4 João Luís <[EMAIL PROTECTED]>:
>
>  Paulo,
>
>  Ja que eh assim, resolvi escrever entao pra engrossar o coro daqueles que
> acham otima sua iniciativa. Eu tambem tenho interesse pelas solucoes.
>
>  Estou pensando ateh em, quando eu tiver um tempinho sobrando, fazer um
> arquivo Latex com elas. Voce permite?
>
>  Um abraco,
>
>  Joao Luis
>
>  - Original Message - From: "Paulo Santa Rita"
> <[EMAIL PROTECTED]>
>  To: 
>  Sent: Friday, April 04, 2008 6:46 AM
>  Subject: Re: [obm-l] Exercicios de Analise 4
>
>
>
>
>  Oi Claudio !
>
>  Nao ha do que agradecer, mas a sua iniciativa nos motiva a prosseguir
>  : as vezes passa pela minha cabeca que ninguem se interessa por estas
>  solucoes e que estou apenas "enchendo o saco" dos membros da lista.
>  Quando algumas pessoas, como  voce fez, se manifesta, nos vemos que o
>  nosso trabalho esta sendo util e a duvida se dissipa.
>
>  A Matematica e Universal e patrimonio de toda a Humanidade. Assim, EU
>  PENSO que, sempre que possivel, devemos divulgar livremente aquilo que
>  sabemos, sobretudo quando trata-se de solucoes de um tema CUJOS
>  FUNDAMENTO ja e amplamente dominado. Estas solucoes ajudam os
>  estudantes a descobrirem MODELOS DE ATAQUE a outros problemas
>  similares.
>
>  Para que esta mensagem nao seja inteiramente pessoal, aqui vai a
>  solucao do problema 4.12
>
>  ( EXERCICIO 4.12 )
>
>  Doravante, sempre que precisar usar somatorios, vou adotar a notacao :
>
>  Si[1,N : F(i)] = F(1) + F(2) + ... + F(N)
>
>  Seja r um numero real,  0 < r < 1. Fazendo b=r*a, temos que 0 < b < a.
>  Ao real F = a - b > 0 correspondera um N0 tal que n > N0 implica | Xn
>  – a | < F, pois LIM Xn= a. Daqui seguira que para n > N0, a – F < Xn,
>  vale dizer, n> N0 => b < Xn. Agora, usando as propriedades dos numeros
>  reais, e facil ver que para quaisquer naturais POSITIVOS "i" e K ( n >
>  N0 ) :
>
>  0 < b^(i/K) < (Xn)^(i/K)  =>   0 < (b^(i/K))*(a^(K-i-1) ) < (
>  (Xn)^(i/K) )*(a^(K-i-1) )
>
>  Logo : 0 < Si[0,K- 1:(b^(i/K))*(a^(K-i-1) )]  <
>  Si[0,K-1:((Xn)^(i/K))*(a^(K-i-1) )]   =>
>  0 < | Si[0,K- 1:(b^(i/K))*(a^(K-i-1) )]  |  <  |
>  Si[0,K-1:((Xn)^(i/K))*(a^(K-i-1) )]  |
>
>  Fazendo c = | Si[0,K- 1:(b^(i/K))*(a^(K-i-1) )]  | e multiplicando
>  tudo por |(Xn)^(1/K)–a^(1/K)|, segue :
>
>  ( DESIGUALDADE 1 )
>  |(Xn)^(1/K)–a^(1/K)|*c < | (Xn)^(1/K)–a^(1/K) |*|
>  Si[0,K-1:((Xn)^(i/K))*(a^(K-i-1) )] |=|Xn-a|
>
>  Dado um E > 0
>
>  ( DESIGUALDADE 2 ) :
>  Como LIM Xn=a, existe um N1 tal que n > N1 => | Xn-a| < c*E
>
>  Vemos portanto que se tomarmos um N2=max{N0,N1}, para todo n > N2 as
>  duas desigualdades ficarao satisfeitas. Usando a transitividade das
>  desigualdades chegamos a :
>
>  n > N2 => |(Xn)^(1/K)–a^(1/K)| < E
>
>  Assim, para um E > 0 qualquer sempre podemos exibir um natural N2 tal
>  que n > N2 implica que |(Xn)^(1/K)–a^(1/K)| < E. Isto estabelece que
>  LIM (Xn)^(1/K) = a^(1/K) , como queriamos demonstrar.
>
>  ***
>
>  Seja LIM Xn=a  e  r=P/Q. Já sabemos que  LIM (Xn)^(1/Q) = a^(1/Q). Mas :
>  LIM (Xn)^(P/Q) = LIM [ (Xn)^(1/Q)*...*(Xn)^(1/Q) ] onde dentro dos
>  colchetes há P fatores. Logo :
>  LIM (Xn)^(P/Q) = LIM (Xn)^(1/Q)*LIM (Xn)^(1/Q)*...*LIM (Xn)^(1/Q)
>  LIM (Xn)^(P/Q) = a^(1/Q)*a^(1/Q)* ... *a^(1/Q) = a^(P/Q)
>
>  O caso P/Q < 0 vai seguir a mesma linha, bastando eliminar o sinal de "-"
>
>  OBS : Quem se interessar pelos exercicios dos capitulos 1 e 2 eu
>  enviei uma copia do arquivo com todas as solucoes para o "Tio Cabri".
>  Ele zipou o arquivo e colocou num ciber-lugar. E so falar com ele. Eu
>  tenho as solucoes de todas as questoes, conforme for encontrando, vou
>  disponibilizando pra voces. Enquanto isso vou publicando as solucoes
>  aqui. Sejam magnanimos e mostrem e expliquem as solucoes para os seus
>  colegas.
>
>  Um Abracao a Todos
>  Paulo Santa Rita
>  6,0630,040408
>
>
>
>  2008/4/3 Claudio Gustavo <[EMAIL PROTECTED]>:
>
> >  Oi Paulo.
> >  Estou respondendo essa mensagem apenas pra agradecer sua iniciativa. Pois
> > essas soluções tenho certeza que ajudarão a muitos outros além de mim.
> >
> >  Abraço

Re: [obm-l] Exercicios de Analise 4

2008-04-04 Por tôpico Paulo Santa Rita
Oi Claudio !

Nao ha do que agradecer, mas a sua iniciativa nos motiva a prosseguir
: as vezes passa pela minha cabeca que ninguem se interessa por estas
solucoes e que estou apenas "enchendo o saco" dos membros da lista.
Quando algumas pessoas, como  voce fez, se manifesta, nos vemos que o
nosso trabalho esta sendo util e a duvida se dissipa.

A Matematica e Universal e patrimonio de toda a Humanidade. Assim, EU
PENSO que, sempre que possivel, devemos divulgar livremente aquilo que
sabemos, sobretudo quando trata-se de solucoes de um tema CUJOS
FUNDAMENTO ja e amplamente dominado. Estas solucoes ajudam os
estudantes a descobrirem MODELOS DE ATAQUE a outros problemas
similares.

Para que esta mensagem nao seja inteiramente pessoal, aqui vai a
solucao do problema 4.12

( EXERCICIO 4.12 )

Doravante, sempre que precisar usar somatorios, vou adotar a notacao :

Si[1,N : F(i)] = F(1) + F(2) + ... + F(N)

Seja r um numero real,  0 < r < 1. Fazendo b=r*a, temos que 0 < b < a.
Ao real F = a - b > 0 correspondera um N0 tal que n > N0 implica | Xn
– a | < F, pois LIM Xn= a. Daqui seguira que para n > N0, a – F < Xn,
vale dizer, n> N0 => b < Xn. Agora, usando as propriedades dos numeros
reais, e facil ver que para quaisquer naturais POSITIVOS "i" e K ( n >
N0 ) :

0 < b^(i/K) < (Xn)^(i/K)  =>   0 < (b^(i/K))*(a^(K-i-1) ) < (
(Xn)^(i/K) )*(a^(K-i-1) )

Logo : 0 < Si[0,K- 1:(b^(i/K))*(a^(K-i-1) )]  <
Si[0,K-1:((Xn)^(i/K))*(a^(K-i-1) )]   =>
0 < | Si[0,K- 1:(b^(i/K))*(a^(K-i-1) )]  |  <  |
Si[0,K-1:((Xn)^(i/K))*(a^(K-i-1) )]  |

Fazendo c = | Si[0,K- 1:(b^(i/K))*(a^(K-i-1) )]  | e multiplicando
tudo por |(Xn)^(1/K)–a^(1/K)|, segue :

( DESIGUALDADE 1 )
|(Xn)^(1/K)–a^(1/K)|*c < | (Xn)^(1/K)–a^(1/K) |*|
Si[0,K-1:((Xn)^(i/K))*(a^(K-i-1) )] |=|Xn-a|

Dado um E > 0

( DESIGUALDADE 2 ) :
Como LIM Xn=a, existe um N1 tal que n > N1 => | Xn-a| < c*E

Vemos portanto que se tomarmos um N2=max{N0,N1}, para todo n > N2 as
duas desigualdades ficarao satisfeitas. Usando a transitividade das
desigualdades chegamos a :

n > N2 => |(Xn)^(1/K)–a^(1/K)| < E

Assim, para um E > 0 qualquer sempre podemos exibir um natural N2 tal
que n > N2 implica que |(Xn)^(1/K)–a^(1/K)| < E. Isto estabelece que
LIM (Xn)^(1/K) = a^(1/K) , como queriamos demonstrar.

***

Seja LIM Xn=a  e  r=P/Q. Já sabemos que  LIM (Xn)^(1/Q) = a^(1/Q). Mas :
LIM (Xn)^(P/Q) = LIM [ (Xn)^(1/Q)*...*(Xn)^(1/Q) ] onde dentro dos
colchetes há P fatores. Logo :
LIM (Xn)^(P/Q) = LIM (Xn)^(1/Q)*LIM (Xn)^(1/Q)*...*LIM (Xn)^(1/Q)
LIM (Xn)^(P/Q) = a^(1/Q)*a^(1/Q)* ... *a^(1/Q) = a^(P/Q)

O caso P/Q < 0 vai seguir a mesma linha, bastando eliminar o sinal de "-"

OBS : Quem se interessar pelos exercicios dos capitulos 1 e 2 eu
enviei uma copia do arquivo com todas as solucoes para o "Tio Cabri".
Ele zipou o arquivo e colocou num ciber-lugar. E so falar com ele. Eu
tenho as solucoes de todas as questoes, conforme for encontrando, vou
disponibilizando pra voces. Enquanto isso vou publicando as solucoes
aqui. Sejam magnanimos e mostrem e expliquem as solucoes para os seus
colegas.

Um Abracao a Todos
Paulo Santa Rita
6,0630,040408



2008/4/3 Claudio Gustavo <[EMAIL PROTECTED]>:
>   Oi Paulo.
>   Estou respondendo essa mensagem apenas pra agradecer sua iniciativa. Pois
> essas soluções tenho certeza que ajudarão a muitos outros além de mim.
>
>   Abraços,
> Claudio Gustavo.
>
> Paulo Santa Rita <[EMAIL PROTECTED]> escreveu:
>
> Ola Pessoal,
>
> Tenho publicado algumas solucoes de exercicios de Analise retirados do
> excelente Livro :
>
> Curso de Analise - Volume 1 - Projeto Euclides - IMPA
> 11 edicao - 2 impressao
> Autor : Elon Lages Lima
>
> Eu nao publiquei nenhuma solucao dos capitulos 1, 2 e 3 porque sao
> assuntos que, em geral, o estudante ja viu em cursos anteriores,
> principalmente na Graduacao. Alem disso, eles sao bastante simples.
> Entretanto, algumas pessoas me escreveram "em off" e pediram que eu
> publicasse 2 ou 3 solucoes desses capitulos. Assim, selecionei 3
> exercicios do capitulo 1 e estou publicando agora. Escolhi os que
> achei mais interessantes ou/e desafiadore. Seguem as solucoes :
>
>
>
>
> NOTACAO : A letra "lambda" sera representada nestes exercicios por
> "m". Os simbolos de uniao e intersecao serao representados
> respectivamente pelos prefixos UNI e INTER. Os simbolos ">" e "<"
> representarao, respectivamente, "contem" e "esta contido". O Simbolo
> de "pertence a" sera representado pela letra "E" e "f_a" representa a
> letra "f" com indice "a". A barra "/" representara a expressao "tal
> que"
>
>
>
> ( EXERCICIO 1.14)
>
>

  1   2   3   4   5   6   7   8   >